101s1.downloadmienphi.net/file/downloadfile2/180/1403331.doc · web viewduring normal office hours,...

221
Page 1

Upload: lylien

Post on 07-Mar-2018

216 views

Category:

Documents


2 download

TRANSCRIPT

Page 1: 101s1.downloadmienphi.net/file/downloadfile2/180/1403331.doc · Web viewDuring normal office hours, University Maintenance will respond directly to a power failure. After hours, the

Page 1

Page 2: 101s1.downloadmienphi.net/file/downloadfile2/180/1403331.doc · Web viewDuring normal office hours, University Maintenance will respond directly to a power failure. After hours, the

TOEIC TEST

New formatReading

Page 2

Page 3: 101s1.downloadmienphi.net/file/downloadfile2/180/1403331.doc · Web viewDuring normal office hours, University Maintenance will respond directly to a power failure. After hours, the

101. Residents began to ------- the area after hearing news the volcano would erupt. (a) gain(b) remake(c) commute(d) evacuate

102. Analysts ------- the country improve its infrastructure to get foreign investment. (a) suggested(b) suggest to(c) suggesting(d) were suggested

103. In accordance with market trends, food and drink companies ------- healthier images. (a) looked(b) sought(c) helped(d) believed

104. With a wide -------, Mr. Gonzales was re-elected president of the South American country.

(a) term(b) voter

(c) margin(d) border

105. The airline avoided a strike by ------- with the

pilots’ union. (a) negotiate

(b) negotiates(c) negotiating

(d) negotiation106. The report showed the U.S. trade deficit

------- in February. (a) shrink(b) shrunk(c) shrunken(d) shrinking

107. Union members ------- the privatization of the country’s airports. (a) replied(b) annoyed(c) protested(d) identified

108. Scientists ------- new ways to attack the fungus damaging the rice fields. (a) develops(b) to develop(c) were developed(d) are developing

109. The controversial film was nominated ------- eight Academy Awards. (a) of(b) for(c) with(d) about

110. The advertising executive resigned over his inappropriate ------- made at a conference. (a) dress(b) acting(c) welcome(d) remarks

111. About 30,000 coffeemakers were ------- after the company received numerous reports of overheating.a) released(b) recalled(c) reentered(d) refigured

112. Pest control was called to ------- the office over the holiday weekend. (a) come(b) treat(c) makes(d) attend

113. Fears of another interest ------- caused stocks to further decline. (a) hike(b) hikes(c) hiked(d) hiking

Page 3

Page 4: 101s1.downloadmienphi.net/file/downloadfile2/180/1403331.doc · Web viewDuring normal office hours, University Maintenance will respond directly to a power failure. After hours, the

114. Commercial logging and overgrazing made the land less compact and ------- to landslides.(a) ready(b) defensive(c) vulnerable(d) threatening

115. In a good year, Mr. Issa could ------- eight tons of cocoa. (a) apply(b) reduce(c) succeed(d) harvest

116. Shareholders voted to have more influence ------- the executive board’s decisions. (a) over(b) from(c) into(d) above

117. Parliament passed a law that would ban the production and sale of genetically ------- food. (a) modify(b) modified(c) modifies(d) modification

118. The company chairman was ------- the acquisition would have a positive financial start. (a) carefree(b) confident

(c) concerned(d) questioned

119. Although the civil war ended four years ago,

democratic ------- have yet to be held. (a) mistakes(b) campaign(c) elections(d) following

120. New packaging helped Green Plus ------- market its already well-known soft drinks. (a) success(b) successes(c) successful(d) successfully

121. According to the satellite survey, the Amazon rainforest is ------- twice as fast as previously estimated. (a) applying(b) removing(c) depleting(d) renovating

122. Increasing energy stocks and a rise in foreign investments ------- to the rise in hedge funds. (a) helped(b) assisting(c) recognizing(d) contributed

123. The celebrity couple sold pictures of their baby for $5 million, which ------- to charity. (a) donated(b) was donated(c) is donation(d) had donated

124. The retiring editor named the senior reporter as his --------. (a) success(b) successor(c) successes(d) successful125. A boom in ethanol research and manufacturing ------- jobs throughout small towns across the nation. (a) create(b) created(c) creating(d) creation126. The position requires all candidates to speak English ------- to another European language. (a) addition(b) in addition(c) of addition(d) additionally

127. Forrest, Inc. increased its second quarter profit and forecast ------- a large tax settlement. (a) after(b) being(c) within(d) because

Page 4

Page 5: 101s1.downloadmienphi.net/file/downloadfile2/180/1403331.doc · Web viewDuring normal office hours, University Maintenance will respond directly to a power failure. After hours, the

128. Continual ------- of trademarks and copyrights increased trade tension in Asia. (a) violence(b) writings(c) violations(d) understanding

129. The European country unexpectedly raised interest rates in ------- of inflation.(a) fear(b) fears(c) feared(d) fearing

130. Owners of Movie Place, a popular movie-themed restaurant, announced a new restaurant ------- in Sacramento this summer. (a) opening(b) to open(c) would open(d) for opening

131. The oil giant will invest nearly £300 million to research renewable energy --------. (a) choice(b) section(c) portions(d) alternatives

132. Although he was widely ignored, the lawmaker ------- his colleagues to pass bills supporting energy alternatives to fight rising oil prices and diminishing reserves.(a) set(b) urged(c) tried(d) defended

133. A large portion of the donation was spent ------- new ways to fight malaria. (a) on promoting

(b) in promotion(c) of promoting

(d) for promotion

134. Future Bank will start offering special incentives to employees who ------- hybrid cars

starting September 1. (a) purchase(b) purchases(c) purchased(d) purchasing135. The poem was as beautiful in its English ------- as it was in its original German form. (a) translate(b) translated(c) translating(d) translation

136. After three weeks of -------, the stock market finally closed at a half percent increase. (a) unsteady(b) weakening(c) changeable(d) instability137. City officials hope the new policy will make offenders ------- accountable. (a) some(b) more(c) also(d) with

138. Although it is not ------- on the menu, Taco Town occasionally offers homemade tamales. (a) listed(b) checked(c) placing(d) showing

139. Sarah and Jonathan will meet this Wednesday ------- their presentation. (a) rehearse(b) rehearsing(c) to rehearse(d) for rehearsing

140. Graduate students went on strike after losing their union -------. (a) represent(b) represents(c) represented(d) representation

Page 5

Page 6: 101s1.downloadmienphi.net/file/downloadfile2/180/1403331.doc · Web viewDuring normal office hours, University Maintenance will respond directly to a power failure. After hours, the

Questions 141-143 refer to the following letter.

Feel the rush of sophistication and class!

The all-new Speedlite X-Class will drive you to a world you have _____ of. 141. (a) met

(b) wanted(c) created(d) dreamed

With more power, handling, and style, the Speedlite X-Class is a revolution _____ auto making. 142. (a) on

(b) in(c) to(d) up

"Motor Trend magazine called it, “Car of the Year.”The Speedlite X-Class _____ equipped with power steering, 5-speed manual transmission, and side curtain airbags.

143. (a) came(b) come (c) comes(d) coming

As a bonus, it also features a satellite navigation system and a built-in DVD player. Change your world today, visit your local Speedlite dealer.

Questions 144-146 refer to the following letter.

Hello everyone,

As suggested _____ human resources, we will be conducting our first diversity workshop next Monday.144. (a) by

(b) into(c) after(d) before

The purpose of the workshop is to build _____ about cultural issues.145. (a) awareness

(b) recognize(c) understand(d) relationship

Staff can sign up for the workshop at the Human Resources Office. Registration is _____.146. (a) curious

(b) original (c) mandatory(d) interesting

Page 7: 101s1.downloadmienphi.net/file/downloadfile2/180/1403331.doc · Web viewDuring normal office hours, University Maintenance will respond directly to a power failure. After hours, the

Thank you for your cooperation and understanding.

Frank MyersDirector of Human Resources.

Questions 147-149 refer to the following letter.Calling all Koi lovers!

The GMAC Convention Center will _____ a Koi Convention May 22-25 from 9-9. Admission is $10. 147. (a) holding

(b) have held(c) be holding(d) been holding

There will be a variety of Koi from Japan on _____ with more than a dozen varieties and sizes of Koi on sale.148. (a) display

(b) showing(c) present(d) available

A special Koi breeding class will be held on Sunday at 2 p.m. Class is _____ to 15 participants. 149. (a) restrict

(b) restricts(c) restricted(d) restricting

To sign up for the workshops or for more information, please visit www.gmac.com/koi.

Questions 150-152 refer to the following letter.Tour Russia’s “Window to the West”

Looking for an exotic vacation? Rich in both culture and history, St. Petersburg showcases the _____ of Russia in one beautiful location.

150. (a) diverse(b) diversity(c) diversify(d) diversification

_____ by Peter the Great 300 years ago, St. Petersburg is now the second largest city in Russia.151. (a) Known

(b) Looked(c) Founded(d) Traveled

Stroll along the main avenue of Nevsky Prospekt, visit The Hermitage museum and czarist castles, or attend a ballet or orchestra _____ at famed Mariinsky Theater.

152. (a) ballroom(b) situation(c) literature

Page 8: 101s1.downloadmienphi.net/file/downloadfile2/180/1403331.doc · Web viewDuring normal office hours, University Maintenance will respond directly to a power failure. After hours, the

(d) performanceCruiseships on the canals of the Neva River also offer a spectacular view of the city.For information and pricing, please visit www.etravelteam.com/russia.

From: Russell Glanz [[email protected]]To: Leslie Bonham [[email protected]]Subject: Monroe Park

Dear Mayor Bonham:

I am writing in regard to the proposed law banning the feeding of pigeons in Monroe Park.I would like to express my support for this legislation. I do not dislike pigeons but I feel that they are unsanitary and a nuisance.I love Monroe Park but have been forced to stay away from the area. I used to go running and have picnics in the park, but the pigeon population is out of hand and it is nearly impossible to do anything without a pigeon getting in the way. Last time my family and I had a picnic, a pigeon kept landing on our table and coming back despite our efforts to make it leave.People must stop feeding these birds. If this legislation passes, I believe park conditions will be overall improved. It will be cleaner and more pleasant.

Thank you for your time and your concern for our city.

Sincerely,

Russell Glanz

153. What is the main purpose of this e-mail?(a) To complain about construction(b) To thank the mayor for his support(c) To express support for a proposed law(d) To persuade the mayor to change his mind

154. What does Mr. Glanz recall?(a) Having a picnic with his family(b) Cleaner streets and less traffic(c) Leaving for work at a later time(d) Reading by the fountain in the park.

From: Jack Bates [[email protected]]To: Sales staffSubject: Tuesday

I just got word that James Macy, Vice President of Sales, will be visiting the store next Tuesday. He will be here to check overall and individual sales, discuss strategy, give tips, and have lunch with us.

I would like everyone to work hard until then and think about what you’re doing right and/or wrong. Mr. Macy is very down-to-earth and friendly. Relax, act natural, be professional, and everything will be fine.

I’m sure Mr. Macy will give great insight into how to improve our sales. Although our sales are slightly above our goals, there is always room for improvement.

Page 9: 101s1.downloadmienphi.net/file/downloadfile2/180/1403331.doc · Web viewDuring normal office hours, University Maintenance will respond directly to a power failure. After hours, the

Regards,

Jim Bates

155. What is the main purpose of the e-mail?(a) To request a meeting(b) To announce a visitor(c) To announce sales goals(d) To discuss individual sales

156. What advice does Mr. Bates give to his employees? (a) Be prompt(b) Be natural(c) Impress your clients(d) Smile and be friendly.

Search Results: 3 out of 10

REAL ESTATE > NEW YORK > $300,000-$500,000

Listing ID #2370-49838

Type: House

Location: Near Salem, NY

Bedrooms: 4

Bathrooms: 2

Realtor: Bethany Windsor

Contact: 1-888-234-1938 ext. 35

Features: Hardwood floors, high ceilings, fireplace, garden patio, second floor terrace

This secluded property is surrounded by trees and has a small pond. It has a newly painted interior, and new storm windows. Situated by Vermont ski areas, this house is a great vacation house.

Click here for a virtual tour

157. What can be concluded about the house?(a) It is in Vermont.(b) It has two stories.(c) It is made of brick.(d) It is in the country.158. What is NOT a feature of the house? (a) A fireplace(b) A small pond

(c) A large garden(d) Hardwood floors.

159. What has been done to improve the house? (a) The deck was repainted.(b) New storm windows were added.(c) Ceiling fans were installed.(d) The front door has been replaced.

Page 10: 101s1.downloadmienphi.net/file/downloadfile2/180/1403331.doc · Web viewDuring normal office hours, University Maintenance will respond directly to a power failure. After hours, the

Attention Merring Employees:

Merring & Co. believes investing time in employees and family is as important as investing in the stock market. Without our wonderful, hard-working employees, our business would be nothing.

To show our appreciation, Merring & Co. will be hosting an employee golf tournament on Saturday, May 3 at Yorkshire Golf Course beginning at 9 a.m. A cash award of $1,000 will go to the winner.

Registration is from March 10-17. Forms are available online or in the General Affairs Department. Registration is $3. Immediate family members are also invited to participate in the tournament.

In addition to the golf tournament, there will be a children’s putting contest at the Yorkshire Mini-Putt. A barbecue will follow the main tournament with refreshments provided.

Please bring your family and join the Merring family for a fun-filled day. For more information, visit www.merring.com/event/employeegolf.

160. What is compared with the importance of employees and family?(a) Golf(b) Friends(c) Appreciation(d) The stock market.161. When will the tournament occur?(a) On March 10(b) On March 17

(c) On May 3(d) On May 9.

162. What will happen after the tournament? (a) A dance(b) A barbecue dinner(c) An official awards ceremony(d) A children’s putting contest.

Michelle Kahn126 N. Clark Street Suite 200Chicago, IL [email protected]’s Woodworks912 Cruse AvenueHelena, MT 59601

Dear Mr. Cruz:

I recently visited a friend who had her cabinets custom-made by your company. I loved the craftsmanship and detail of the cabinets and would like to know more about the services you offer.My mother’s birthday is in two months and I would like to replace her coffee table with something special.Although your company is based in Montana, I was wondering if you would be able to deliver to Chicago. If you would, please send me a catalog or some company brochures about the work your company offers. I’m looking for something elegant.Thank you for your time and help. I look forward to hearing from you.

Page 11: 101s1.downloadmienphi.net/file/downloadfile2/180/1403331.doc · Web viewDuring normal office hours, University Maintenance will respond directly to a power failure. After hours, the

Sincerely,Michelle Kahn

163. How did Ms. Kahn hear about Gilbert’s Wordworks?(a) From a friend(b) From a billboard(c) From a commercial(d) From a newspaper article.

164. What does Ms. Kahn want to have made?(a) A desk

(b) A bookshelf(c) Some cabinets(d) A coffee table

165. What best describes the style Ms. Kahn prefers?(a) Modern(b) Homely(c) Practical(d) Sophisticated.

Byron rejects $29.5 billion offer

Drugmaker Byron rejected a $29.5 billion hostile takeover bid by Crotrex after a long meeting at Byron’s company headquarters in Baltimore.

Calling the bid inadequate after beating market expectations for two consecutive quarters, Byron’s chief Floyd Lehman hinted the company might be willing to deal “if Crotrex doesn’t undervalue Byron,” which is currently worth about $39 a share.

Yet a spokesman for Crotrex said it would be unwilling to negotiate the price but would consider changing other terms of the agreement. Talks are expected to resume next week.

166. What does the article mainly discuss?(a) A possible merger(b) A drop in share prices(c) A hostile takeover bid(d) A announcement of plant closures

167. What can be concluded about Byron? (a) It is based in Baltimore.(b) It manufactures automobiles.

(c) The CEO is unwilling to negotiate.(d) It did not meet market expectations.

168. What is the problem? (a) The price offered was too low.(b) Competition has been increasing.(c) Union members have been on strike.(d) Company shares have continued to fall.

Page 12: 101s1.downloadmienphi.net/file/downloadfile2/180/1403331.doc · Web viewDuring normal office hours, University Maintenance will respond directly to a power failure. After hours, the
Page 13: 101s1.downloadmienphi.net/file/downloadfile2/180/1403331.doc · Web viewDuring normal office hours, University Maintenance will respond directly to a power failure. After hours, the

169. What did Mr. Reilly most like about the store?(a) Its appearance(b) Its management(c) Its customer service(d) Its product selection

170. What did Mr. Reilly purchase?(a) Headphones(b) A car stereo(c) An mp3 player(d) A satellite radio.

171. How did Mr. Reilly feel before making his purchase? (a) Reluctant(b) Confident(c) Satisfied(d) Disagreeable.

172. Who is John H.?(a) A store manager(b) Mr. Reilly’s friend(c) A staff in audio dept.(d) A customer service representative.

Page 14: 101s1.downloadmienphi.net/file/downloadfile2/180/1403331.doc · Web viewDuring normal office hours, University Maintenance will respond directly to a power failure. After hours, the
Page 15: 101s1.downloadmienphi.net/file/downloadfile2/180/1403331.doc · Web viewDuring normal office hours, University Maintenance will respond directly to a power failure. After hours, the

173. What did Ms. Henderson order?(a) CDs(b) DVDs(c) Books(d) Magazines.

174. How many items were ordered? (a) Two(b) Three(c) Four(d) Five

175. How much must Ms. Henderson pay?(a) $17.99(b) $42.97(c) $49.97(d) $52.97.

176. What is NOT true about Ms. Henderson’s order? (a) It is a gift. (b) It will be shipped to Italy.(c) It qualified for a discount.(d) It was ordered from a catalog.

Page 16: 101s1.downloadmienphi.net/file/downloadfile2/180/1403331.doc · Web viewDuring normal office hours, University Maintenance will respond directly to a power failure. After hours, the

Playtime makes Brainy Babies

By Karen Viera

Playtime, Inc., a leading maker of infant and toddler products, announced a new division including books and teaching materials called Baby School.

Marketed as a baby’s first experience with learning, the division is aimed at the growing number of parents who enroll their children in educational centers at and sometimes before the age of two.

Ranging from numbers to animals and even foreign languages, Baby School Books will include titles from most subjects. “We came up with the idea after hearing about basic foreign language classes attended by parents with their infants,” said Playtime Planning Executive Robert R. Hershire. “Studies show that educating children at their earliest years can help increase scores and possibly boost IQs. Parents are responding positively, which is encouraging.”

Baby School’s first publications will include a book of beginning words in phrases in Spanish, French, and German. There will also be a book of numbers and beginning math as well as introductory science, which will include topics such as animals and space.

“We’re confident we will get a positive response based on the interest we’ve already experienced,” said Hershire referring to high orders from book and baby stores. Baby School products will debut this Friday.

177. What is Playtime?(a) A new school(b) A children’s game(c) A children’s store(d) A product manufacturer.

178. Who is Karen Viera? (a) A parent(b) A teacher(c) A reporter(d) An executive.

179. What might be included in Baby School Books?(a) A lesson about planets(b) Tips on how to discipline(c) An introduction of Japanese words(d) Advice on how to cook for children.

180. The word “debut” in paragraph 5, line 3 is closest in meaning to:(a) come(b) unveil(c) present(d) promote.

SEE CHICAGO!

I have two orchestra-side tickets for Saturday, July 30 for the musical Chicago but have to leave town for business. The show is at the Ambassador Theatre on 49th Street in Manhattan. Tickets are $175, originally $230, but willing to negotiate. Please call Phil at (917) 690-9879--------------------------------------------------------------------------------------------------------------------------------From: Amelia Deco [[email protected]]To: Geneva Herrera [[email protected]]

Page 17: 101s1.downloadmienphi.net/file/downloadfile2/180/1403331.doc · Web viewDuring normal office hours, University Maintenance will respond directly to a power failure. After hours, the

Subject: Saturday

Geneva,

How have you been? Sorry I had to cancel our dinner on Friday. I’ve been so busy with work I haven’t had time to do anything.

Anyway, I’d like to make it up to you. I just bought tickets for Chicago this Saturday. It starts at 8 p.m. and the seats are great. We could have an early dinner before we go. It’ll be my treat.

Please let me know if you’re interested as soon as possible so I can make reservations somewhere. I’d love to catch up and see how you’re doing. You can reach me at my office.

I look forward to seeing you!

Amelia

181. What is being advertised?(a) An air ticket(b) Ballet tickets(c) Musical tickets(d) Baseball tickets.

182. How much did Phil probably pay?(a) $49(b) $175(c) $230(d) $300.183. What does Ms. Deco want to do on Saturday at 8 p.m? (a) See Chicago

(b) Have dinner(c) Pick up Ms. Deco(d) Make reservations

184. What is implied from the e-mail? (a) Ms. Herrera has been working a lot.(b) Ms. Herrera enjoys performance art.(c) The women were supposed to meet on Saturday.(d) The women have not seen each other in a while

185. Where can Ms. Deco be reached? (a) At her home(b) At her office(c) At the theater(d) On her mobile phone.

To: Natalie Johnson [[email protected]]From: [email protected]: Grand Opening Sale

Dear Valued Customer:

Evers Grocery would like to invite you to the grand opening of our new store at 33rd Avenue and Highway 26 this weekend starting Friday at 6 a.m. Our new store will be open 24 hours a day, with the exception of holidays, for your convenience.

We’ve taken everything good about Evers and made it better. Our new store has an expanded produce section, a larger bakery, a new deli, and even a food court. Don’t forget to visit our natural foods department featuring organic and vegetarian food and products.

Customers can also enter a drawing each day to win $500 worth of groceries. Drawings will be held twice a day with winners announced at noon and 5 p.m.

Page 18: 101s1.downloadmienphi.net/file/downloadfile2/180/1403331.doc · Web viewDuring normal office hours, University Maintenance will respond directly to a power failure. After hours, the

Please visit our new location this weekend and enjoy special savings. You can also receive an additional 10 percent off your final bill with our special coupon for preferred customers. Just print it out and give it to the cashier.

Thank you for shopping with Every Grocery!

Sincerely,

Lee HammondDirector Customer Service.------------------------------------------------------------------------------------------------------------------------------

186. Why is the store considered convenient?(a) It closes only on holidays.(b) It is in the center of the city.(c) It has a bigger produce section.(d) It is the biggest store in the city.

187. What is NOT a new department at Evers Grocery?(a) The Deli(b) A sushi Bar(c) A food Court(d) Natural Foods.

188. When will the drawing be held?

(a) Once per day(b) Twice per day(c) On Saturday morning(d) On Sunday night.

189. What can be implied about Ms. Johnson?(a) She has a large family.(b) She shops every Sunday.(c) She frequently uses coupons.(d) She has shopped at Evers in the past.

190. When is the store opening?(a) In July(b) In August(c) In September(d) In October.

From: Linda Eriksson [[email protected]]To: Lars Alexandersson [[email protected]]Subject: Cruise invoice and itineraryAttachment: alexandersson.doc

Dear Mr. Alexandersson,

Scandinavia Cruises would like to thank you for your patronage. As a returning customer, we have given you the

Page 19: 101s1.downloadmienphi.net/file/downloadfile2/180/1403331.doc · Web viewDuring normal office hours, University Maintenance will respond directly to a power failure. After hours, the

Captain’s Special rate and upgraded your status to Gold, which upgrades your deck and saves you about $4,000.

Captain’s Special | Gold Customer | Suite Deck B5

Passenger(s)Lars Alexandersson $9,645.00Marie Alexandersson $9,645.00

Subtotal $19,290.00Tax $648.00------------------------Total $19,938.00

Attached is your 12-day itinerary. Please save this information for your records. We hope you enjoy your cruise and look forward to serving you in the future.

Sincerely,

Linda ErikssonCruise designer

*********Scandinavia CruisesStrand Boulevarden 262Copenhagen2100 Denmark+45-3543-7900------------------------------------------------------------------------------------------------------------------------------

Page 20: 101s1.downloadmienphi.net/file/downloadfile2/180/1403331.doc · Web viewDuring normal office hours, University Maintenance will respond directly to a power failure. After hours, the

191. How much is the base-price of the cruise per person?(a) $648(b) $4,000(c) $9,645(d) $19,290.

192. Why is Ms. Eriksson offering a special rate?(a) Mr. Alexandersson is a friend.(b) Mr. Alexandersson is a former colleague.(c) Mr. Alexandersson is a frequent customer.(d) Mr. Alexandersson is an executive at Halmstad Limited.

193. What does Ms. Eriksson ask of Mr. Alexandersson? (a) To pay within 12 days

(b) To confirm his registration(c) To pick up his new membership card(d) To keep the information for his records.

194. How long with Mr. Alexandersson be in Tallin?(a) For one day(b) For two days(c) For three days(d) For four days.

195. What country will NOT be visited?(a) Sweden(b) Russia(c) Germany(d) Denmark.

From: Emily Eastwood [[email protected]]To: Arthur DeWitt [[email protected]]Subject: AdvertisementAttachment: forkids.psd

Dear Mr. DeWitt,

Thank you for your help on Thursday. It was nice meeting you.I'd like to submit my advertisement to run for two weeks from September 2-17. As you said, the discounted price for

Page 21: 101s1.downloadmienphi.net/file/downloadfile2/180/1403331.doc · Web viewDuring normal office hours, University Maintenance will respond directly to a power failure. After hours, the

non-profit organizations for a quarter page, two-week ad is $353.80. Please send me an invoice so I can send you a check.

Attached is our advertisement in a Photoshop file format. Please let me know if you have any problems with the ad. I would like to update it next week as the show approaches.I'm looking forward to seeing our ad next Friday. If you are available on Sunday, Sept. 17, I'd like to invite you to our show.

Have a great week.

Yours truly,Emily Eastwood------------------------------------------------------------------------------------------------------------------------------Children’s Fall Fashion Show*Sunday, September 17 at 4 p.m.

City Galleria Lobby

Proceeds will benefit the Children’s HospitalSponsored by For the Kids

*If your child would like to volunteer as a model, please contact Emily at 498-2983 for more information.

196. What can be NOT concluded from the e-mail?(a) Mr. DeWitt and Ms. Eastwood have met.(b) Mr. DeWitt works for the City Gazette.(c) Ms. Eastwood works for a non-profit organization.(d) Mr. DeWitt will design an advertisement for Ms. Eastwood.

197. How large will Ms. Eastwood’s advertisement be?(a) 1/8 page(b) 1/4 page(c) 1/2 page(d) A full page.

198. What will happen the following week?(a) Mr. DeWitt will send an invoice.

(b) Ms. Eastwood will visit Mr. Dewitt.(c) Ms. Eastwood will change the advertisement.(d) Ms. Eastwood will approve the advertisement.

199. What is being advertised?(a) A fashion show(b) A model search(c) A casting call(d) A children’s play.

200. Where will the money be donated?(a) To an orphanage(b) To For The Kids(c) To a children’s hospital(d) To the elementary school.

ANSWER101. D 102. A 103. B 104. C 105. C 106. B 107. C 108. D 109. B 110. D111. B 112. B 113. A 114. C 115. D 116. A 117. B 118. B 119. C 120. D121. C 122. D 123. B 124. B 125. B 126. B 127. A 128. C 129. A 130. C131. D 132. B 133. A 134. A 135. D 136. D 137. B 138. A 139. C 140. D141. D 142. B 143. C 144. A 145. A 146. C 147. C 148. A 149. C 150. B151. C 152. D 153. C 154. A 155. B 156. B 157. D 158. C 159. B 160. D161. C 162. B 163. A 164. D 165. D 166. C 167. A 168. A 169. D 170. A171. A 172. C 173. B 174. B 175. B 176. D 177. D 178. C 179. A 180. B

Page 22: 101s1.downloadmienphi.net/file/downloadfile2/180/1403331.doc · Web viewDuring normal office hours, University Maintenance will respond directly to a power failure. After hours, the

181. C 182. C 183. A 184. D 185. B 186. A 187. B 188. B 189. D 190. B191. C 192. C 193. D 194. A 195. C 196. D 197. B 198. C 199. A 200. C

Page 23: 101s1.downloadmienphi.net/file/downloadfile2/180/1403331.doc · Web viewDuring normal office hours, University Maintenance will respond directly to a power failure. After hours, the

101. For prices, please consult the ------- catalogue. (a) enclose(b) enclosed(c) encloses(d) enclosure

102. Payment should be sent by bank ------- in dollars. (a) draft(b) money(c) cashier(d) transport

103. The warehouse will ------- open during the maintenance work. (a) pause(b) delay(c) allow(d) remain

104. Attendance at the meeting on the 25th is ------.

(a) attended(b) mandatory

(c) interested(d) requesting

105. All visitors must report to ------- on arrival.

(a) secure(b) secured

(c) secures(d) security

106. Tickets will go ------- sale four weeks before the

performance. (a) at(b) to(c) on(d) in

107. One percent of the cost of construction has been ------- to the purchase of artworks for the lobby. (a) divided(b) suggested(c) allocated(d) decorated

108. Critics of the plan fear the new regulations will be impossible to ------. (a) advise

(b) enforce(c) increase(d) persuade

109. It is hoped that the new packaging will be more ------- to prospective purchasers. a) strong(b) colorful(c) appealing(d) different

110. Because of the costs, the board is ------- to approve the measures. (a) powerful(b) reluctant(c) difficult(d) expensive

111. For more information, ------- our website www.bigbusiness.com.(a) look(b) show(c) check(d) offer

112. The movement of the control knobs on the prototype is not yet ------- enough. (a) smooth(b) smoother(c) smoothly(d) smoothing

113. The new painting will ------- in the boardroom. (a) hang(b) place(c) enjoy(d) improve

114. ------- January 1st, trash will be collected on Tuesdays.(a) Begin(b) Began(c) Begins(d) Beginning

115. We are interested in becoming the North American ------- for your products, and would to like to arrange a meeting to discuss this in detail. (a) represent(b) represents(c) to represent(d) representative

Page 24: 101s1.downloadmienphi.net/file/downloadfile2/180/1403331.doc · Web viewDuring normal office hours, University Maintenance will respond directly to a power failure. After hours, the

116. A loose electrical ------- caused the equipment to overheat. (a) error(b) worker(c) operation(d) connection

117. The price of copper reached an all-time ------- last week. (a) top(b) record(c) output(d) mining

118. This product is shipped in 10-gallon -------. (a) drums(b) boats

(c) supply(d) postage

119. The court decided that the advertisement does not

------- our products. (a) mistake(b) misapply(c) misfortune(d) misrepresent

120. We have a dedicated export department for ------- sales. (a) abroad(b) overseas(c) stranger(d) foreigner

121. The contract states that we retain ------- of the architect’s plans. (a) priority(b) building(c) ownership(d) application

122. The documents have been ------- by a failure to maintain proper storage conditions. (a) sent(b) ruined(c) ordered(d) discounted

123. The bank returned the ------- because it lacked an authorizing signature. (a) cash(b) check(c) coins(d) money

124. Use of third-party accessories with this product will ------- the warranty. (a) void(b) destroy(c) disable(d) validate

125. Upon opening, confirm contents of this box ------- the enclosed parts list. (a) upon(b) within(c) toward(d) against

126. A 30-horsepower motor is available as an optional --------. (a) energy(b) upgrade(c) increase(d) reinforcement

127. The restaurant will be closed ------- the health code violations are addressed. (a) for(b) until(c) during(d) through

128. Two-thirds of the university’s incoming ------- class is from out of state. (a) grade(b) tuition(c) freshman(d) graduated

129. The ------- new pipeline would transport 300,000 barrels of oil per day.(a) propose(b) proposed(c) proposal(d) proposing

130. Vancouver-based M-Corp has pledged $5 million to help create ------- for the assembly of the aircraft.

Page 25: 101s1.downloadmienphi.net/file/downloadfile2/180/1403331.doc · Web viewDuring normal office hours, University Maintenance will respond directly to a power failure. After hours, the

(a) pilot(b) machine(c) software(d) workforce

131. The Crossroads Shopping Plaza has had a 30 percent ------- rate for the past two years. (a) store(b) price(c) credit(d) vacancy

132. Troe Industries will move its production to ------- in Tuscaloosa, Alabama in 2007.(a) profits(b) products(c) factories(d) customers

133. On Wednesday, the company announced layoffs that will affect nearly 7 percent of its 8,700 ------- worldwide. (a) awards

(b) stories(c) catalogs

(d) employees

134. The city has a history of ------- first-rate sporting events.

(a) hosting(b) beating(c) grading(d) visiting

135. The market chain claims that it ------- some 5,000 new products to its shelves annually. (a) adds

(b) hires(c) sells(d) replaces

136. ------- the change in management, losses, in terms of quarterly net profit, have fallen dramatically. (a) If(b) By(c) Since(d) Because

137. Indirect employment adds nearly 15,000 more jobs to those ------- by foreign companies in the state. (a) used(b) moved(c) worked(d) provided

138. Instead of playing safe with its repertoire, City Opera has continued to ------- risks. (a) do(b) take(c) have(d) get

139. The ability to program in both C and FORTRAN would be ------- for this position. (a) an asset(b) a salary(c) a manager(d) an employee

140. Thursday’s ------- of Tim Reynolds as CEO ends Seybol Inc.’s seven-month search for a top executive. (a) appointed(b) to appoint(c) appointing(d) appointment

Page 26: 101s1.downloadmienphi.net/file/downloadfile2/180/1403331.doc · Web viewDuring normal office hours, University Maintenance will respond directly to a power failure. After hours, the

Questions 141-144 refer to the following letter.

Feathers Flying

During the four quarters of 2005, Feather Airline’s share _____ rose 103 percent, compared with Bryant Airlines’ shares, which rose only 62 percent.

141. (a) wing(b) price (c) engine(d) ticket

In the fourth quarter alone, Feather Airline stock more than _____ to hit $18 per share. 142. (a) rose

(b) spent(c) lifted(d) doubled

Its closing price last Friday was at $26.68 per share, an _____ high for the airline.143. (a) all-time

(b) all the time(c) every time(d) each time

Feather Airline’s current market _____ is $3.94 billion.144. (a) bank

(b) aisle(c) store(d) value

Questions 145-148 refer to the following letter.

CEO Values Opinions

Lawrence Smith, Chief Executive at Dynamic Technologies, encourages teamwork over competition among managers and business _____ in his company.

145. (a) units(b) drawers(c) offices(d) calendars

When big _____ must be made, she always asks managers to give him two different options, with all the positive and146. (a) times

(b) ideas(c) folders(d) decision

negative _____ associated with each option..147. (a) issues

(b) orders(c) tables(d) workers

Page 27: 101s1.downloadmienphi.net/file/downloadfile2/180/1403331.doc · Web viewDuring normal office hours, University Maintenance will respond directly to a power failure. After hours, the

Smith is so dedicated to building teamwork that 30 percent of managers’ end-of-the-year bonuses are decided by the amount of _____ they put into teamwork.

148. (a) pad(b) word(c) group (d) effort

Questions 149-152 refer to the following letter."35th Annual Bridge Race to Be HeldThe 35th Annual Bridge Race will be held in Potterstown, OH next Saturday.

The five kilometer race goes across the bridge and back _____." 149. (a) two

(b) pair(c) twice(d) double

The first Bridge Race was held in 1971, to inaugurate the _____ Potterstown Bridge, which was built to replace an aging

150. (a) now(b) common(c) current(d) outdated

and dilapidated _____ that spanned the Fauntleroy River. 151. (a) race

(b) event(c) river(d) bridge

"The Bridge Race has become a major event in Potterstown; some 15,000 people run the race each _____.152. (a) year

(b) prize(c) runner(d) course

For more information about the Potterstown Bridge Race, visit http://www.bridgerace.org."

Welcome to QuickRenew Online License Renewal!Thank you for using New York’s new QuickRenew online system to apply for your driver’s license. Please note that our policy has changed as of June 2005. It is now our policy not to renew driver’s licenses that have outstanding tickets or violations of any kind.You may use this system to renew your license only if you meet ALL the following criteria:

* You currently have a driver’s license and obtained it within the past three years* Your license was lost or destroyed AND your name, date of birth, and gender were correct on your lost or destroyed license* Your address has not changed* Your license is not suspended, has not expired, and is not invalid for any reason

Page 28: 101s1.downloadmienphi.net/file/downloadfile2/180/1403331.doc · Web viewDuring normal office hours, University Maintenance will respond directly to a power failure. After hours, the

If you do not meet ALL the criteria above, you must visit one of our offices in person to renew or replace your driver’s license.

153. When did the company change its policies?(a) May 2005(b) June 2005(c) September 2005(d) November 2005

154. According to the passage, which of the following is NOT a condition required to use the online system?(a) No tickets are outstanding.(b) No change of address is needed.(c) The current license was issued in another state.(d) The current license was issued within the past 3 years.

155. What did Martha Carlson acknowledge about the office furniture?(a) That it was paid for in cash(b) That it was delivered on time(c) That it was delivered undamaged(d) That it was paid for by credit card

156. Which of the following was NOT part of Martha Carlson’s order? (a) A desk(b) A lamp(c) A chair(d) A file cabinet.

Plaster Mold Casting

Plaster mold casting is used to make limited production parts.

The part to be cast is placed into plaster, which hardens around the part forming a mold. The mold is carefully cut in half and the part inside removed. The two halves of the plaster mold are then reassembled to create a whole piece. Molten metal is poured into the mold. After the metal cools down, the plaster mold is broken and the metal piece is removed, cleaned, and polished.

Cast parts using this technique range in weight from 30g to 7kg. Section thicknesses can be as small as 0.6mm. Copper and magnesium are commonly cast using this process.

Page 29: 101s1.downloadmienphi.net/file/downloadfile2/180/1403331.doc · Web viewDuring normal office hours, University Maintenance will respond directly to a power failure. After hours, the

157. According to the passage, what kind of parts is the plaster mold casting process used to make?(a) Thin parts(b) Heavy parts(c) Precision parts(d) Limited production parts158. What is the lower end of the weight range for parts made using the plaster mold casting technique? (a) 7g(b) 16g

(c) 20g(d) 30g.

159. Which of the following metals is mentioned as being used in plaster mold casting? (a) Zinc(b) Steel(c) Copper(d) Aluminum

World of Travel Magazine1205 Madison AvenueNew York, NY

Dear Lisa Heller,

Thank you for your comments on Margaret Dawson’s latest article, “A Walk Through the Eastern Gardens.” A copy of your letter has been forwarded to the author for her response. I am sure you will be hearing from her in the near future.

I am pleased that you found our article informative and interesting, and hope that you will continue to read our publication. Should you have any comments or questions in the future, please do not hesitate to write to this office.

Adventure Safely!Mark GreenbaumEditor in Chief,World of Travel Magazine.

160. In what kind of magazine did Margaret Dawson’s article appear?(a) An art magazine(b) A food magazine(c) A travel magazine(d) A gardening magazine.

161. What does Mark Greenbaum say happened to the letter Lisa Heller sent?(a) It was sent to an author.

(b) It was entered into a contest.(c) It was published in the magazine.(d) It was filed for future reference.

162. What does Mark Greenbaum invite Lisa Heller to do? (a) Apply for a fulltime job(b) Submit a magazine article(c) Visit when she is in the area(d) Write again if she has any questions.

Investment Pays Off

Last January, Samantha Richards Couture, an $800 million dollar woman’s fashion designer and merchandiser, spent $400 million on a buyout of Sager Brand Stores, a chain that specializes in selling low-priced clothing in small towns throughout rural North America. The investment took less than a year to pay off. Yesterday, Samantha Richards announced it had nearly doubled its third-quarter profits, over the previous year. The stock market responded by pushing Samantha Richards share prices to their highest in the company’s history.

Page 30: 101s1.downloadmienphi.net/file/downloadfile2/180/1403331.doc · Web viewDuring normal office hours, University Maintenance will respond directly to a power failure. After hours, the

163. What is learned about Samantha Richards Couture?(a) It imports textiles from Asia.(b) It is worth more than $1 billion.(c) It is seeking to enter new markets.(d) It designs and sells woman’s clothing.

164. What did Samantha Richards Couture do the previous year?(a) Purchased another company

(b) Emerged from financial troubles(c) Released a successful product line(d) Sold its most profitable business unit

165. What is learned about Sager Brand Stores?(a) It operates internationally.(b) It owes nearly $800 million.(c) It has more than 6,000 outlets.(d) It sells mainly in small towns.

The Eiffel Tower

The Eiffel Tower is one of the most recognizable landmarks in the world. Made entirely of iron latticework, it stands 986 feet tall, not including the 79-foot antenna on top, and weighs in at an impressive 10,100 tons.

For 40 years the Tower held the honor of being the world’s tallest building. These days it is far from the tallest, but it is still visited by more than 6 million people each year.

The Tower is open to visitors Tuesday through Sunday from 11 a.m. to 9 p.m

166. Which of the following is NOT described about the Eiffel Tower in the brochure?(a) Its age(b) Its height(c) Its weight(d) Its building material

167. On which day is the Eiffel Tower closed to visitors? (a) Monday

(b) Tuesday(c) Sunday(d) Wednesday.

168. For how long was the Eiffel Tower the world’s tallest structure? (a) 17 years(b) 24 years(c) 30 years(d) 40 years

Page 31: 101s1.downloadmienphi.net/file/downloadfile2/180/1403331.doc · Web viewDuring normal office hours, University Maintenance will respond directly to a power failure. After hours, the
Page 32: 101s1.downloadmienphi.net/file/downloadfile2/180/1403331.doc · Web viewDuring normal office hours, University Maintenance will respond directly to a power failure. After hours, the

169. From the information in the graph, what can be said about the year 2001?(a) Support sales rose to their highest level.(b) Hardware sales and support sales were the same.(c) Support sales were much higher than hardware sales.(d) Hardware sales and support sales both increased from the previous year

170. What general trend is shown by the graph?(a) Support sales rose every year.(b) Hardware always exceeded support sales.(c) Support always exceeded hardware sales.

(d) Hardware sales increased while support sales remained unchanged.171. In which year were hardware sales the highest? (a) 1999(b) 2000(c) 2001(d) 2003.

172. How much support was sold in 2003?(a) $5.0 million(b) $12.5 million(c) $27.5 million(d) $30.0 million.

Page 33: 101s1.downloadmienphi.net/file/downloadfile2/180/1403331.doc · Web viewDuring normal office hours, University Maintenance will respond directly to a power failure. After hours, the

Alice Kwak2551 Lancey StreetToronto, Ontario M2O 2R5P. (566) 734-4470E-mail: [email protected]

Ms. Rory SaundersHuman Resources ManagerTrinity Client Publications881 Second AvenueToronto, Ontario M20 3K2

Dear Ms. Saunders:

I am writing in regard to the Administrative Assistant position that is available at Trinity Client Publications.

I have just completed the Office Administration program at Frayer College and am excited to try my skills in the real world. I am literate in basic computer programs, and have writing, editing, and critical thinking skills. I work well with tight deadlines, and am a highly-motivated self-starter.

At past jobs I have edited letters, memos, and proposals, and corresponded with clients and donors. I am efficient and accurate in all my work. Please consult the enclosed resume for additional information about my work experience.

Thank you for taking the time to consider my application. If you have any questions you can reach me at (566) 734-4470 or at [email protected].

Sincerely,Alice KwakEncl.

Page 34: 101s1.downloadmienphi.net/file/downloadfile2/180/1403331.doc · Web viewDuring normal office hours, University Maintenance will respond directly to a power failure. After hours, the

173. In which of the following areas does Alice Kwak claim to have experience?(a) Using databases(b) Editing letters(c) Arranging travel(d) Organizing files.174. Which of the following has been included with Alice Kwak’s letter? (a) A resume(b) A photograph(c) A reference list(d) A sample of her work

175. Who is Rory Saunders?(a) A copy editor(b) A Career Center employee(c) A human resources manager(d) A teacher at Frayer College.

176. What does Alice Kwak describe herself as being? (a) Organized(b) A team player(c) A self-starter(d) Good with people

Page 35: 101s1.downloadmienphi.net/file/downloadfile2/180/1403331.doc · Web viewDuring normal office hours, University Maintenance will respond directly to a power failure. After hours, the

177. In which year was whole milk consumed the most?(a) 1980(b) 1985(c) 1990(d) 1995.

178. What trend does the graph show? (a) Consumption of whole milk is declining.(b) Consumption of lowfat milk is increasing.(c) Consumption of all milk types is decreasing.(d) Consumption of skim milk is increasing faster than other types.

179. What was the per capita milk consumption for skim milk in 1995?(a) 30 gallons(b) 50 gallons(c) 70 gallons(d) 90 gallons.

180. What is true about the graph?(a) It measures milk in liters.(b) It spans a four year period.(c) It compares three kinds of milk.(d) It shows household consumption of milk.

To: Dianne ThomasFrom: Jason Levy MDate: Tuesday May 25, 2005 8:37:12 a.m.Subject: A favor to ask---------------------------------------------------------------------Hello Dianne,

I need you to do a big favor for me. There’s a package sitting on my desk that needs to be sent to the London office. They had asked me to edit our annual report before it goes to press and they need it back this week. The manuscript is packaged and addressed; it needs to be sent with some kind of express mail service.

Please go online and look at the In-A-Flash Delivery Service site (www.inaflashdelivery.com).

Enter the destination (London office), how much it weighs (1 pound), when you need it to arrive (overnight). You can

Page 36: 101s1.downloadmienphi.net/file/downloadfile2/180/1403331.doc · Web viewDuring normal office hours, University Maintenance will respond directly to a power failure. After hours, the

pay for it online and they’ll automatically send someone over to the office to pick the package up. Make sure you print a receipt when you pay for it, and make sure you get a receipt when the package gets picked up. The package weighs about a pound. It needs to arrive in London by Friday.

I have meetings all day and won’t be in the office until around 4p.m. Can you please see that the package goes out this morning? Once it’s been sent, can you send Peggy an e-mail and just let her know the package is out the door and that she can expect it before Friday?

Thanks,

Jason==========================================================================

181. Where does Jason say the package can be found?(a) On his desk(b) At reception(c) On the coffee table(d) By the photocopy machine.

182. What does Jason ask Dianne to do?(a) Go to the post office to send the package.(b) Leave the confirmation number on his desk.(c) Purchase additional insurance for the package.(d) Print a receipt when she pays for the shipping.

183. When does the package need to get to London? (a) Tuesday(b) Wednesday

(c) Thursday(d) Friday

184. How much does it cost to send a package to Europe in three days? (a) $18.45(b) $26.5(c) $38.95(d) $44.15

185. For which kind of package does the table give prices? (a) Gifts, over five pounds(b) Gifts, up to three pounds(c) Documents, up to one pound(d) Documents, two to four pounds.

M E M O R A N D U MLiberty Legal Services 302 Pine Street, Suite 2300Los Angeles, CA 80214

Page 37: 101s1.downloadmienphi.net/file/downloadfile2/180/1403331.doc · Web viewDuring normal office hours, University Maintenance will respond directly to a power failure. After hours, the

Dear Colleagues,

I am writing to introduce the newest member on our staff. Andrea Langley comes to us straight out of Hanfard Law School, where she graduated at the top of her class. As is typical of our new lawyers, she will spend her first six months here working with various people in the office.

This month Andrea will be working with Karen Jackson in our real estate division. Next month she will work with Larry Dean in licensing. That is as far as we have her schedule planned. If any of you have some time to spend with Andrea and have some interesting projects coming up, please let me know and we may be able to arrange for her to work with you. Please keep in mind that we do not want an intelligent and eager young lawyer like Andrea doing busy work. Her mind is much more useful to us than that.

Please take a moment to stop by Andrea’s desk and introduce yourselves. And congratulate her for passing her exam.

Thanks,

ScottScott HartmanSenior Partner.------------------------------------------------------------------------------------------------------------------------------To: Scott HartmanFrom: Nancy StrongDate: July 1, 2005 10:45:19 a.m.Subject: Andrea Langley

Scott,

I got your memo about Andrea Langley. I’m starting to work on an interesting and difficult case that I think would be a good learning experience for her; I could use the help while she’s making her rounds.

The case I want her for is a mergers and acquisitions case. It’s for a merger between two high-powered technology companies. Due to their backgrounds, it will require a fair amount of paperwork. There will also be a lot of research required, and I think it would give her a real taste of what most work in mergers and acquisitions is like.

If it’s okay with you, would you please run it by Andrea? We’d really love her down in this office for a few weeks.

Thanks,

Nancy Strong.

186. What is learned about Andrea Langley?(a) She is Nancy Strong’s niece.(b) She recently graduated from school.(c) She worked as a legal secretary for several years.

(d) She signed a one-year contract to work with Liberty.

187. What will Andrea Langley do during her first six months at Liberty?

Page 38: 101s1.downloadmienphi.net/file/downloadfile2/180/1403331.doc · Web viewDuring normal office hours, University Maintenance will respond directly to a power failure. After hours, the

a) Work with several lawyers(b) Concentrate on a single case(c) Learn about California’s laws(d) Specialize in one area of law.

188. What does Scott Hartman NOT ask people to do?(a) Introduce themselves to Andrea Langley(b) Andrea Langley to observe them in court(c) Suggest projects for Andrea Langley to work on(d) Congratulate Andrea Langley for her accomplishment.

189. In which area of law does Nancy Strong work?(a) Adoption

(b) Licensing(c) Real estate(d) Corporate mergers.

190. What does Nancy Strong say about the case she is working on?(a) It is likely to be resolved out of court.(b) It will involve a great deal of research.(c) It is receiving publicity in the newspapers.(d) It requires a person with strong computer skills.

Tuesday September 5, 2005

8:00 ------------------------------------------------------------------------------9:00 Meeting, Kevin Larson ---discuss budget for next year10:00 -----------------------------------------------------------------------------11:00 -----------------------------------------------------------------------------12:00 Meeting, Kalfax Printers 233 E. 14th Street1:00 -------------------------------------------------------------------------------2:00 -------------------------------------------------------------------------------3:00 Hair Trends, cash only4:00 Jessica Masson, office ---annual review5:00 -------------------------------------------------------------------------------------------------------------------------------------------------------------------------------------------------------------To: Cathy ReidFrom: Amy SchneiderDate: September 5, 2005Subject: Lunch-------------------------------------Dear Cathy,

I am SO sorry, but I have to cancel lunch today. You know I’ve been complaining about the printing company we use at work for several months now. We’re finally trying to get a new printing company, and I have a meeting with them today at noon. We’re hoping to have them start by the end of the month. We have a few big jobs coming up and we’d like to try our new printer out on some smaller jobs first. So this meeting today is very important and will probably take 2 hours. I tried for something later, but noon was the only time I could coordinate with them.

I’m going to get my hair cut at 3:00. If you’d like to meet me at Hair Trends on 4th Ave., we can catch up for a little while, but I need to be back in the office by 4:00.

No need to call, if you decide to meet me at the hair salon, just show up. Otherwise I’ll call you later to reschedule lunch for next week. My treat.

Sorry!!

Amy.

Page 39: 101s1.downloadmienphi.net/file/downloadfile2/180/1403331.doc · Web viewDuring normal office hours, University Maintenance will respond directly to a power failure. After hours, the

191. At what time is Amy Schneider planning to work on a budget for the following year?(a) 9:00(b) 11:00(c) 12:00(d) 3:00.

192. What does Amy Schneider have scheduled for 4:00 p.m.?(a) A haircut(b) An annual review(c) An job interview(d) A meeting with a client.

193. Why does Amy Schneider apologize to her friend? (a) She has to cancel their meeting.(b) She did not invite her to a party.(c) She forgot about their appointment.

(d) She did not send her a birthday card.

194. What does Amy Schneider suggest that her friend do?(a) Call her later(b) Stop by after work(c) Meet her at a hair salon(d) Come by her office later.

195. How long does Amy Schneider expect her meeting with the printer to take?(a) One hour(b) One and a half hours(c) Two hours(d) Two and a half hours.

Roger Lee, Ph.D.Head, Search CommitteeDepartment of FinanceTealand School of BusinessLindberg UniversityLacy, NM 82144

January 23, 2006

Dear Dr. Lee,

I recently learned that you have an opening for a professor in your department. I am quite interested in moving to Lacy, NM. My background is in accounting and I am currently a tenured professor at the Harper School of Business.

I have excellent publishing and teaching records, and have served on several administrative committees. I have attached my CV for you to look at. If you need more information, or if it looks like I may be a good fit for your department, please contact me.

Thank you for your consideration,

William ChangDepartment of FinanceHarper School of BusinessSchool of BusinessRidgeland UniversityRidgeland, TN 22106(403) 538-3194e-mail: william.chang@ridgeland.edu------------------------------------------------------------------------------------------------------------------------------

Page 40: 101s1.downloadmienphi.net/file/downloadfile2/180/1403331.doc · Web viewDuring normal office hours, University Maintenance will respond directly to a power failure. After hours, the

William Chang102 Percy LaneRidgeland, TN 22114(403) 223-1016

Department of FinanceHarper School of BusinessSchool of BusinessRidgeland UniversityRidgeland, TN 22106(403) 538-3194e-mail: [email protected]

EDUCATION

1994-1997 The Haverford School of BusinessPh.D. Accounting

1985-1987 Institute of Management, Glasgow, ScotlandMaster of Business Administration

EXPERIENCE

2003-present Harper School of Business, Ridgeland UniversityAssociate Professor of Accounting

2001-2003 Assistant Professor of Accounting

1997-2001 Keller School of Business, Garfield UniversityAssistant Professor of Marketing

1987-1994 Hapland Worldwide(Hapland Worldwide is an advertising agency.)Position Held: Senior Accountant

Awards

2004 2004 Teaching AwardAward given to outstanding teachers of the core accounting classes

2002 Trent Bayland AwardAward given in recognition of the best accounting paper derived from a Ph.D. thesis.

196. Where does William Chang currently work?(a) A database center(b) A business school(c) A research center(d) An accounting firm.

197. When did William Chang work at Hapland Worldwide?(a) 1985-1987(b) 1987-1994

Page 41: 101s1.downloadmienphi.net/file/downloadfile2/180/1403331.doc · Web viewDuring normal office hours, University Maintenance will respond directly to a power failure. After hours, the

(c) 1997-2001(d) 2001-2003.

198. In which area did William Chang receive his Ph.D.?(a) Accounting(b) Administration(c) Project manager(d) Account management.

199. At which school did William Chang get his Ph.D.?

(a) Harper School of Business(b) Keller School of Business(c) Tealand School of Business(d) Haverford School of Business.

200. What did William Chang do in 2004?(a) He was promoted.(b) He won an award.(c) He published a book.(d) He started a company.

ANSWER101. B 102. A 103. D 104. B 105. D 106. C 107. C 108. B 109. C 110. B111. C 112. A 113. A 114. D 115. D 116. D 117. B 118. A 119. D 120. B121. C 122. B 123. B 124. A 125. D 126. B 127. B 128. C 129. B 130. C131. D 132. C 133. D 134. A 135. A 136. C 137. D 138. B 139. A 140. D141. B 142. D 143. A 144. D 145. A 146. D 147. A 148. D 149. C 150. C151. D 152. A 153. B 154. C 155. C 156. B 157. D 158. D 159. C 160. C161. A 162. D 163. D 164. A 165. D 166. A 167. A 168. D 169. B 170. A171. A 172. C 173. B 174. A 175. C 176. C 177. A 178. A 179. A 180. C181. A 182. D 183. D 184. C 185. C 186. B 187. A 188. B 189. D 190. B191. A 192. B 193. A 194. C 195. C 196. B 197. B 198. A 199. D 200. B

Page 42: 101s1.downloadmienphi.net/file/downloadfile2/180/1403331.doc · Web viewDuring normal office hours, University Maintenance will respond directly to a power failure. After hours, the

101. American businesses rely on just-in-time delivery of inventory and components to improve _____, reduce lead times, and trim costs. (a) efficient(b) efficiency(c) efficiently(d) be efficient

102. The course will be broken into six modules, each of which stands alone as an instructional _____. (a) quiz(b) unit(c) school(d) teacher

103. Philanthropists Henry and Elsie Wellman today will announce a $20 million _____ to Scripp’s Cancer Institute, to provide seed money for promising new cancer research projects. (a) donate(b) donated(c) donation(d) donating

104. Attoyot Industries is _____ for creating more than 190,000 jobs in the past two years, making it the state’s

largest nongovernment employer. (a) hiring

(b) planning(c) dependable

(d) responsible

105. Proper pricing, accurate quotations, choosing the terms of the sale, and selecting the payment method are

critical _____ in selling a product or service overseas. (a) costs

(b) credits(c) elements

(d) problems

106. Paul Hamilton _____ after 11 seasons as head coach of the Elmhurst football team to focus on his

responsibilities as the school’s athletic director. (a) worked out(b) named after(c) turned over(d) stepped down

107. Citizens Electronics Corp. needs to downsize, to bring it more in line with its shrinking market _____. (a) link(b) plug(c) share(d) outlet

108. The report concluded by recommending that a business _____ be hired to help things run more smoothly. (a) manager(b) meeting(c) strategy(d) schedule

109. It _____ four craftsmen about 30 hours to build, pholster, and finish an authentic Brunton black and cherry dining room chair. (a) takes(b) times(c) keeps (d) holds110. Most scientists doubt Dr. Richards’ claim to _____ signs of microscopic life on Mars. (a) find(b) finding(c) are found(d) having found

111. The Hostetler-Graz _____ of Muchen Automotive released the first official photos of its new N-class sedan.(a) engine(b) feature(c) vehicle(d) division

112. An earthquake struck about 120 miles off Northern California early Sunday, near _____ a larger earthquake occurred last week. (a) how(b) that(c) when(d) where

113. After loading the worksheet, review it to make sure the correct _____ has been loaded into the input cells. (a) data(b) addition(c) computer(d) software

114. All hotel suites _____ high-speed Internet access, complimentary breakfast, beverages, and free laundry and valet services.

Page 43: 101s1.downloadmienphi.net/file/downloadfile2/180/1403331.doc · Web viewDuring normal office hours, University Maintenance will respond directly to a power failure. After hours, the

(a) charge(b) travel(c) include(d) reserve115. High school students should be offered extra _____ in courses that are required to prepare them for college. (a) assistance(b) graduation(c) scholarship(d) registration

116. Communicating with color is a proven way to improve the response you get from print materials _____ direct-mail and meeting handouts. (a) as to(b) as for(c) such as(d) to such

117. Hydrochloric acid is always _____ in the stomach and is a crucial element for proper digestion. (a) basic(b) present(c) various(d) allowed

118. If your apartment needs _____, contact the office during normal business hours to arrange for a service call. (a) rent(b) key

(c) repairs(d) roommates

119. MGK Research provides research and advice for

executives, helping them to improve performance and _____ costs with the aid of technology.

(a) reduce(b) reduces(c) reducing(d) reduction

120. Margaret Hanzlik received her B.A. degree in English Literature _____ the University of Columbus in 2001. (a) by(b) on(c) from(d) while

121. In 1962, there were an estimated 10,000 computers in the world, and although they were _____ by today’s standards, each cost hundreds of thousands of dollars. (a) priced(b) primitive(c) expensive(d) programmed

122. A two-tablespoon _____ of peanut butter contains 12.2 grams of unsaturated fat and no cholesterol. (a) piece(b) glass(c) serving(d) measuring

123. Tamara Johnson’s hand-designed bowls, cups, and plates have become a commercial _____ locally, and she now plans to sell her wares on the Internet. (a) craft(b) store(c) success(d) television

124. In certain industries, rapidly fluctuating _____ can create the twin problems of excess inventory and inventory shortages. (a) charge(b) demand(c) request(d) estimate125. From the start, Wilbur’s poetry was characterized by a formal and refined _____ that he helped make popular. (a) book(b) word(c) style(d) letter

126. Once your registration has been approved, your membership will remain effective until the _____ January. (a) end(b) after(c) following(d) beginning

127. The company hired a consulting _____ to help with introducing the new product to the Boston area. (a) firm(b) guest

Page 44: 101s1.downloadmienphi.net/file/downloadfile2/180/1403331.doc · Web viewDuring normal office hours, University Maintenance will respond directly to a power failure. After hours, the

(c) program(d) traveler

128. All first-year master’s degree students at the Daniels Institute are required to take an introductory oceanography course, _____ of their previous aca-demic background. (a) anyway(b) despite(c) regardless(d) nevertheless

129. The survey concluded that the two-day convention provided the city with a total estimated economic _____ of $8.5 million.(a) product(b) benefit(c) weekend(d) condition

130. Our products are warranted to be free of _____ in materials and workmanship. (a) goods(b) metals(c) designs(d) defects

131. Many theorists contend that constructivism represents a viable model for explaining how mathematics _____. (a) learn(b) is learned(c) are learned(d) is to learn

132. According to studio estimates issued on Sunday, the new adventure film has earned $71.1 million across the United States and Canada _____ opening on Wednesday.(a) once(b) since(c) during(d) because

133. When Diablo, Inc. took over its former partner last month, it demonstrated an _____ of the growing importance of retail applications. (a) understand

(b) understood(c) understands

(d) understanding

134. Southern Star Energy currently _____ more than 130 oil wells in Texas and Oklahoma.

(a) operates(b) operating(c) operations(d) has operated

135. The industrial revolution was driven by profound social _____, as Europe moved from a primarily agricultural and rural economy to a capitalist and urban economy. (a) ages(b) roots(c) changes (d) periods136. Most airlines offer discounts for travelers making _____ through their Web- sites. (a) travel(b) luggage(c) airports(d) reservations

137. If the computer is not installed and used in accordance with the manufacturer’s instructions, it may cause _____ with radio and television reception. (a) appliance(b) equipment(c) reception(d) interference

138. Heat-related problems are more likely to occur among workers who have not been given time _____ to working in the heat. (a) adjusted(b) to adjust(c) adjustment(d) are adjusting

139. Economic growth in many countries will be _____ until Chinese demand for Asian goods picks up. (a) sent off(b) held back(c) taken down(d) checked out

140. We are located _____ 1075 Bellevue Way, just north of Bellevue Square Mall in Belgate Plaza. (a) at(b) of(c) to(d) with

Page 45: 101s1.downloadmienphi.net/file/downloadfile2/180/1403331.doc · Web viewDuring normal office hours, University Maintenance will respond directly to a power failure. After hours, the
Page 46: 101s1.downloadmienphi.net/file/downloadfile2/180/1403331.doc · Web viewDuring normal office hours, University Maintenance will respond directly to a power failure. After hours, the

Questions 141-144 refer to the following letter.

It is never too _____ to start thinking about your child’s or your grandchild’s education. 141. (a) early

(b) short(c) quick(d) young

At Resident Investment Services, our Financial Consultants are ready to help you get started as _____ as you want. 142. (a) ever

(b) soon(c) always(d) usually

Whether it’s a 529 College Savings Plan, an UGMA/UTMA Custodial Account, or simply a Mutual Fund, we have the _____ to help you find the option that’s right for you.

143. (a) profit(b) savings(c) beginning(d) expertise

So stop by any one of our branches or give us a call. Our Financial Consultants are ready _____ you through all your college planning needs, one step at a time.

144. (a) to walk(b) walking(c) for walking(d) to have walked

Questions 145-148 refer to the following letter.

Oil prices may fall next year as output rises, analysts said in a recent _____.145. (a) gain

(b) total(c) yield(d) survey

The increase in prices has encouraged producers to _____ in new equipment and develop new fields.146. (a) use

(b) labor(c) invest(d) pursue

Analysts expect that crude oil traded in London will average $52.30 _____ barrel in the third quarter.147. (a) for

(b) per(c) via(d) over

Respondents expect crude to average $50.50 this year and $46.50 next year. London crude futures touched $60.26 a barrel today, the highest since the contract was _____ in 1988.

Page 47: 101s1.downloadmienphi.net/file/downloadfile2/180/1403331.doc · Web viewDuring normal office hours, University Maintenance will respond directly to a power failure. After hours, the

148. (a) drilled(b) entered(c) consumed(d) introduced

Questions 149-152 refer to the following letter.Water and electricity are a dangerous _____,

149. (a) mix (b) set (c) sum(d) tie

which is why there are so many regulations governing such matters as the use of electrical _____, voltage limits, and types of electrical outlet permitted in wet areas.

150. (a) methods(b) contracts (c) economies(d) appliances

However, regulations vary greatly between countries, from the types of switches and outlets allowed _____ the room, to the _____ needed before someone can tackle electrical work in bathrooms.

151. (a) in(b) into(c) onto (d) after

However, regulations vary greatly between countries, from the types of switches and outlets allowed _____ the room, to the _____ needed before someone can tackle electrical work in bathrooms.

152. (a) exchanges(b) governments(c) qualifications (d) administrations

Office Supplies, Products & Office Furniture From Anytime Office Supplies!

Anytime Office Supplies offers RUSH DELIVERY in 1-2 Days for all office supplies, office products,business equipment & furniture.

Anytime Office Supplies is your single source for over 35,000 products, including office furniture, meeting supplies, and much more. Our online office supply ordering system is extremely easy to use and our customer service staff will do anything to make sure we meet your needs. Our 35 wholesale warehouses & mega depots provide efficient sources for delivery throughout the United States.

153. What special service does Anytime Office Supplies offer?(a) Free shipping(b) Quick delivery(c) Discounted prices(d) Free installation

154. According to the advertisement, where can Anytime Office Supplies’ products be purchased?(a) Online(b) Through a catalog(c) At its own retail outlets(d) From major furniture stores.

Page 48: 101s1.downloadmienphi.net/file/downloadfile2/180/1403331.doc · Web viewDuring normal office hours, University Maintenance will respond directly to a power failure. After hours, the

Stopped Cold Above Mars

The recent solar activity has claimed a far-off victim: a radiation meter aboard the Mars Argo spacecraft.

The instrument stopped working in late October, government officials said last week, after being bombarded by radiation from a particularly intense solar storm.

The meter, which had worked well in the two years that Argo has been orbiting Mars, was used in part to determine the potential radiation risk to future human explorers. Mission engineers have tried to get the meter working again but have not succeeded. They say they will keep trying in the coming weeks.

155. What does the report say was damaged on the Mars Argo spacecraft?(a) A meter(b) A telescope(c) A solar panel(d) A solar cell

156. Which of the following is true about the Mars Argo spacecraft? (a) It had problems two weeks earlier.(b) It was built by American engineers.(c) It has been orbiting Mars for two years.(d) It is scheduled to land on Mars’ surface.

To: All StaffFrom: Kathy WhiteDate: July 25, 20—Re: Back-up for Power Outages/Major Systems Failure

In the wake of last week’s major blackout, many people have been asking about our campus’s electrical backup systems.

In the event of disruption of normal AC power at any node, Telephone Services maintains back-up batteries that can power the system. We also maintain a set of back-up telephone lines and a complement of spare equipment. The Veterinary School is equipped with back-up generators. If the power goes out, the generators will cycle on and restore power to the system.

During normal office hours, University Maintenance will respond directly to a power failure. Afterhours, the school will contact the local electrical company.

157. According to the memorandum, when did the most recent power failure occur?(a) One day earlier(b) One week earlier(c) Two weeks earlier(d) One month earlier

158. Which part of the campus has back-up generators? (a) The Cafeteria(b) The Plant Nursery

(c) The Medical School(d) The Veterinary School.

159. Who will handle power failures at the university after normal working hours? (a) The Police(b) The Fire Department(c) The local electrical company(d) The University Maintenance Department

Page 49: 101s1.downloadmienphi.net/file/downloadfile2/180/1403331.doc · Web viewDuring normal office hours, University Maintenance will respond directly to a power failure. After hours, the

Pollock’s buys condiment brands from French firm

H.J. Pollock’s Co., the United States’ largest ketchup maker, has agreed to buy the U&P Foods and Perkins sauce divisions from France’s Groupe Dumar for $852 million.

The brands, which are marketed primarily in Britain, the United States, and Canada, accounted for close to $292 million in revenue last year, Dumar said.

Dumar said it will also grant Cleveland, Ohio-based Pollock’s a license to manufacture its O’Shea brand of sauces in Europe as part of the deal.

The sale adds condiments including Perkins Worcestershire sauce, U&P Sauce, O’Shea soy sauce and Ranee spices to Pollock’s famous original ketchup. Besides controlling 60 percent of the U.S. ketchup market, Pollock’s makes brands including Rui frozen potatoes and Slimdown diet foods.

160. What did Pollock’s Co. purchase?(a) Another company(b) New product lines(c) New production facilities(d) License to use a new technology.

161. What is learned about H.J. Pollock’s Co.?(a) It has facilities throughout Europe.(b) It is the largest ketchup maker in the U.S.

(c) It is the leading food distributor in the U.S.(d) It earned $292 million in revenues the previous year.

162. Which of the following products is Pollock’s NOT planning to sell? (a) Soy sauce(b) Diet foods(c) Mayonnaise(d) Frozen potatoes.

Mouth Guard for Migraine Relief

Dr. James Beard, a dentist and migraine sufferer, has come up with a mouth guard designed to alleviate painful headaches. James calls the device the NTI (for nociceptive trigeminal inhibition) System. It is small, covering just the two front teeth, and designed to keep the jaw from clenching while the wearer sleeps. In clinical trials, more than 80 percent of patients felt some relief after using the NTI for two months. It costs around $500 and is available on rescription from a dentist.

163. What is the NTI designed to do?(a) Cure colds(b) Stop snoring(c) Improve sleep(d) Ease headaches

164. How is the NTI designed to be worn?(a) On the head(b) In the mouth

(c) Over the eyes(d) Around the neck

165. Which claim is made about the NTI?(a) It was tested on people.(b) It was developed in Europe.(c) It was approved by the government.(d) It was designed by a team of scientists.

Marketing Communications5801 South Ellis Ave., Suite 2010Chicago, IL 60637

Page 50: 101s1.downloadmienphi.net/file/downloadfile2/180/1403331.doc · Web viewDuring normal office hours, University Maintenance will respond directly to a power failure. After hours, the

September 2, 20—

Mr. Leo Payne, Chief Executive OfficerMain Deal Realty6525 N. Sheridan Rd.Chicago, IL 60626

Dear Mr. Payne,

During the four years Marketing Communications served as Great Scott's public relations agency, that organization received 500 percent more positive press than it ever had in its history.

We would like the opportunity to make a presentation to you to serve as Main Deal's public relations agency to handle your entire marketing, public relations, and advertising programs. We feel sure we can offer you the same kind of representation that helped Great Scott become a leader in your industry. We have specialized in the real-estate industry in Chicago for more than seventeen years.

I suggest we meet at your office next Wednesday at 2:00 P.M. Our presentation will take forty minutes. During our years as Great Scott's public relations agency, we worked with Dusty Parker, Shannon Monroe, and Elisa Banks of your company. I'm sure they can give you additional information on our qualifications, and you may want them to attend our presentation.

I will call your office to confirm a meeting time. We look forward to meeting with you.

Sincerely,

Marcus Silver.

166. What does Marcus Silver want Leo Payne to do?(a) Respond by phone(b) Attend a luncheon(c) Visit Silver’s office(d) Listen to a presentation

167. Which of the following is a claim Marcus Silver makes about Marketing Communications? (a) It offers a money-back guarantee.(b) It is designed for small businesses.

(c) It specializes in the real estate industry.(d) It is the leading public relations agency in Chicago.168. What does Marcus Silver hope Dusty Parker, Shannon Monroe, and Elisa Banks will do? a) Schedule a meeting with Leo Payne(b) Make a final decision by the end of the week(c) Sign a contract with Marketing Communications(d) Provide information about Marketing Communication

Page 51: 101s1.downloadmienphi.net/file/downloadfile2/180/1403331.doc · Web viewDuring normal office hours, University Maintenance will respond directly to a power failure. After hours, the
Page 52: 101s1.downloadmienphi.net/file/downloadfile2/180/1403331.doc · Web viewDuring normal office hours, University Maintenance will respond directly to a power failure. After hours, the

169. According to the graph, what is the cost of producing 10,000 New Vision 225 Band Saws?a) $0.25 million(b) $0.50 million(c) $0.75 million(d) $1.00 million

170. How many units does Mercury need to sell before revenue becomes equal to costs?(a) 10000(b) 20000(c) 30000(d) 40000

171. What is the total revenue from selling 40,000 units?

(a) Between $0.75 and $1.00 million(b) Between $1.00 and $1.25 million(c) Between $1.25 and $1.50 million(d) Between $1.50 and $1.75 million.

172. What trend does the graph show?(a) As the number of units produced increases, costs remain fixed.(b) As the number of units produced increases, revenues increase faster than costs.(c) Increasing the number of units produced does not decrease the costs to produce them.(d) Increasing the number of units produced does not raise the revenues received from selling them.

Page 53: 101s1.downloadmienphi.net/file/downloadfile2/180/1403331.doc · Web viewDuring normal office hours, University Maintenance will respond directly to a power failure. After hours, the

Review: History of Costume

The “History of Costume”or “Zur Geschichte der Kostume”was printed from 1861 to 1880 in Munich by the publishing firm of Braun and Schneider. It was originally published as individual plates in a German magazine titled “Munchener Bilderbogen.” Later, these plates were collected and bound into book form. The total publication consisted of 125 pages, with four color illustrations per page, for a total of 500 costume designs. These plates show historical dress from antiquity to the end of the 19th century.

This book is an excellent source for students who are studying the history of fashion, and for costume designers. One must be aware though, that these illustrations have a Victorian perspective to their designs. The last 35 pages show contemporary folk dress (c.1880) from most European, Asian, and African countries. These provide a source for researching plays that take place during the Victorian period, such as “The King and I” or “The Sea Gull.” The original book was published in German, so at times, the English translation is confusing. This is especially noticeable in the contemporary folk dress plates where many of the countries mentioned now have different names or no longer exist.

If you wish to obtain additional information on the original book, Denver Publications has an excellent reproduction of the printing entitled “Historic Costume in Pictures” by Braun and Schneider.

Page 54: 101s1.downloadmienphi.net/file/downloadfile2/180/1403331.doc · Web viewDuring normal office hours, University Maintenance will respond directly to a power failure. After hours, the

173. According to the review, how was the “History of Costume” first published?(a) In book form(b) As individual plates in a German magazine(c) As part of a research paper(d) As a series of framed prints.

174. Which of the following is said to be included in the book? (a) 19th century photographs(b) 500 pictures of costumes(c) Original sewing patterns(d) Nearly 2,000 pages of text

175. What does the review state is a problem with the book?(a) It is poorly translated.(b) It is not historically accurate.(c) Its images are poorly reproduced.(d) It does not include enough countries.

176. According to the reviewer, who might find the book useful? (a) World travelers(b) College students(c) Fashion designers(d) Theater researchers

Page 55: 101s1.downloadmienphi.net/file/downloadfile2/180/1403331.doc · Web viewDuring normal office hours, University Maintenance will respond directly to a power failure. After hours, the

Gredler Energy Opens Hydrogen Energy Station In Sweden

Ostrum GE, the second largest power company in the Nordic countries, officially opened its first Hydrogen Energy Station (HES), with technology products provided by Gredler Energy Systems Corporation. The new station will supply clean hydrogen fuel to three fuel cell buses as part of the pres-tigious Green Europe Advanced Transport (GREAT) program. Stockholm is one of four GREAT cities that will be using Gredler Energy hydrogen infrastructure products.

Mr. Peter Russell, Chief Operating Officer of Gredler Energy, was in Stockholm for the station opening and commented, “The opening of this station represents the first step in the introduction of a hydrogen infrastructure in the City of Stockholm. Ostrum and the City of Stockholm have taken an essential step towards creating a pathway to environmentally sustainable urban transportation solutions and we are delighted to be part of this important movement.”

The Stockholm HES is comprised of four modules: pressurized water electrolysis-based hydrogen generation, compression, high-pressure storage and hydrogen fuel dispenser. The station is capable of producing approximately 120kg per day of high-purity, high-pressure hydrogen using Gredler Energy’s proprietary technology. Each fuel cell bus carries approximately 40kg of hydrogen at 350bar (5,000psi).

Gredler Energy Systems Corporation is the world leading developer and supplier of integrated hydrogen solutions, all using the company’s proprietary hydrogen generation water electrolysis technology along with products from corporate partners.

177. Who will be in charge of the new hydrogen energy station?(a) Ostrum GE(b) The GREAT Program(c) The City of Stockholm(d) Gredler Energy Systems.

178. How much hydrogen will the new station produce each day? (a) 40kg(b) 120kg(c) 350bar(d) 5,000psi

179. The word “sustainable”in paragraph 2, line 4 is closest in meaning toa) Livable(b) Deliverable(c) Combustible(d) Maintainable.

180. What is true about the GREAT program?(a) It operates in four countries.(b) It is operated by the Gredler Corporation.(c) It promotes the use of non-polluting fuels.(d) It studies the effects of hydrogen on the atmosphere

WELLCRAFT 950 OUTDOOR PROPANE BURNER

Equipment Instructions

Please read and understand these instructions prior to operating this unit.

Turn off the valve at the air mixer.

Connect the regulator to the cylinder and tighten.

Page 56: 101s1.downloadmienphi.net/file/downloadfile2/180/1403331.doc · Web viewDuring normal office hours, University Maintenance will respond directly to a power failure. After hours, the

Open the cylinder and check all fittings for leaks.

Adjust the regulator to desired pressure. The regulator offers low to high pressure. Turning the handle clockwise (located on the top of the regulator) will increase the pressure. The burner is also adjustable by turning the valve to increase or decrease the amount of flame.

NEVER allow a vessel to go empty, as you could cause damage due to the intense heat the burner creates. NOTE: Always turn off the service valve at the cylinder when not in use.

PLEASE NOTE

This is an outdoor cooker only. Never use this propane unit indoors and do not store the cylinder indoors. If you encounter problems with this unit or require additional information, please feel free to contact our customer service department at 952-808-3232 or write us at 2800 Southcross Drive West, Burnsville, Minnesota 55306.---------------------------------------------------------------------------------------------------------------------------James E. Bay110 N. Main Ave.Odessa, MN 56276-3014October 10, 20—

Wellcraft Tools & EquipmentCustomer Service Department2800 Southcross Drive WestBurnsville, Minnesota 55306To whom it may concern,

I recently purchased a new Wellcraft 950 Outdoor Propane Burner. I have operated it twice, carefully following your instructions, and have noticed a problem with the pressure adjustment controls. I have tried varying the pressure by adjusting both the regulator handle and the valve, as per the instructions contained in the box. The problem I have found is that the regulator handle seems to get stuck at a certain point when attempting to lower the pressure, and then suddenly shuts off the flame. This has happened several times.

The handle difficulties indicate to me that I purchased a defective unit. Since this burner is under warranty, I am returning it with the express hope that you will be able to exchange it for a functional burner. I have purchased Wellcraft products in the past, and have been satisfied with their overall quality. I trust that the 950 Outdoor Burner will meet all my needs, once I have a fully functioning model.

Thank you for your consideration.

Best regards,

James E. Bay

181. What warning does Wellcraft make about its burner?(a) The valve should be connected before use.(b) Only Wellcraft propane fuel should be used.

(c) The propane cylinder should not be stored indoors.(d) The regulator should be checked frequently for leaks.

Page 57: 101s1.downloadmienphi.net/file/downloadfile2/180/1403331.doc · Web viewDuring normal office hours, University Maintenance will respond directly to a power failure. After hours, the

182. How can the burner pressure be adjusted?(a) By opening the gauge on the cylinder(b) By tightening the valve on the air mixer(c) By using the valve or the regulator handle(d) By turning the regulator handle counterclockwise.

183. Which problem does James E. Bay report with his burner? (a) The regulator handle gets stuck.(b) The valve adjustment is missing.(c) The burner emits a strong smell of gas.(d) The burner shuts off for an unknown reason

184. What does James E. Bay want Wellcraft to do? (a) Repair his burner(b) Give him a full refund(c) Send him a new Wellcraft 950 burner(d) Exchange his burner for a different model

185. What does James E. Bay state regarding Wellcraft? (a) He has had problems contacting them by phone.(b) He has never purchased their products before.(c) He has had problems with their products in the past.(d) He has been satisfied with past purchases he has made.

Page 58: 101s1.downloadmienphi.net/file/downloadfile2/180/1403331.doc · Web viewDuring normal office hours, University Maintenance will respond directly to a power failure. After hours, the

----------------------------------------------------------------------------------------------------------------------------

How to Read Your bill

1. Account Summary/Amount Due: This explains the previous amount due, current amount now due, and payment due date, plus any outstanding balance and late payment charges since your last billing.

2. Service Address: This indicates the service location. The service address may differ from the billing address.

3. Gas Usage: This is the billing period, as well as your meter readings, at the beginning and end of the current billing period. You will also find the amount of gas (measured in CCF; 1 CCF =100 cubic feet) consumed during the billing period.

4. R01-RES Non-Heating Charges: This rate is for non-heating customers. The Customer Service Charge is a flat rate that depends on what kind of customer you are (heating vs. nonheating). The cost to deliver the gas through Patriot’s distribution system is the Delivery Charge, and the cost of each unit of natural gas is the Commodity Charge. The Fuel Charge is a variable rate that might fluctuate over the course of the year, and is multiplied by the quantity of gas consumed over the billing period.

5. Billing Period: This section compares gas consumption and average daily temperature between the current billing period (month) and the same period last year.

186. How much does Joseph Bloom need to send to Patriot Gas Company?(a) 10.79(b) 83.97(c) 87.17(d) 171.14.

187. How many cubic feet of gas does one CCF represent?(a) 1(b) 10(c) 100(d) 1000.

188. What does the Commodity Charge represent?(a) The charge for gas used in heating

(b) The price of each unit of natural gas(c) The quantity of gas consumed over the billing period(d) The cost to deliver gas through the distribution system.

189. How much is the per unit Delivery Charge?(a) 0.0193(b) 0.3831(c) 0.6171(d) 0.8487.

190. Which of the following is true regarding Joseph Bloom’s bill?(a) It must be paid within 30 days.(b) A 1% late charge has been added.(c) The account is one month overdue.(d) The amount due covers two billing periods.

Page 59: 101s1.downloadmienphi.net/file/downloadfile2/180/1403331.doc · Web viewDuring normal office hours, University Maintenance will respond directly to a power failure. After hours, the

-----------------------------------------------------------------------------------------------------------------------------Dear Dr. Murphy,

Thank you very much for agreeing to make a third meal available to my son Brian while he is enrolled in Fairview’s after-school band program. I do have one concern, though, about the food being offered after school. The choices do not seem to have any relation to recognized medical studies about such things as excessive fat, sugar, and useless junk food.

Brian informs me that in recent days his after-school menu has included fried fish with fried potato nuggets, a hot dog with potato chips, and fried chicken with French fried potatoes. The most healthful piece of food offered to Brian in the past week seemed to be a small piece of green pepper floating on a sea of cheese on a pizza.

I do not feed my son this sort of food at home, and I don’t think it’s appropriate for a school to serve. Your regular lunch fare usually-includes a salad and a cooked vegetable. Is there any reason why these couldn’t be offered as part of the after-school meal, as well? I would very much like to talk to the school dietician or whoever has designed the after-school menu about the school’s third meal food offerings.

Thank you for your help.

Sincerely,

Karen Diamond

Page 60: 101s1.downloadmienphi.net/file/downloadfile2/180/1403331.doc · Web viewDuring normal office hours, University Maintenance will respond directly to a power failure. After hours, the

191. On how many days of the week does Karen Diamond’s son receive after-school meals??(a) 2 days(b) 3 days(c) 4 days(d) 5 days.

192. What is learned about Fairview’s after - school meal program?(a) Meals must be served after 4:00 p.m.(b) Some food is provided by local restaurants.(c) Eligibility is determined by a national program.(d) The School Board does not provide additional staff resources for it.

193. What complaint about the third meal program does Karen Diamond make? (a) The food is not healthy.(b) The portions are too small.

(c) No vegetarian options are offered.(d) Fruit is never included with a meal.

194. Which of the following meals was NOT served to Karen Diamond’s son?(a) Hot dog with potato chips(b) Cheese pizza with a garden salad(c) Fried fish with fried potato nuggets(d) Fried chicken with French fried potatoes.

195. What does Karen Diamond ask to do?(a) Receive a refund from the school(b) File a formal letter of complaint(c) Speak with the school’s dietician(d) Change to a different meal program.

“From : [email protected] : Thu Jul 26, 20-- 01:44:43 PM US/PacificTo : [email protected] : Request to attend conference

Dear Marsha,

I am writing to request permission to attend the three-day “Internet Marketing Conference”in San Francisco, September 13-15. Attached is a brochure describing the seminar. The cost of attending is approximately $1,200, plus transportation (I’m willing to drive), hotel, and meals.

I have spoken with Fred Black, Creative Director of DiPerno and Jones Advertising, who went to last year’s conference in Mexico, and he recommends it highly. As the Marketing Manager for Klein Graphics, I think it would benefit the company tremendously to have me learn the latest trends in this important field.

Please let me know if you would like more information from me about the conference or how Iwould expect the training to benefit my job at Klein Graphics.

Thank you in advance,

Tara Branson”---------------------------------------------------------------------------------------------------------------------------“IMC San Francisco 2006

- The 9th Internet Marketing Conference- Date: 13-15 September- Location: San Francisco Conference Center255 South Airport BoulevardSouth San Francisco, CA 94080

The best Internet Marketing Conference comes to San Francisco. Learn Internet marketing, meet industry

Page 61: 101s1.downloadmienphi.net/file/downloadfile2/180/1403331.doc · Web viewDuring normal office hours, University Maintenance will respond directly to a power failure. After hours, the

professionals and get in-depth overviews of everything you need to know about the fast-paced, web-based world.

Three days of Internet marketing immersion Tuesday, September 13th - “Business Day”- a great business concept is the key to Internet marketing success. Get business-savvy.

Wednesday, September 14th - “Technology Day”- technologies and new developments that your Internet business needs to know now.

Thursday, September 15th - “People Day”- People are the key to organizational success. Develop the means to focus on your most important assets, your staff and customers.

After three days at the IMC 2006 conference, you will gain a head start on the competition with the best people skills, superior marketing techniques and a great business concept.

Expert advice. Expert results.IMC 2006 welcomes experts from across the Internet marketing industry. More than 30 topics will be discussed from search engines, domain names and analytics to blogs, pay-per-click and rich media. Everything you ever wanted to know about Internet marketing and more.RegistrationThe registration fee for IMC 2006 is $1,190 for the full three-day conference. If you are looking for guaranteed Return On Investment, IMC 2006 is the ultimate investment. Discover the potential of Internet Marketing.”

196. What is Tara Branson’s job?(a) Graphic Designer(b) Creative Director(c) Marketing Manager(d) Advertising Director.197. What is learned about the Internet Marketing Conference?(a) It costs more than $1,200.(b) It is being held in a hotel.(c) It changes location every year.(d) It takes place over a three-day weekend.198. What is expected to be the focus on the second day of the Internet Marketing Conference?(a) Business(b) Technology

(c) Human resources(d) Government guidelines.

199. What does Tara Branson claim to have done?(a) Attended the previous year’s conference(b) Received permission from her supervisor(c) Negotiated a discount on registration fees(d) Spoken with a previous conference attendee.

200. What does Tara Branson offer to do?(a) Drive to the conference(b) Pay her own daily expenses(c) Put together a detailed budget(d) Write a report when she returns.

ANSWER101. B 102. B 103. C 104. D 105. C 106. D 107. C 108. A 109. A 110. D111. D 112. D 113. A 114. C 115. A 116. C 117. B 118. C 119. A 120. C121. B 122. C 123. C 124. B 125. C 126. C 127. A 128. C 129. B 130. D131. B 132. B 133. D 134. A 135. C 136. D 137. D 138. B 139. B 140. A141. A 142. B 143. D 144. A 145. D 146. C 147. B 148. D 149. A 150. D151. A 152. C 153. B 154. A 155. A 156. C 157. B 158. D 159. C 160. B161. B 162. C 163. D 164. B 165. A 166. D 167. C 168. D 169. B 170. B171. C 172. B 173. B 174. B 175. A 176. D 177. A 178. B 179. D 180. C181. C 182. C 183. A 184. C 185. D 186. D 187. C 188. B 189. D 190. D191. B 192. C 193. A 194. B 195. C 196. C 197. C 198. B 199. D 200. A

Page 62: 101s1.downloadmienphi.net/file/downloadfile2/180/1403331.doc · Web viewDuring normal office hours, University Maintenance will respond directly to a power failure. After hours, the

101. Children younger than 5 years old are _______ free to festival events. (a) admit(b) admitted(c) admission(d) admittedly

102. Electronics retailer Buyer's World announced plans to _______ rebates on software purchases. (a) forecast(b) transport(c) eliminate(d) compromise

103. Mr. Barry oversaw the development of the Nissan Building _______ its initial stages to its completion. (a) of(b) to(c) from(d) about

104. WCM's quarterly report _______ the gains the

computer company made in its software division. (a) reflect

(b) reflects(c) reflector

(d) reflection

105. Beginning in August, free parking _______ available to anyone taking the commuter train from Green City.

(a) were(b) to be

(c) being(d) will be

106. _______ offices were set up in the conference room

until building renovations were completed. (a) Test

(b) Form(c) Invented

(d) Makeshift

107. Ms. Jefferson asked her secretary to _______ the pamphlets into three categories. (a) separate(b) separation(c) separately(d) separateness

108. _______ most of his co-workers, Chris prefers to arrive early for work rather than stay late. (a) Among(b) Except(c) Unlike(d) Following

109. Please _______ the text to the left side of the document and place all titles in bold letters. (a) align(b) spell(c) degree(d) contrast

110. _______ individual merchants provide product prices, Computrade.com is not responsible for inaccurate price information. (a) For(b) While(c) However(d) Because

111. Cornell SDX has _______ $50 billion in assets, making it the largest investment fund in the market. (a) caused(b) borrowed(c) prevented(d) accumulated

112. Employee contracts are eligible for _______ each year. (a) renew(b) renewal(c) renewed(d) renewing

113. _______ the simplified characters used on the Chinese mainland, traditional Chinese characters are used for writing in Taiwan and Hong Kong. (a) As far as(b) On behalf of(c) In regards to(d) In contrast to

114. The latest issue of Traveler's Monthly _______ an article on the magazine's founder. (a) wrote(b) profits(c) features(d) captures

Page 63: 101s1.downloadmienphi.net/file/downloadfile2/180/1403331.doc · Web viewDuring normal office hours, University Maintenance will respond directly to a power failure. After hours, the

115. Storeowners removed outdoor signs and decorations in _______ of the windstorm. (a) damages(b) relation(c) recovering(d) anticipation

116. After _______ in Italy for two years, Carmen was able to speak Italian fluently. (a) live(b) lived(c) living(d) livable

117. The sales tax _______ from the Oakdale Shopping Outlets is used to benefit residents of Oakdale County. (a) revenue(b) display(c) banquet(d) language

118. A power outage _______ the intersection of Prospect and Finnegan Roads was reported. (a) out(b) near

(c) since(d) during

119. Due to subway system repairs, service on line 7 has

been _______ until Tuesday. (a) reserved

(b) suspended(c) collected

(d) distributed

120. Rycell Co. _______ its offer to purchase Buckly-Finch after reevaluating company finances. (a) withdrew(b) withdraw(c) withdrawn(d) withdrawal

121. The evidence suggests people used the machine _______ the late 18th century. (a) off(b) plus(c) until

(d) between

122. Critics _______ that the plot of the film was too intricate and would confuse most moviegoers. (a) favored(b) produced(c) complained(d) appreciated

123. Founded ten years ago, the Youth Media Network _______ the leading source of print and broadcast news for adolescents. (a) become(b) becomes(c) becoming(d) has become

124. Carrington Medical Group offers employees competitive salaries and benefits _______. (a) pack(b) packed(c) packers(d) packages

125. As team leader, Yvette is responsible for _______ tasks to team members. (a) delegating(b) supporting(c) originating(d) participating

126. Former football player Tito Grimaldi is _______ a commentator and analyst for The Nightly Football Recap. (a) currently(b) carefully(c) purposely(d) personally

127. Bus schedules are updated frequently, _______ check our Web site for the latest departure and arrival times. (a) so(b) if(c) neither (d) whereas 128. Employees are not permitted to refund customer purchases unless _______ by a manager or assistant manager. (a) supervise(b) supervised(c) supervisor

Page 64: 101s1.downloadmienphi.net/file/downloadfile2/180/1403331.doc · Web viewDuring normal office hours, University Maintenance will respond directly to a power failure. After hours, the

(d) supervision

129. For technical questions _______ the Lutan Air Web site, please e-mail [email protected] or call our help desk at (914) 777-4341C. (a) after(b) toward(c) regarding(d) throughout

130. Despite the onset of heavy rains, the outdoor concert was not _______. (a) cancel(b) cancelled(c) canceling(d) cancellation

131. Monthly observations are _______ as part of the employee evaluation process. (a) educated(b) equipped(c) installed(d) implemented

132. The Seville Group must acquire at least one more _______ before it can purchase the property.(a) command(b) obstacle(c) investor(d) demonstration

133. Bakerville's historic waterfront _______ thousands of tourists during the summer. (a) attracts

(b) attraction(c) attractive

(d) attractiveness

134. The state legislature's most recent law fines drivers caught driving more than 10 miles per hour _______ the

speed limit. (a) into(b) above(c) inside

(d) against

135. In neighborhoods near universities, _______ a home into a rental property is generally a good idea. (a) owning(b) deciding(c) decorating(d) converting

136. _______ June 16, four more buses will provide rush-hour service on the Hudson-Cornell route. (a) Defining(b) Building(c) Effective(d) Traveling

137. City guides and maps of city landmarks are available to _______ at the information center. (a) visit(b) visitors(c) visiting(d) visitation

138. By studying under famed pianist Rachel Frangella, Timothy learned to _______ her trademark style and technique. (a) submit(b) distinct(c) reproduce(d) character

139. Dale received _______ plane ticket less than one week before the conference in Mexico. (a) he(b) his(c) him(d) himself

140. Links.com was discredited following reports that some articles _______ exaggerated facts. (a) trained(b) contained(c) succeeded(d) entertained

Page 65: 101s1.downloadmienphi.net/file/downloadfile2/180/1403331.doc · Web viewDuring normal office hours, University Maintenance will respond directly to a power failure. After hours, the

Questions 141-143 refer to the following letter.

From: Victoria BlancheTo: Department managersSubject: Ordering schedule

Beginning August 23, the administrative department will implement a new policy regarding office supply orders. Orders will no longer be placed irregularly, but will instead conform to a schedule.

All department managers should make a list of the supplies needed by their employees and _______ an order form for their department.

141. (a) complete(b) completed(c) completion(d) completeness

Department managers will have the ability to approve or _______ order requests they deem excessive or unnecessary.

142. (a) prove(b) apply(c) reject(d) permit

Orders for new supplies will be placed on the last Monday of every month. Supply request forms must be received from department managers _______ the preceding Friday at the latest.

143. (a) by(b) but(c) either(d) through

Under this system, supplies should arrive at the office no later than the first Monday of the next month.

Thank you,

Victoria Blanche

Questions 144-146 refer to the following letter.

Invest in Romania

Investment registration for companies

The Romanian government offers 4-day business registration for companies like yours. By visiting one of the 12 government offices located throughout the country, you can _______ all the information you will need to register your business.

144. (a) try(b) edit(c) obtain(d) destroy

Each office also provides access to the official Romanian Business Database, where you can find current information on potential business partners, suppliers, manufacturers, and distributors already working _______ Romania.

145. (a) at

Page 66: 101s1.downloadmienphi.net/file/downloadfile2/180/1403331.doc · Web viewDuring normal office hours, University Maintenance will respond directly to a power failure. After hours, the

(b) within(c) during(d) opposite

Additionally, the database will _______ your company to other businesses throughout Europe.146. (a) connect

(b) connected (c) connection(d) connective

Questions 147-149 refer to the following letter.Dear Ms. Weaver,

I regret to inform you that I will not be able to uphold the _______ of my lease. I have been offered a job overseas and will leave the country at the end of December.

147. (a) task(b) terms(c) reading(d) comments

I'm aware that I must forfeit my security deposit of $750, equivalent to one month's rent payment, _______ stated in the early-release clause of my lease.

148. (a) as(b) yet (c) than (d) such

I'm also aware that I'm responsible for paying the _______ three months of rent until a new tenant or sublessee can be found. I'll try my best to locate someone as soon as possible. I appreciate your understanding in this matter.

149. (a) post

(b) addition

(c) gathering

(d) remaining

Sincerely,Mike Hodge

Questions 150-152 refer to the following letter.

The Top Ten Most Environmentally Friendly Cities

Number 8: Danforth

Despite its status as the nation's second largest and third most densely populated city, Danforth _______ both Gambridge and Andover in environmental efficiency.

150. (a) ranges(b) maximizes(c) surpasses(d) determines

First, its high population _______ led to the development of an extensive public transportation system.151. (a) concentrate

Page 67: 101s1.downloadmienphi.net/file/downloadfile2/180/1403331.doc · Web viewDuring normal office hours, University Maintenance will respond directly to a power failure. After hours, the

(b) concentrated(c) concentrating(d) concentration

The Danforth Metropolitan Transportation System is used by more than 73 percent of Danforth citizens, most of _______ do not own cars, to commute to work every day.

152. (a) that (b) whom (c) what (d) which

Second, the city recently formed the Danforth Green Construction Committee (DGCC). The DGCC was established to ensure compliance with environmental standards at land development and building construction sites within the city. The committee has already approved the construction of three buildings in central Danforth that will be built with recycled steel.

September 19

Theresa Long40 Stuart StreetBoston, Massachusetts 02116

Dear Ms. Long,

I was pleased to see your advertisement for a Manager of Financial Services in the Boston Record on September 17. I think my background and experience match well with your requirements. My resume is enclosed for your review.

At my former post as financial manager at the Cromwell Land Initiative, I was able to improve operating efficiency by reducing outstanding balances and expenses. If you are seeking an experienced, successful financial manager, please consider what I have to offer. I would be honored by an opportunity to speak with you further about the position. My phone number is (781) 324-7732.

Thank you for your time and consideration. I look forward to hearing from you.

Sincerely,

Ryan Graham

153. What is the purpose of Mr. Graham's letter?(a) To ask for advice(b) To promote a product(c) To comment on an article(d) To apply for a job opening

154. What did Mr. Graham enclose in the letter?(a) A resume

(b) An advertisement(c) An expense report(d) A newspaper article

The Alexander Wendell Agency (AWA)

The Alexander Wendell Agency is the leading advertising agency in the country. Founded 167 years ago by

Page 68: 101s1.downloadmienphi.net/file/downloadfile2/180/1403331.doc · Web viewDuring normal office hours, University Maintenance will respond directly to a power failure. After hours, the

Alexander R. Wendell, the agency has grown to represent more than 1,000 clients, including Lysik Pharmaceuticals, Phonetel, and Atlas Air.

The AWA currently employs more than 9,000 people and operates more than 300 offices in 80 countries. Ten years ago, CEO Dominique Lawrence created a division called AWA-Ethix to guide AWA clients in implementing socially responsible campaigns and projects.

155. What is true about the Alexander Wendell Agency?(a) It is a leading travel agency.(b) It employs less than 1,000 people.(c) It does not have overseas offices.(d) It represents about 1,000 clients

156. According to the information, what did Ms. Lawrence do? (a) Appoint a new CEO(b) Found the company(c) Start a new program

(d) Resign from her post.Special Offer

Subscribe to Photography Today and get 60 percent off the newsstand price.

Please select one of the following options:__x_ Send me 12 issues (one year) for $21____ Send me 24 issues (two years) for $42

Subscriber InformationE-mail address: [email protected] name: Tonette HawkinsMailing address: 251 West 38th StreetBayonne, New Jersey 07002Phone number: (201) 858-1110

157. What is Ms. Hawkins requesting(a) Internet service(b) A concert ticket(c) Camera accessories(d) A magazine subscrip

158. How much will Ms. Hawkins be billed? (a) $12(b) $21(c) $24(d) $42.

Travel Free with the North-Rail Rewards Program

With the North-Rail Rewards Program, your business will earn travel points for every North-Rail business-class ticket purchased with a company credit card. Travel points can later be redeemed for commuter train tickets, allowing businesses with employees who travel often to ride North-Rail trains for free to presentations, meetings with clients, and other business-related events.

Signing up to receive travel rewards is easy:* First, visit www.north-rail.com/business and register your company's credit card information.* Next, choose up to five employees who will have authorization to use the company credit card to purchase North-Rail tickets.* Then, visit our Web site and complete a reservation form to reserve tickets, or call 1-800-447-3339. You can also purchase tickets at any regional North-Rail train station.

159. What type of tickets must customers purchase?(a) One-way tickets

(b) Round-trip tickets(c) First-class tickets

Page 69: 101s1.downloadmienphi.net/file/downloadfile2/180/1403331.doc · Web viewDuring normal office hours, University Maintenance will respond directly to a power failure. After hours, the

(d) Business-class tickets

160. What information are customers asked to provide?(a) Their travel dates(b) Their travel itinerary(c) The names of five employees(d) The city they are traveling to.

161. What is NOT listed as a way for customers to purchase tickets?(a) Visiting an office(b) Calling a phone number(c) Going to a train station(d) Completing an online form.

From: The Hilton Preservation Society [[email protected]]To: Doreen Arie [[email protected]]Subject: A special offer for HPS membersDate: September 8

Dear Ms. Arie,

As a valued member of the Hilton Preservation Society, we would like to extend the following offer to you on behalf of Century Card.

The new Century Platinum Card offers you a chance to support the Hilton Preservation Society every time you use it. Each time you use your Century Platinum Card, a donation equaling 15 percent of your purchase will be made to the Hilton Preservation Society's Restoration Campaign to help restore historic buildings and landmarks. Century Bank's Century Platinum Card also offers HPS members these additional benefits:

(A) Discounts at affiliated hotels and airlines,(B) 0% APR on high-interest balance transfers and cash advances,(C) No annual fee and low APR on all purchases, and(D) Secure online access to your account information.

Page 70: 101s1.downloadmienphi.net/file/downloadfile2/180/1403331.doc · Web viewDuring normal office hours, University Maintenance will respond directly to a power failure. After hours, the

162. What is the purpose of the e-mail? (a) To request membership fees(b) To advertise an upcoming event(c) To offer Ms. Arie a credit card service(d) To remind Ms. Arie to renew her membership

163. For what will money be donated?(a) Restoring historic buildings(b) Funding environmental projects(c) Producing a monthly newsletter(d) Supporting political campaigns

164. What is stated about annual fees?(a) They are not collected.(b) They depend on account activity.(c) They can be paid in installments.(d) They make up 15 percent of funding.

Page 71: 101s1.downloadmienphi.net/file/downloadfile2/180/1403331.doc · Web viewDuring normal office hours, University Maintenance will respond directly to a power failure. After hours, the

From: Daniel Travers [[email protected]]To: Sandrine Walters [[email protected]]Subject: Your Sound Byte order

Dear Ms. Walters,

Thank you for purchasing from the Sound Byte online store. Your order has been received, processed, and will be shipped within 2 business days. Please review this summary of your order and notify customer service if there are any discrepancies.

Order Reference Number: 7833091

Order SummaryBilling addressSandrine Walters12 Market StreetSaint Louis, Missouri 63103

Shipping addressSame as billing address

Order DetailsQuantity Description Item Price Item Total----------------------------------------------------------1 Greyson 5-disc CD Player $129.99 $129.99with Remote ControlSubtotal $129.99Tax $ 7.23Shipping $ 9.08----------------------------------------------------------Total $146.30

Customer service can be reached by responding to this e-mail or by calling 1-800-665-3838. Thank you for your order.

Daniel TraversCustomer Service DepartmentSound Byte Online

Page 72: 101s1.downloadmienphi.net/file/downloadfile2/180/1403331.doc · Web viewDuring normal office hours, University Maintenance will respond directly to a power failure. After hours, the

165. According to the e-mail, what happened to Ms. Walters' order? (a) It was shipped.(b) It was modified.(c) It was cancelled.(d) It was processed

166. How many items did Ms. Walters order?(a) One

(b) Three(c) Five(d) Seven.

167. What is included in Ms. Walters' order?(a) A free CD(b) A remote control(c) A factory rebate(d) A 14-day warranty.

Page 73: 101s1.downloadmienphi.net/file/downloadfile2/180/1403331.doc · Web viewDuring normal office hours, University Maintenance will respond directly to a power failure. After hours, the

Attention employees:

Last week I met with Mr. Hendrix to discuss our overseas marketing campaigns. As you know, we now have retail stores operating in South Africa and Argentina, in addition to having our products carried by department stores in France, China, and Japan.

Although our sales are at a satisfactory level, Mr. Hendrix would like to increase our brand recognition abroad to make Fields Sports' apparel more competitive with both local and foreign equivalents. This will also help us introduce our sporting equipment line to international consumers.

To discuss this matter further, I would like the entire marketing staff to meet on Wednesday, March 14 at 11 A.M. At the meeting, we will discuss ways to further the presence of Fields Sports' apparel through special events, sponsorship, celebrity endorsement, and any other tactics.

Thank you and see you on Wednesday.

Charlize

168. In what country are Fields Sports' products NOT sold? (a) In China(b) In Japan(c) In France(d) In Australia

169. What is indicated about Fields Sports?(a) Their sales are declining.(b) They primarily produce sports apparel.(c) They host a variety of sports competitions.(d) Their products can only be purchased in department stores.

170. When will the staff meeting be held? (a) On March 11(b) On March 12(c) On March 14(d) On March 20

171. What topic will be addressed at the staff meeting?(a) Increasing brand recognition(b) Sponsoring local sports teams(c) Introducing new products overseas(d) Opening a retail store in South Africa.

Page 74: 101s1.downloadmienphi.net/file/downloadfile2/180/1403331.doc · Web viewDuring normal office hours, University Maintenance will respond directly to a power failure. After hours, the

Ashton Black Named President of InfoQuestBy Karina Todd

In November, Ashton Black was appointed President of InfoQuest after serving seven years as their chief financial officer.

Black, who attended Yale School of Management near his hometown in Connecticut, began his career as a financial analyst at Robertson, Inc. Soon after, he was promoted to Head of Research at the brokerage firm.

Working alongside Burt Edwards, named CEO of InfoQuest in June, Black plans to make InfoQuest more competitive. Despite earning nearly $775 million in profits on advertising sales, the company still struggles to compete with rival company Database.com. InfoQuest also faces growing competition from companies emerging in the industry, and is looking for ways to rebuild company morale, attract employees with innovative ideas, and produce better results for loyal investors.

Industry analysts are not yet certain whether Ashton Black is the right man for the job. “Being president of the company will require more leadership from Black than ever before,” said Theresa Spears, chief financial analyst at Mires Investments. ”InfoQuest needs to do a lot of work if they want to keep up with Database.com, and Black seems reluctant to change InfoQuest’s business strategy.”

172. The word “serving” in paragraph 1, line 1, is closest in meaning to:(a) Helping(b) Presenting(c) Completing(d) Encouraging.

173. When did Mr. Edwards become CEO of InfoQuest?(a) Seven years ago(b) One year ago(c) In June(d) In November.

174. What is true about Mr. Black? (a) He plans to retire soon.

(b) He is originally from Connecticut.(c) He serves as a board member at Database.com.(d) He has worked at InfoQuest his entire career.

175. What is InfoQuest's greatest concern?(a) Declining profits(b) An unqualified staff(c) Debt from construction projects(d) Competition from other companies.

176. What does Ms. Spears suggest about Mr. Black's leadership?(a) It will attract more investors.(b) It will raise the company's status.(c) It will conflict with that of Mr. Edwards.(d) It will not change the company's situation.

Pristine AirTips for traveling with children

Here are some things you can bring on your trip that are sure to keep children occupied and relaxed during a long flight.

* Paper with colored pencils or crayonsThis allows parents and children to do activities together, or for children to do something independently. With blank paper and colored pencils or crayons, children can write, draw, or make their own games. For more excitement, pack small figures to use as board-game pieces in your carry-on luggage and let children design their own game boards.

Page 75: 101s1.downloadmienphi.net/file/downloadfile2/180/1403331.doc · Web viewDuring normal office hours, University Maintenance will respond directly to a power failure. After hours, the

* A deck of cardsParents can teach children their favorite card game or show them card tricks. In return, children can do the same for parents.

* BooksIt is always a good idea to pack a book on a long trip. Children will especially enjoy stories that take place in the country to which they are traveling. This will give them a chance to preview the culture and history they will learn about on vacation.

* Gifts from Pristine AirOnce your international flight has taken off, a flight attendant will offer any children traveling with us a picture map of the world and a souvenir passport with stickers that children can affix once they have visited a country. There is also a children’s airline magazine in the pouch in front of your seat.

177. According to the information, what is difficult for children?(a) Long car rides(b) International flights(c) Going shopping with parents(d) Waiting for doctor's appointments.

178. What are children encouraged to ma? (a) A schedule(b) A game board(c) A shopping list(d) A family portrait.

179. What is mentioned as a possible activity?(a) Taking a walk(b) Singing songs(c) Playing cards(d) Watching a movie.

180. What is NOT offered to children?(a) A map(b) Snacks(c) Stickers(d) A magazine.

Page 76: 101s1.downloadmienphi.net/file/downloadfile2/180/1403331.doc · Web viewDuring normal office hours, University Maintenance will respond directly to a power failure. After hours, the

City Cuts Spending for Rink RenovationsBy Niko Lennox

March 29 - Over the weekend, city officials postponed the project to renovate Dennis Park's ice skating rink indefinitely. Officials stated that the move was prompted by the project's rising cost.

The site of the ice skating rink, named after its founding donor Elouise Dennis, was being developed by Y&B Rinks, a facilities management service from Ottawa. Last year, Y&B Rinks submitted an application to the city board for funding to renovate the rink and received approval. Recently, however, unforeseen problems with water drainage forced the company to reassess its funding needs, and the city would not approve a second request for finances.

The unfinished renovations to the park included expanding the rink to accommodate more than 1,000 skaters, adding a retractable roof, and building indoor locker rooms. -------------------------------------------------------------------------------------------------------------------------

Dear Mr. Stewart,

I read an article in last week's newspaper about the city government's rejection of your second request for funding to renovate Dennis Park Rink. As a child, I often visited the rink with my family, and through the years ice-skating has developed into one of my favorite leisure activities.

I'd like to see thousands of children in the Ottawa area experience the same joy I did at Dennis Park Rink. For this reason, I would like to fund the renovation efforts you have so graciously begun. I'd like to meet with you at your earliest convenience to discuss how we can collaborate to improve this valuable community resource. Please call me as soon as possible at (970) 845-7628.

Sincerely,

Steadman Murphy

181. The word “move” in paragraph 1, line 2 of the article, is closest in meaning to:(a) Movement(b) Decision(c) Activation(d) Inspiration.

182. Why did Y&B Rinks request more money?(a) There were problems during construction.(b) They wanted to hire more construction employees.(c) Construction materials were more expensive than expected.(d) They did not receive all of the funding they were promised.

183. What change was NOT part of the project? (a) Building a roof

(b) Adding stadium seats(c) Making the rink larger(d) Constructing locker rooms.

184. Why does Mr. Murphy write the letter?(a) To offer funding to Y&B Rinks(b) To suggest a fundraising idea(c) To motivate citizens to support the project(d) To encourage the city government to fund the project.

185. What does Mr. Murphy imply in his letter?(a) He used to be in the city government.(b) He read the March 29 newspaper article.(c) He lived near Dennis Park Rink as a child.(d) He enrolled his children in ice-skating lessons.

Page 77: 101s1.downloadmienphi.net/file/downloadfile2/180/1403331.doc · Web viewDuring normal office hours, University Maintenance will respond directly to a power failure. After hours, the

From: Customer service [[email protected]]To: Yuna Dautry [[email protected]]Subject: Coupon offer

Dear Ms. Dautry,

We apologize for the mistake we made in processing your October 28 order of item #99820 from our Web site. We have since corrected the error in our computer system and shipped the item you originally requested. The item was shipped on November 11 and should arrive within 5 to 7 business days.

Because your business is important to us, we have waved the shipping fee on your revised order. We would also like to present you with a 50 percent off coupon to be used on your next purchase in our online store. Please find the coupon as an attachment to this e-mail.

Sincerely,

Lynn RussoCustomer ServiceOffice Supplier.-------------------------------------------------------------------------------------------------------------------------Office Supplier

50 percent off your next office supply purchase at www.officesupplier.com

This coupon may not be used on computer equipment, software, and other electronics. This coupon is not valid in conjunction with other offers, on shipping, or on gift card purchases. This coupon is only valid for one-time use on online orders of in-stock items.

This coupon is not valid on orders with shipping addresses outside of the United States. Office Supplier reserves the right to choose the method of delivery for each order. Delivery fees are nonrefundable if the order or part of the order is returned, unless the product is damaged or defective.

*Please enter the following coupon code during checkout: XB7749JK009W

186. When did Ms. Dautry place an order?(a) On October 5(b) On October 7(c) On October 28(d) On November 11.

187. What can be inferred about the item Ms. Dautry ordered?(a) It was damaged.(b) It was not in stock.(c) It was not delivered.(d) It was not the item she ordered.

188. What can Ms. Dautry use the coupon to do?

(a) Order items online(b) Purchase gift cards(c) Buy computer software(d) Pay for shipping costs189. What is true about the coupon?(a) It can be used more than once.(b) It will expire after thirty days.(c) It is valid for international addresses.(d) It cannot be used to buy electronic items.

190. According to the coupon, how can Ms. Dautry receive the discount?(a) By completing an online form(b) By entering a code on the Web site

Page 78: 101s1.downloadmienphi.net/file/downloadfile2/180/1403331.doc · Web viewDuring normal office hours, University Maintenance will respond directly to a power failure. After hours, the

(c) By presenting the coupon to an employee (d) By mailing it to the store with a receipt.

July 8Sophie Boonyarat1 Waterman StreetProvidence, Rhode Island 02906

Dear Ms. Boonyarat,

I've enclosed a revision of the banquet menu as we discussed yesterday afternoon. Please review it to ensure all the changes we talked about have been made. Per your request, I added a vegetarian dish. Of the three choices you presented, I thought pasta would be the best option since it can also be offered to non-vegetarian guests.

When you've finished reviewing the menu, please call me at (401) 421-0009 so we can begin preparing for the event.

Sincerely,Bruce GellarGellar & Sons Catering.-----------------------------------------------------------------------------------------------------------------------The Fowler Business AssociationNetworking Banquet - Proposed Menu

Appetizers (served at 5:40 P.M.)Bread: Gourmet Italian rolls topped with garlic butterSalad: House salad with Italian, blue cheese, or honey mustard dressingSoup: Portobello mushroom soup and vegetable minestrone soup

Main Dishes (served at 6 P.M.)Chicken: Honey-glazed chicken served with risotto or a baked potato and mixed vegetablesSalmon: Char-grilled salmon served with risotto or a baked potato and mixed vegetablesFettuccini (vegetarian): Fettuccini noodles served with spinach in a cream sauce

Desserts (served at 7 P.M.)Cake: Chocolate or angel food cakeIce cream: Vanilla, chocolate, or strawberryFruit: An assortment of bananas, apples, oranges, and grapes

Beverages (available throughout the evening)Water, assorted soft drinks, iced tea, grape juice, and apple juice.

191. What type of event will take place?(a) An awards dinner(b) A lunch conference(c) A networking banquet(d) A restaurant opening.

192. Who is Mr. Gellar?(a) A caterer(b) An event planner(c) A restaurant owner

(d) An organization's president.

193. Which item was recently added to the menu? (a) Fruit(b) Salmon(c) Chicken(d) Fettuccini.

194. What appetizer will NOT be served?

Page 79: 101s1.downloadmienphi.net/file/downloadfile2/180/1403331.doc · Web viewDuring normal office hours, University Maintenance will respond directly to a power failure. After hours, the

(a) Portobello cream soup(b) House salad with dressing(c) Fresh vegetables with dip(d) Italian rolls with butter.

195. What is indicated about beverages?(a) They will not be refilled.(b) They will only be available after 6 P.M.(c) They will be served throughout the event.(d) They will stop being served after dessert.

Due to the recent weather and resulting construction delays, the work schedule has been changed for the next two weeks. Hopefully, this will allow us to get back on schedule so the building will be finished by early July, as we originally projected.

Also, it's supposed to rain again towards the end of this week, so the work schedule for Friday is tentative. I'll provide more details about that as we get more information on the conditions for those days.

If you have any questions about the new work schedule, or if you cannot meet the work requirements due to prior obligations, please call me as soon as possible at 903-8821.

Thanks,

Tom.------------------------------------------------------------------------------------------------------------------------------Chopko Construction

On-site supervisor: Tom Chopko, Wendy Chopko

Wednesday May 30

Main task: Installation of roof structure framing

Morning shift: Joe Flores, Sam Smith, Dannie Churski

Afternoon shift: Pat Oh, Leslie Jera, Jay Capshaw

Thursday May 31

Main task: Roof installation

Morning shift: Jay Capshaw, Pat Oh, Sam Smith

Afternoon shift: Joe Flores, Dannie Churski, Leslie Jera

Friday June 1

Main task: Roof installation

Morning shift: Dannie Churski, Leslie Jera, Jay Capshaw

Afternoon shift: Sam Smith, Joe Flores, Pat Oh

Page 80: 101s1.downloadmienphi.net/file/downloadfile2/180/1403331.doc · Web viewDuring normal office hours, University Maintenance will respond directly to a power failure. After hours, the

196. Why was the work schedule changed?(a) Because of a holiday(b) Because of the weather(c) Because of employee absences(d) Because of budget restrictions.197. When is the building expected to be finished?(a) In late June(b) In early July(c) In late July(d) In early August.198. What is indicated about the schedule for June 1? (a) It might change.(b) It will be posted on Thursday.

(c) It is only for half of the day.(d) It must be approved by Mr. Chopk.199. According to the schedule, on what section of the building will employees work?(a) The roof(b) The floors(c) The windows(d) The foundation.200. Who will NOT work during the morning shift on May 30?(a) Pat Oh(b) Sam Smith(c) Joe Flores(d) Dannie Churski.

ANSWER101. B 102. C 103. C 104. B 105. D 106. D 107. A 108. C 109. A 110. D111. D 112. B 113. D 114. C 115. D 116. C 117. A 118. B 119. B 120. A121. C 122. C 123. D 124. D 125. A 126. A 127. A 128. B 129. C 130. B131. D 132. C 133. A 134. B 135. D 136. C 137. B 138. C 139. B 140. B141. A 142. C 143. A 144. C 145. B 146. A 147. B 148. A 149. D 150. C151. D 152. B 153. D 154. A 155. D 156. C 157. D 158. B 159. D 160. C161. A 162. C 163. A 164. A 165. D 166. A 167. B 168. D 169. B 170. C171. A 172. C 173. C 174. B 175. D 176. D 177. B 178. B 179. C 180. B181. B 182. A 183. B 184. A 185. B 186. C 187. D 188. A 189. D 190. B191. C 192. A 193. D 194. C 195. C 196. B 197. B 198. A 199. A 200. A

Page 81: 101s1.downloadmienphi.net/file/downloadfile2/180/1403331.doc · Web viewDuring normal office hours, University Maintenance will respond directly to a power failure. After hours, the

101. Every Saturday afternoon in a parking lot on Cedar Street, collectors come to _______ off their prized vintage cars. (a) show(b) turn(c) hold(d) watch

102. Everybody _______ has a company-issued cellular phone should e-mail their name and phone number to Cassandra Miller by 5 P.M. tomorrow. (a) who(b) whom(c) which(d) where

103. The most _______complaint among the staff is not having enough opportunities for professional development. (a) common(b) commonly(c) is common(d) is commonly

104. Because this month has been unusually _______, my

team and I have been able to take the time to clean out file cabinets and archive old project materials.

(a) slow(b) busy

(c) late(d) active

105. The number of people submitting new _______ for

jobless aid dropped by 11,000 last week. (a) claim

(b) claims(c) claimed

(d) claiming

106. When we _______ the world's economic development on a timeline we can see periods in which

certain business sectors experienced an economic boom. (a) examine

(b) examines(c) examining

(d) examination

107. Business was slow overall on Wall Street this spring, but Richardson Holdings still _______ to show strong results. (a) earned(b) spread(c) traded(d) managed

108. The _______ retail price for the shoe polish is $3.99, roughly 25 percent lower than other shoe polishes currently on the market. (a) suggest(b) suggested(c) suggesting(d) suggestion

109. The board will take until Monday to choose between two competing bids: one from Neptune, and the _______ from an unnamed Russian conglomerate. (a) ever(b) other(c) either(d) neither

110. Political leaders have reached an _______ on a health bill that will extend health coverage to millions of Americans. (a) agree(b) agreed(c) agreeing(d) agreement

111. The meeting with Mr. Kim _______ until Tuesday, December 15th, at 2:00 p.m. (a) postponed(b) had postponed(c) has postponed(d) has been postponed

112. The report noted that agriculture consumes _______ 95 percent of all the water humans use. (a) widely(b) foully(c) roughly(d) publicly

113. Consumer confidence fell in August to its lowest _______ since November 2004. (a) point(b) price(c) attendance

Page 82: 101s1.downloadmienphi.net/file/downloadfile2/180/1403331.doc · Web viewDuring normal office hours, University Maintenance will respond directly to a power failure. After hours, the

(d) enrollment

114. Daniel Peterson, Trapani's new CEO, has _______ to hold company-wide meetings every three months. (a) promise(b) promises(c) promised(d) promising

115. The hotel provides free shuttle _______ to the train station and the airport. (a) room(b) service(c) baggage (d) shipping116. The suburbs north and east of the city may get as much as eleven inches of _______ by tomorrow morning. (a) snow(b) snows(c) snowed(d) snowing

117. Governor Hester sees small business as the best hope for _______ the state's economy. (a) revitalize(b) revitalized(c) revitalizing(d) revitalization

118. You cannot, as a manager, _______ entirely on statistics to gauge employee performance, because statistics can often be misleading. (a) rely

(b) trust(c) expect

(d) convince

119. More of the world's hot chiles _______ from Central Mexico than from any other part of the world.

(a) eat(b) come

(c) made(d) grow

120. The city's median real estate prices declined 3.4 percent this year, according to an _______ by the Home Realty Market Report. (a) analyze(b) analysis

(c) analyzed(d) analyzing

121. The value of _______ mid-sized cars that get good mileage has soared alongside the rising price of gasoline. (a) use(b) uses(c) used(d) using

122. John O'Connell recommends that we _______ to a new anti-virus software package. (a) delete(b) switch(c) replace(d) purchase

123. Heffitter Inc. strongly encourages its _______ to use all their vacation days. (a) employ(b) employees(c) employing(d) employment

124. Mr. Hansen was born in Denmark, grew up in Germany, went to university in England, _______ earned his law degree in the United States. (a) or(b) so(c) and(d) also

125. Rent for a one-bedroom apartment in the center of town _______ $900 per month. (a) averages(b) is average(c) was averaged(d) has an average

126. The Bookstore will close for inventory on Thursday at noon; we will open in the morning as usual but no books will be sold _______ noon. (a) after(b) since(c) almost(d) besides

Page 83: 101s1.downloadmienphi.net/file/downloadfile2/180/1403331.doc · Web viewDuring normal office hours, University Maintenance will respond directly to a power failure. After hours, the

127. Peel the potatoes into thin slices and lay them on a baking sheet, overlapping them slightly, until the sheet is _______. (a) cover(b) covers(c) covered(d) covering

128. April is a special time _______ the colorful landscapes of Holland, when“ tulip season” is in full bloom! (a) experienced(b) experiencing(c) to experience(d) have experienced129. The GT500 is designed to operate normally in temperatures _______ from -10°C to 40°C. (a) ranging(b) heating(c) freezing(d) operating

130. Students cannot leave the library to return to the classroom or to go to any other area in the building without _______ from the librarian. (a) permit(b) permitted(c) permission(d) permitting

131. All _______ with the client must be cleared with Barbara Sommers in the legal department. (a) statements(b) transmission(c) communication(d) announcements

132. Boulder College is actively recruiting professors who are well-known in _______ fields.(a) his(b) its(c) those(d) their

133. Life in the City, Quentin Nagano's first album in five years, _______ in stores and online on August 15th. (a) to release

(b) has released(c) is a release

(d) will be released

134. To ensure that the work is completed _______ the

deadline, I recommend hiring a part-time assistant for Ms. Marshall.

(a) by(b) of(c) in(d) over

135. Bronson's firm is moving to a larger suite in the same office tower where it is _______ located. (a) mostly(b) largely(c) recently(d) currently

136. Wichita, Kansas is to civil aviation _______ Detroit used to be to automobiles: the center of the industry. (a) what(b) when(c) where(d) which

137. If you are thinking about _______ to the beach for the weekend, don't miss WBNC's up-to-the-minute weather forecasts. (a) go(b) gone(c) going(d) to go

138. Simon Capital owns interests in several local malls, including the Springfield and Montgomery malls, which have been seeing steady _______ in sales. (a) decline(b) declines(c) declined(d) declining

139. _______ many teaching hospitals have moved toward privatization, the government still owns most of the nation's medical centers. (a) Unlike(b) Despite(c) Although(d) Throughout

Page 84: 101s1.downloadmienphi.net/file/downloadfile2/180/1403331.doc · Web viewDuring normal office hours, University Maintenance will respond directly to a power failure. After hours, the

140. Stock prices surged yesterday, as _______ that two companies were seeking to buy Benko, Inc. raised investors' hopes. (a) a report

(b) to report(c) was reported(d) were reporting

Page 85: 101s1.downloadmienphi.net/file/downloadfile2/180/1403331.doc · Web viewDuring normal office hours, University Maintenance will respond directly to a power failure. After hours, the

Questions 141-143 refer to the following letter.

THIS WEEK IN ART

Connected by history but more than an ocean away, the Canary Islands and San Antonio are getting to know each other better _______ cultural exchanges.

141. (a) under(b) about(c) toward(d) through

Some of _______ city's earliest Old World settlers arrived from the Canary Islands in 1731, and their surnames live on in the names of our streets and neighborhoods - as well as in some of our residents!.

142. (a) a(b) the(c) each(d) those

For this year's Fotoseptiembre Festival, seven contemporary photographers from the Canary Islands are set to attend a reception from 6 to 8 P.M. Tuesday for their exhibit,“ A Ojo De Canario.” Their photos will be on _______ through November 30 at the San Antonio.

143. (a) camera(b) display(c) ceremony(d) attendance

Questions 144-146 refer to the following letter.

To:“ Steve Cashman”From:“ Ivan Cocker”Date: Monday, February 12, 20-- 9:05 p.m.

Subject: Re: Video Presentation----------------------------------------------

Hi Steve,

Regarding the technology you want to use on your upcoming Web-based video presentation, I think you are on the right track. I know that, theoretically at least, it is possible to use JavaScript to control a video player when it is embedded in a Web page. I have also _______ that it's not likely to work across different browsers and platforms - but it's probably okay in an intranet environment where you can control the delivery media.

144. (a) hear(b) heard(c) hears(d) hearing

As for integrating text _______ your video, this is fairly simple, but should not be done with JavaScript. The easy way to do it is to re-edit the video with the words superimposed, and then re-export the video.

145. (a) so(b) with(c) over(d) among

Page 86: 101s1.downloadmienphi.net/file/downloadfile2/180/1403331.doc · Web viewDuring normal office hours, University Maintenance will respond directly to a power failure. After hours, the

I am available to take on either or both of these projects, so if you decide to go forward with this work, just let me _______.

146. (a) know(b) known(c) knowing(d) knowledge

Questions 147-149 refer to the following letter.GET 50% OFF THE SUNDAY PAPERWhen You Purchase Your Favorite Magazine!

Beginning February 17, _______ the Denver Express News every Sunday for 50% off the newsstand price when you purchase one of these magazines at a participating FER Grocery Store:

* Natural Life* Men's Quarterly* Denver Tonight!* Cooking.

147. (a) receive(b) receives(c) received(d) receiving

Not only will you get your favorite magazine, but you will _______ enjoy the Sunday paper,148. (a) too

(b) also(c) either(d) besides s

which comes packed with money-saving coupons, fascinating features, the latest sports highlights and in-depth news _______!Talk about a super Sunday! Stop by your local FER today!

149. (a) story

(b) article

(c) coverage

(d) reporter

Questions 150-152 refer to the following letter.

Glacier Real EstateP.O. Box 202801Helena, MT 59620

August 2, 20—

Roberta Van Stansen225 Cruise Avenue

Page 87: 101s1.downloadmienphi.net/file/downloadfile2/180/1403331.doc · Web viewDuring normal office hours, University Maintenance will respond directly to a power failure. After hours, the

Helena, MT 59601

Dear Ms. Van Stansen,

I am a real estate broker with Glacier Real Estate and _______ had a client call me regarding a Montana City/East Helena property that is listed in your name. The property is located at:

150. (a) daily(b) clearly(c) rapidly(d) recently

1 Jackson Creek RoadMontana City, Montana MT 59634

My client is very _______ in exploring the possibility of purchasing this property from you. If this is of interest to you, I would appreciate you contacting me at your earliest convenience. You can call me at (406) 458-8840.

151. (a) open(b) probable(c) concerned(d) interested

Thank you very much for your _______. I look forward to hearing from you soon. 152. (a) vendor

(b) proposal(c) attention(d) possession

Best Regards,Harper RathmannGlacier Real Estate

Stereo City's New CEO Alters Reporting Plans

In his first public move as Stereo City Corp.'s new chief executive, Robert Berumen scrapped quarterly chats with industry analysts.

Instead of conducting quarterly conference calls, the company will provide financial results by issuing a“ comprehensive press release.” Industry analyst and television host Donna Nees commented that“ increasingly, companies are abandoning the conference calls with analysts.”

The consumer electronics retailer also said it will communicate details on its progress strictly through quarterly news releases and required government filings.

Stereo City spokesman Greg Lively said that the company would announce second quarter financial results on July 21.

153. How will Stereo City release its quarterly reports?(a) On its Web site

(b) In a conference call(c) Through a press release(d) Via a live television broadcast

Page 88: 101s1.downloadmienphi.net/file/downloadfile2/180/1403331.doc · Web viewDuring normal office hours, University Maintenance will respond directly to a power failure. After hours, the

154. Who made the decision to change the way results are reported?(a) Donna Nees

(b) Greg Lively(c) The new CEO

(d) A consultantBenjamin Watson Senior Center120 West Main StreetSt. Louis, MO

January 22, 20—

Richards & Leeds Accounting Services550 2nd AvenueSt. Louis, MO

To our friends at Richards and Leeds Accounting Services,

On behalf of our executive board and all our members I would like to thank you for your continued support of the Benjamin Watson Senior Center.

As you know, the Benjamin Watson Senior Center provides essential services to senior citizens throughout the metropolitan St. Louis area, and is funded entirely through local contributions. Your continued support allows us to continue to fulfill our mission.

Thank you again for your donations throughout the year. We look forward to seeing you at this year's community spaghetti dinner, to be held March 19th at the center.

Best regards,

Anthony Klein, DirectorBenjamin Watson Senior Center

155. Why has Anthony Klein written this letter?(a) To announce an upcoming fundraising event(b) To thank a supporter for making a donation(c) To ask for money to support a senior center(d) To announce the results of a fundraising drive

156. What is learned about the Benjamin Watson Senior Center? (a) It is celebrating an anniversary.(b) It is having financial difficulties.(c) It is allied with a national network.

(d) It is supported entirely by donations.Got Gas?

If you would like to speak to someone about converting your home to Timmler natural gas heat or are interested in natural gas products, call 1-800-GAS-9999, ext. 76. You can also go to www.timmlerenergydelivery.com or fill out and return this reply card. A representative will contact you within a week.

Name: ________Address: ___________________________________________City: ________ State ________ Zip ________Telephone (___) _______________

Page 89: 101s1.downloadmienphi.net/file/downloadfile2/180/1403331.doc · Web viewDuring normal office hours, University Maintenance will respond directly to a power failure. After hours, the

Best time to call ________ A.M. ________ P.M.

I am interested in natural gas for:___ Heating ___ Cooking___ Water heater ___ Clothes Dryer___ Gas fireplace ___ Outdoor grill___ Pool heater ___ Other: _________________

What is the approximate age of your furnace or boiler?___ Less than 10 years___ More than 10 years

Timmler Energy Delivery

* Offers on natural gas heating equipment only available to homeowners in Brooklyn, Queens, Staten Island, and Long Island where Timmler Energy Delivery gas service is available, and who already use natural gas in their homes but not for heating. Customer is required to pay for the installation of the heating equipment. All programs and incentives are subject to change and some restrictions may apply.

157. Why would someone call 1-800-GAS-9999?(a) To schedule a furnace delivery(b) To file a complaint about service(c) To discuss problems related to billing(d) To arrange for natural gas service for a residence

158. What information does the form ask people for? (a) The age of their heating equipment(b) The account number they are inquiring about(c) The date and time that the problem occurred(d) The number of people who use the water heater each day.

159. What is a restriction on the offer described in the form?(a) Customers must agree to a 2-year service agreement.(b) Employees of Timmler Energy Delivery are not eligible.(c) Homeowners must currently use gas for purposes other than heating.(d) All equipment deliveries must be made during regular business hours

From: Sean NovakTo: All EmployeesDate: Monday, March 20Subject: Visiting Science Reserves Coordinator--------------------------------------------------------------------

Molly Eakins from Science Periodicals Reserves will be setting up shop here in the Business School Library starting January 18, 20--. She will be working here during the duration of the construction of the Interdisciplinary Science Building, which will be going up right next door to us.

Molly will be located in the Business Periodicals Reserves unit and will be available Monday through Friday 9 a.m. - 4:30 p.m. Many of her afternoons will be spent on the construction site, so she has suggested that people who need to see her might have better luck in the morning. Her telephone and e-mail address will remain the same.

160. What does the e-mail announce?(a) The hiring of a new coordinator(b) The creation of a new job position

(c) The start of a construction project(d) The temporary relocation of a staff member.

Page 90: 101s1.downloadmienphi.net/file/downloadfile2/180/1403331.doc · Web viewDuring normal office hours, University Maintenance will respond directly to a power failure. After hours, the

161. What is learned about the Business School Library?(a) It does not have a periodicals reserve section.(b) It has the same coordinator as the Science Library.(c) It is next door to the Interdisciplinary Science Building.

(d) It will be undergoing construction beginning in early January

162. What does the e-mail suggest that people do? (a) Reserve periodicals by telephone or email(b) Try to reach Molly Eakins on weekday mornings(c) Make note of Molly Eakins's new contact information(d) Borrow science periodicals through the business library

Spelling Bee Slated for May 20 at Webb High School

The Parents Action Committee on Life Academic Concerns (PAC-LAC) is preparing for its 18th annual spelling bee.

It is open to children in first through seventh grades and will be held at 10:45 A.M. on May 20th in the auditorium at Webb High School, 108 McPherson Avenue. The registration fee is $10 per student.

If you are interested in enrolling a child, you must complete a registration form and return it along with the registration fee (check or money order - please do not send cash) payable to PAC-LAC with a self-addressed, stamped envelope; one registration per envelope.

Forms and information are available by calling Alfred Soler at 717-9087 between 7 P.M. and 9 P.M. Monday through Friday. The registration deadline is Monday, May 15th.

Mail completed forms to PAC-LAC, P.O. Box 27555, Lansing, MI 48910.

Spelling Bee sample word lists, rules, rehearsal dates, and schedule will be mailed upon receipt of the registration fee.

There will be no refunds.

Page 91: 101s1.downloadmienphi.net/file/downloadfile2/180/1403331.doc · Web viewDuring normal office hours, University Maintenance will respond directly to a power failure. After hours, the

163. What is PAC-LAC?(a) A parents' organization(b) A committee of academics(c) The name of Webb High School's spelling team(d) The company that is sponsoring the spelling bee

164. Who is eligible to participate in the Spelling Bee?(a) Children in grades 1-7(b) Students aged 16 and under(c) All Webb High School students

(d) All children enrolled in Alfred Soler's program.

165. What will be sent out after the registration fee is paid? (a) A t-shirt(b) A registration receipt(c) The rules for the contest(d) The directions to the event

Page 92: 101s1.downloadmienphi.net/file/downloadfile2/180/1403331.doc · Web viewDuring normal office hours, University Maintenance will respond directly to a power failure. After hours, the

The World's Best Telephone Headsets

The Olassen Wireless Telephone Headset represents a revolution in telephone headset design.

Every headset contains two embedded microphones. The first microphone picks up every nuance in your voice before transmission; the second microphone filters out unwanted background noise. The result is an enhanced signal that sounds even better than face-to-face conversation.

This revolutionary microphone technology is further enhanced by a speaker that includes base amplification, so you hear the full voice of the person on the other end of the line.

Find the nearest Olassen retail outlet at www.olassenacoustics.com, or call 1-888-845-9000.

Page 93: 101s1.downloadmienphi.net/file/downloadfile2/180/1403331.doc · Web viewDuring normal office hours, University Maintenance will respond directly to a power failure. After hours, the

166. What is a feature of the Olassen Wireless Telephone Headset?(a) It has two microphones.(b) It has adjustable headphones.(c) It has lightweight earphones.(d) It has a digital volume control.

167. What claim is made about the Olassen Wireless Telephone Headset?(a) It fits comfortably on any adult.

(b) It works with any wireless telephone.(c) It was originally developed for the music industry.(d) It produces better sound than face-to-face conversation.

168. How can the Olassen Wireless Telephone Headset be purchased? (a) Through a Web site(b) At Olassen retail outlets(c) Through the Olassen catalog(d) At several department stores

Page 94: 101s1.downloadmienphi.net/file/downloadfile2/180/1403331.doc · Web viewDuring normal office hours, University Maintenance will respond directly to a power failure. After hours, the

MEMORANDUM

From: Michael Santucci, Senior VP, HRTo: All Slater Systems EmployeesSent: Monday, September 18, 20-- 4:01 P.M.Subject: Slater Systems Annual Compensation Statement---------------------------------------------------------------

Fellow employees,

Within the next week, you will be receiving in your home mail a confidential, personalized Compensation Statement from Slater Systems. The feedback we received from employees on these statements last year was very positive and indicated that we should continue to issue these statements on an annual basis.

Your Compensation Statement details the many items that make up your total compensation, including your base salary, the employer-paid portion of your health benefits, the Slater Systems Retirement Plan and the Company's matching contribution to the Slater Systems 401(k) Savings Plan. Please take the time to review your Statement carefully.

Although I am certain that you will find the Compensation Statement helpful, we would like to hear your opinion. Please call or e-mail your local Human Resources department or me with your thoughts or questions.

Sincerely,

Michael SantucciSenior VP, HRSlater Systems

169. What is learned about Michael Santucci?(a) He is a new employee.(b) He is the CEO of Slater Systems.(c) He has requested a Compensation Statement.(d) He works in the Human Resources department.

170. How will employees receive their Compensation Statements? (a) By e-mail(b) By regular mail(c) By accessing a company Web site(d) By meeting with their supervisors

171. What can be inferred about the Compensation Statements?

(a) Employees find them useful.(b) They are being discontinued.(c) Employees must request them.(d) They are updated twice a year.

172. What is learned about Slater Systems?(a) It contributes to its employees' retirement accounts.(b) It has recently changed its employee benefits package.(c) It conducts salary reviews for all employees annually.(d) It is planning to reduce benefits to certain employees.

Page 95: 101s1.downloadmienphi.net/file/downloadfile2/180/1403331.doc · Web viewDuring normal office hours, University Maintenance will respond directly to a power failure. After hours, the

Dixie Rose Makes Another Stride in Recovery

The St. Louis Derby winner Dixie Rose had the cast changed on his broken hind leg, his veterinarian said yesterday, and the plates and screws that were inserted three weeks ago appear to be working.

The first cast, which was placed on Dixie Rose's leg after four hours of surgery on May 21, was removed under general anesthesia, the University of Missouri School of Veterinary Medicine's Skroll Center said in a news release.

“His leg looks excellent,” Dr. Debra Gregory said in the release. She added that the incision had healed well and the graft was taking.

Dr. Gregory implanted a plate and 27 screws to help the ankle fuse a day after Dixie Rose fell during the Holloway Stakes race in Charleston, South Carolina.

“All of the implants look unchanged,” Dr. Gregory said.“ The prognosis is good. It's unlikely he will race again, but the leg should recover fully.”

Dixie Rose, a three-year-old colt, remains in the intensive care unit at the hospital in Columbia, MO.

173. What is learned about Dixie Rose?(a) He won the St. Louis Derby.(b) He won three of his last four races.(c) He was unable to start the Holloway Stakes.(d) He was not expected to win the Holloway Stakes.

174. Who is Debra Gregory? (a) A horse trainer(b) Dixie Rose's owner(c) The head of the Skroll Center(d) The doctor treating Dixie Rose.

175. How can Dixie Rose's condition be described?(a) As critical(b) As recovered(c) As improving(d) As unchanging.

176. What information is NOT mentioned in the article?(a) How old Dixie Rose is(b) How Dixie Rose became injured(c) How Dixie Rose's injury was treated(d) How much Dixie Rose has earned in winnings.

To: From: Date: November 4, 20-- 2:40:18 PM ESTSubject: [AIE] Your membership ID card will be mailed within the next week------------------------------------------------------

Dear Jack McCarrol,

We are sending you this e-mail message to confirm that your membership ID card/receipt has been printed and will be mailed within the next week. We apologize for delays in processing recently as we upgraded our membership system.

If you would like an electronic copy of your receipt e-mailed to you (in addition to the printed receipt already on its way), please let us know.

Page 96: 101s1.downloadmienphi.net/file/downloadfile2/180/1403331.doc · Web viewDuring normal office hours, University Maintenance will respond directly to a power failure. After hours, the

If you have any questions about your membership, please contact us at [email protected] or telephone 703-836-0662 (toll free in the United States: 888-547-2296). Thank you for being part of AIE!

Sincerely,

AIE Member ServicesAssociation of Industrial Engineers (AIE)P.O. Box 7845Alexandria, Virginia 22314Web http://www.aie.org/

177. Why has this e-mail been sent?(a) To describe the benefits of AIE membership(b) To request additional personal information(c) To apologize for sending something to the wrong person(d) To explain that a delay in processing has been resolved.

178. What can be inferred about Jack McCarrol? (a) He joined AIE online.(b) He recently joined AIE.(c) He runs his own engineering firm.(d) He wrote a letter of complaint to AIE.

179. What information is provided about AIE?(a) A fax number(b) A street address(c) A toll-free phone number(d) A direct number to the manager's office.

180. What is offered to Jack McCarrol in the email?(a) Access to a members-only Web site(b) An electronic version of his receipt(c) Information about upcoming members only events(d) A free listing of his business in the membership directory.

Circulation at the top 10 newspapers

Average paid weekday circulation of the 10 largest U.S. newspapers for the 6 months ended June 30 and percentage change from the same period a year ago:---------------------------------------------------Newspaper Circulation % Change---------------------------------------------------USA Daily 2,272,815 2.09---------------------------------------------------The Wall Street Report 2,049,786 -1.00---------------------------------------------------The New York Messenger 1,142,464 0.05---------------------------------------------------Los Angeles Herald 851,832 -5.40---------------------------------------------------The Washington News 724,242 -3.77---------------------------------------------------New York Daily Star 708,477 -3.75---------------------------------------------------New York Tribune 673,389 0.08---------------------------------------------------Chicago Chronicle 579,079 1.11---------------------------------------------------Houston Bulletin 512,888 -4.11---------------------------------------------------The Phoenix Globe 438,765 3.11 -------------------------------------------------------------------------------------------------------------------------

Page 97: 101s1.downloadmienphi.net/file/downloadfile2/180/1403331.doc · Web viewDuring normal office hours, University Maintenance will respond directly to a power failure. After hours, the

From: Kenneth Behan, Managing Editor, Kelbyville DispatchTo: Editorial and reporting staffDate: April 8, 20—Re: Circulation numbers

Attachments: circulation.doc--------------------------------------------

All,

I've sent you this clipping from the USA Daily on newspaper circulation to make a point. We have been feeling pressure from above to increase circulation. But this year's numbers are up two percent compared to last year's! That is magnificent when you consider that firstly, our population base here in the Kelbyville area has actually shrunk by four percent in the last year; and secondly, almost none of the major newspapers in the nation are doing as well as we are. So I want to say congratulations to everybody. You are doing terrific, terrific work and I am well aware of it.Keep it up.

181. What does the table show?(a) The percent change in circulation from a year earlier.(b) The percentage of market share each newspaper controls.(c) The annual revenue generated by weekday newspaper sales.(d) The total number of newspapers sold over the past 6 months.

182. Which newspaper showed the largest percentage drop in circulation?(a) USA Daily(b) Los Angeles Herald(c) Houston Bulletin(d) The Phoenix Globe.

183. How can the circulation of the New York Tribune be described? (a) As unsteady(b) As variable

(c) As improved(d) As unchanged.

184. What does Kenneth Behan imply about his newspaper?(a) Its sales are growing faster than expected.(b) Its circulation figures compare favorably to other newspapers.(c) Its writers and reporters work harder than those at other newspapers.(d) Its audience is different from most of the area's other daily newspapers.185. What does Mr. Behan point out about his newspaper?(a) It has won more awards than the USA Daily has.(b) Its online version now has more readers than its print version.(c) Its circulation has gone up even as its customer base has decreased.(d) It has a smaller population base than most newspapers with similar circulation.

Page 98: 101s1.downloadmienphi.net/file/downloadfile2/180/1403331.doc · Web viewDuring normal office hours, University Maintenance will respond directly to a power failure. After hours, the

New Travel Statistics Released

U.S. airline passengers last year arrived late to their destinations and lost their luggage more often than passengers in previous years, yet complained less frequently, according to statistics released last Friday by the U.S. Department of Air Transportation.

According to the department's report, 75 percent of all passenger airplanes last year failed to arrive on time. Airplanes the previous year had a 77 percent on-time arrival rate. Additionally, there were 6.7 reports of lost or damaged baggage for every 1,000 passengers, up from 6.6 reports per 1,000 the previous year. That statistic has risen steadily since 2002.

Among national carriers, Colorado Airlines and SkyClear Air had the lowest rates of mishandled baggage, while Northeast Atlantic and BTB Airlines reported the highest. The report also found that Colorado and Limitless Air had the highest on-time arrival rates, while the lowest were recorded by JetHigh Air and Skyward Air.------------------------------------------------------------------------------------------------------------Best airports for on-time arrivals:----------------------------------------------Percentage of on-time arrivals:Cincinnati 83.14Salt Lake City 82.82Phoenix 80.27

Best airports for on-time departures:----------------------------------------------Percentage of on-time departures:Salt Lake City 85.68Cincinnati 84.48San Diego 82.42

Worst airports for on-time arrivals:----------------------------------------------Percentage of on-time arrivals:Newark 62.56New York City 64.16Chicago 68.23Worst airports for on-time departures:----------------------------------------------Percentage of on-time departures:Chicago 68.84Atlanta 71.61Newark 71.87

186. What statistic has risen annually since 2002?(a) The number of late arrivals(b) The number of airline passengers(c) The number of lost or damaged bags(d) The number of passenger complaints.

187. Which of the following airlines reported the fewest mishandled bags?(a) JetHigh Air(b) BTB Airlines(c) SkyClear Air(d) Northeast Atlantic.

Page 99: 101s1.downloadmienphi.net/file/downloadfile2/180/1403331.doc · Web viewDuring normal office hours, University Maintenance will respond directly to a power failure. After hours, the

188. What is implied about Colorado Air? (a) It had problems in several areas.(b) It reported the fewest customer complaints.(c) It had more passengers than most other airlines.(d) It was better than most of its rivals in several categories

189. What was reported as the highest percentage rate for on- time departures?(a) 62.56

(b) 68.84(c) 83.14(d) 85.68.

190. Which airport reported the worst on-time arrival rate?(a) Newark(b) Chicago(c) Phoenix(d) Cincinnati.

SWIMMING WITH SHARKS

Oahu's Most Thrilling Adventure!

Guaranteed Sharks!

- No experience necessary- No age limit- All gear and equipment provided

Observe them in the open ocean from the safety of a shark cage!

A metal cage with strong Plexiglas windows gives you the closest, most exhilarating shark encounter you could ever imagine!

808-222-9393Oahu Shark Adventurewww.oahusharkadventure.com.-----------------------------------------------------------------------------------------------------------------Brendan,

I think I found the perfect activity for Mr. Mendoza when he's visiting the Honolulu offices next month. Check out this Web site: www.oahusharkadventure.com

This might seem a little extreme, but we all know how much Mr. Mendoza loves the ocean and scuba diving. We bring him scuba diving on every trip he takes out here. But I also know that he's fascinated by sharks. The last time he was in Hawaii he was reading a book about them and would bring them up in conversation every chance he had. So I think this would be perfect for him - something different but something he would enjoy very much.

My brother-in-law went on a shark adventure through Oahu Shark Adventure when my sister and her family came to visit me last summer. He said it was very safe, very well organized, and an experience that he will never forget.

So what do you think? Perhaps we can discuss the idea with the whole team on Thursday. We can find out if we have any volunteers to go on a shark-viewing adventure with Mr. Mendoza - if we're all in agreement that it's a good idea.

Page 100: 101s1.downloadmienphi.net/file/downloadfile2/180/1403331.doc · Web viewDuring normal office hours, University Maintenance will respond directly to a power failure. After hours, the

Jung-wha

191. What information does the advertisement provide?(a) A list of prices(b) A schedule of times(c) Recommended sites for visitors(d) Equipment and experience requirements.

192. Why has Jung-wha written the note?(a) To recommend a hotel for Mr. Mendoza(b) To suggest an activity for Mr. Mendoza(c) To ask about Mr. Mendoza's travel plans(d) To inform Brendan about Mr. Mendoza's proposal.

193. What can be inferred about Mr. Mendoza? (a) He is moving to Honolulu.(b) He is writing a travel book.

(c) He will be visiting his brother-in-law.(d) He has traveled to Hawaii several times.

194. What does Jung-wha suggest doing?(a) Delaying a schedule meeting(b) Discussing her idea in a meeting(c) Calling Mr. Mendoza at his office(d) Studying Mr. Mendoza's proposal further.195. In the note, the phrase“ Check out” in paragraph 1, line 2, is closest in meaning to(a) pay for(b) look at(c) leave out(d) ask about.

Staff Profile: Dr. Green

Joseph Green, professor of chemical engineering, is an expert on interfacial processes, particularly on the development and application of x-ray scattering techniques for on-site studies of mineral-fluid interfaces.

Green earned bachelor's and master's degrees, as well as his 1960 Ph.D. in chemical engineering, at the University of Indiana. After four years at the Vivinex Corporation, he joined the faculty of Haverton University in 1963. He left Haverton in 1982 to become senior scientific advisor at Petro Energy Corporate Research Labs, and joined Staunton University's Chemical Engineering Department as professor in 1987.

Green was elected to the U.S. Engineering Council in 1990.------------------------------------------------------------------------------------------------------------------------------United Chemical Laboratories

1000 Corporate Drive

Lacy, NM 82144

February 10, 20—

Joseph Green, Ph.D.

Chemical Engineering Department

College of Chemistry

Staunton University

Menlo Park, CA 94025

Dear Professor Green,

Page 101: 101s1.downloadmienphi.net/file/downloadfile2/180/1403331.doc · Web viewDuring normal office hours, University Maintenance will respond directly to a power failure. After hours, the

In light of your renowned research and your contributions in the area of mineral-fluid interfaces, we would like to formally invite you to serve as an Invited Speaker at the upcoming Interfacial Process Symposium, to be held at United Chemical Laboratories, in Lacy, N.M., on May 15.

We hope your lecture will include a discussion of trends and current developments in X-ray scattering technology. We will provide roundtrip airfare, hotel accommodation, roundtrip airport transportation, as well as an honorarium of US $800.

If you are willing to accept our offer, we would appreciate you contacting us before April 1. I also will need your up-to-date curriculum vitae, and a brief abstract of our talk for inclusion in the conference proceedings. Please let us know if you will need to use a computer, overhead projector, or other display technology. We will be happy to arrange for copying any handouts you may have.

Your contribution to this symposium would be of enormous benefit to all the participants.

Please do not hesitate to contact me if you have any concerns and/or suggestions.

Yours sincerely,

Bruce Nagy, Ph.D.

Director of Corporate Research

United Chemical Laboratories

196. What is learned about Dr. Green?(a) He occasionally works as an advisor for private corporations.(b) He has spent his entire career studying mineral - fluid interfaces.(c) He received his graduate and undergraduate degrees from the same university.(d) He began working at Staunton University shortly after obtaining his doctorate.

197. What is listed as one of Dr. Green's accomplishments?(a) Winning a national engineering award(b) Founding a scientific research institute(c) Developing an important research technique(d) Establishing a new department for research.

198. What is learned about the event that Dr. Green has been invited to attend?

(a) It is an annual conference.(b) It is being held in a university.(c) It is sponsored by a private corporation.(d) It is focussed exclusively on x-ray scattering technology.

199. Which of the following does Bruce Nagy NOT offer to do for Dr. Green?(a) Pay him for his appearance(b) Pick him up at the airport(c) Give him a tour of the area(d) Make copies of his handouts.

200. What does Bruce Nagy ask Dr. Green to do?(a) Provide a copy of his lecture notes(b) Respond before the beginning of April(c) Give a demonstration of his latest invention(d) Leave time open for questions from the audience.

Page 102: 101s1.downloadmienphi.net/file/downloadfile2/180/1403331.doc · Web viewDuring normal office hours, University Maintenance will respond directly to a power failure. After hours, the

ANSWER101. A 102. A 103. A 104. A 105. B 106. A 107. D 108. B 109. B 110. D111. D 112. C 113. A 114. C 115. B 116. A 117. C 118. A 119. B 120. B121. C 122. B 123. B 124. C 125. A 126. A 127. C 128. C 129. A 130. C131. C 132. D 133. D 134. A 135. D 136. A 137. C 138. B 139. C 140. A141. D 142. B 143. B 144. B 145. B 146. A 147. A 148. B 149. C 150. D151. D 152. C 153. C 154. C 155. B 156. D 157. D 158. A 159. C 160. D161. C 162. B 163. A 164. A 165. C 166. A 167. D 168. B 169. D 170. B171. A 172. A 173. A 174. D 175. C 176. D 177. D 178. B 179. C 180. B181. A 182. B 183. D 184. B 185. C 186. C 187. C 188. D 189. D 190. A191. D 192. B 193. D 194. B 195. B 196. C 197. C 198. C 199. C 200. B

Page 103: 101s1.downloadmienphi.net/file/downloadfile2/180/1403331.doc · Web viewDuring normal office hours, University Maintenance will respond directly to a power failure. After hours, the

101. Raymond and Howell proposed job cuts following a two-year decline in sales ______. (a) figures(b) actions(c) employers(d) information

102. In the past three years, our marketing manager ______ to Europe, Australia, Brazil and Thailand. (a) had gone(b) has been(c) gone(d) went

103. ______ moving to a northern climate, be sure to properly winterize your automobile. (a) gives(b) given(c) giving(d) is giving

104. An increase in a nation's rate of savings ultimately ______ to lower interest rates for business and consumer loans. (a) lead(b) leads(c) leader(d) leading

105. We need to get ______ paperwork from him before the end of the fiscal year. (a) him(b) himself(c) he(d) his106. Any information ______ about the revised holiday work schedule can be obtained from Ms. Zeldenrust in Human Resources. (a) volunteered(b) needed(c) donated(d) called

107. Ever since the drivers' strike started, citywide buses have been ______ running thirty minutes to one hour late. (a) finally(b) constantly(c) concurrently(d) very

108. More than 6.55 million workers are ______ in the manufacturing industry, according to government data. (a) employed(b) employer(c) employing(d) employment

109. Fair Oaks Transit trains were back on schedule Monday after a mudslide disrupted last Friday morning's ______. (a) section(b) commute(c) tracking(d) beginning

110. Sam's Steakhouse is ______ better than famous national chains with larger advertising budgets. (a) argue(b) arguing(c) argument(d) arguably

111. _____ Daily Dogs targets its advertising campaign to pet owners, it could be headed for Chapter 11 bankruptcy. (a) Until(b) Unless(c) Whereas(d) Except

112. Mocha Monkeys Coffee ______ very quickly because of its cheap prices and unique drinks. (a) expands(b) are expanding(c) is expanding(d) to expand

113. Notice: The meeting ______ for today has been postponed until Thursday at 4 P.M. (a) scheduled(b) happening(c) important(d) open

114. The new computer chips will ______ be delayed because the hurricane on the East Coast has disrupted air traffic. (a) probability(b) probably(c) probable

Page 104: 101s1.downloadmienphi.net/file/downloadfile2/180/1403331.doc · Web viewDuring normal office hours, University Maintenance will respond directly to a power failure. After hours, the

(d) probe

115. We would like to thank the Elog Foundation for its generous ______ to our charity fundraiser. (a) donor(b) donate(c) donated(d) donation

116. After carefully ______ the bid packages from each vendor, we have decided to award the contract to Thomas Cullen. (a) considered(b) considering(c) considerate(d) consideration

117. Economists predict a ______ in housing prices due to the recent reduction in unemployment. (a) development(b) growth(c) rise(d) lift

118. Changes in American eating ______ over the last twenty years are blamed for a variety of health problems. (a) meals(b) habits(c) dishes(d) restaurants

119. The Holden Complex ______ has vacancies despite its high rent, inconvenient location and large annual maintenance fee. (a) unusually(b) rarely(c) frequently(d) assuredly

120. Be informed that this medication is for temporary relief of symptoms, and is not intended ______ your annual immunization shot. (a) replaced(b) replace(c) to replace(d) replacing121. The Healthcare community was shocked ______ Ricardo Peter's resignation after only one year as president of Healthcorps.

(a) in(b) up(c) of(d) by

122. Alanis bought renter's insurance to ______ her personal belongings while living in an apartment. (a) protect(b) protector(c) protection(d) protective

123. The personnel department requires that all ______ be interviewed at least three times before a hiring decision is made. (a) applications(b) applicants(c) appliances(d) applicator

124. During office remodeling there will be ______ interruptions to ADSL, copier and fax machine service. (a) usual(b) often(c) frequent(d) little

125. Kindly ______ your colleagues to tip the waitress at least 15 percent. (a) remind(b) remember(c) order(d) command

126. It is probably safe to say that of all the board members, Mr. Cain has the most ______. (a) result(b) effect(c) influence(d) conclusion

127. Sales of new homes this month are down 2% from last month, ______ 6% from a year ago. (a) too(b) and(c) as well(d) along with

Page 105: 101s1.downloadmienphi.net/file/downloadfile2/180/1403331.doc · Web viewDuring normal office hours, University Maintenance will respond directly to a power failure. After hours, the

128. Since Dawson Creek Fashion was late getting its new styles on the market, there was not ______ opportunity to meet pre-Christmas sales projections. (a) many(b) little(c) much(d) big

129. The CEO of Star Records announced plans for aggressive expansion, saying the company will purchase ______ new properties in the next two years. (a) many(b) little(c) much(d) any

130. Human resources workers serve a vital ______ in screening qualified candidates for the company's managerial positions. (a) play(b) function(c) skill(d) piece

131. In a 20-minute address to computer science students, the CEO of Electro-lite discussed the availability of technology jobs in today's job ______. (a) market(b) maximum(c) document(d) department

132. The meteorologist on Channel 2 said that he was ______ certain that the cold front will result in snow tomorrow.(a) reasoning(b) reason(c) reasonable(d) reasonably

133. Ms. Benihana is knowledgeable in several disciplines, but especially biomedical engineering is her area of ______. (a) expert (b) expertise (c) experts (d) expertness134. The chairperson of Walter Bros. announced that this year's annual bonuses will be ______ to company employees on Friday. (a) awarded(b) transacted

(c) published(d) discarded

135. All international orders are ______ via Global Express Mail within five to ten business days. (a) spent(b) enrolled(c) collected(d) delivered

136. Most shareholders still fail ______ both the immediate impacts and future implications of corporate mergers and takeover bids. (a) understood(b) understanding(c) to understand(d) have understood

137. ______ all forms of advertising, research shows that direct mail reaches the greatest number of people at the lowest cost. (a) Through(b) In(c) Between(d) Of

138. In order to compete for a shrinking pool of qualified programmers, companies that are ______ conservative are now offering flextime and work-from-home options. (a) sometimes(b) always(c) infrequently(d) otherwise

139. The sales representative suggested an initial meeting to discuss the proposal as soon as the beta product is ready, ______ that might be. (a) no matter(b) which(c) whenever(d) what

140. Mr. Carter got ______ in trouble by failing to acknowledge his secretary's birthday. (a) he(b) himself(c) him(d) his

Page 106: 101s1.downloadmienphi.net/file/downloadfile2/180/1403331.doc · Web viewDuring normal office hours, University Maintenance will respond directly to a power failure. After hours, the

Questions 141-143 refer to the following letter.

Memory Map Gameby Riven Learning Co.

DescriptionFor 1-6 players, 5-11 years old

Children can ______ their observation and memorization skills with this puzzle that features a colorful picture map of a city.

141. (a) cause(b) study(c) practice(d) manufacture

To begin the game, children must place the 125 picture cards faced down. Each player then draws a picture card, turns it over, and waits for their turn to ______ the picture card to the map.

142. (a) add(b) addition (c) additive(d) additionally

______ a player cannot find where their picture card goes on the map, they must pick up another picture card.143. (a) If

(b) So(c) And(d) Because

The first player to get rid of all their picture cards wins the game.Price $21.99

Questions 144-146 refer to the following letter.

Dear Arcish Furniture,

I recently purchased a leather sofa from your outlet. The sales representative, Maureen, ______ me to purchase the 10-year warranty. I was told that if my furniture was accidentally damaged, even by a pet, the store would repair it at no cost to me.

144. (a) persuasion(b) persuasive(c) persuading(d) persuaded

Last week, my cat scratched the sofa, leaving three deep marks in the leather. The marks are clearly visible from a distance. I was surprised to learn from the manager that the 10-year ______ does not cover damage done by household pets when I was informed otherwise.

145. (a) stock(b) service(c) warranty(d) insurance

I have reviewed the fine print on the contract, and I can see that pet damage is not covered. However, this is not what Maureen told me when I bought the sofa. I feel I was ______ about the terms of the warranty. I accept responsibility

Page 107: 101s1.downloadmienphi.net/file/downloadfile2/180/1403331.doc · Web viewDuring normal office hours, University Maintenance will respond directly to a power failure. After hours, the

for not reading the full contract, but I trusted that I was being given all the facts.146. (a) misrepresented

(b) misspelled(c) misled(d) mistaken

I would like Arcish Furniture to fix my couch. I await your response.Sincerely,

Marcel Boule

Questions 147-149 refer to the following letter.Tampa Daily News - Available in Print and Online

The Tampa Daily News has two versions of its printed newspaper available online.One version ______ the printed edition, including the editorial and news pages.

147. (a) is the same(b) has the same(c) is the same as(d) as has the same

This is available at http://www.tampanews.com.

The other ______ is a special online edition that includes additional sports, lifestyle, and real estate pages.148. (a) version

(b) section (c) article (d) reporter

The electronic edition ______ has a complete classified section as well as up-to-date movie listings.

149. (a) too(b) and(c) plus(d) also

The electronic edition is available at http://www.tampanews.com/electronicnews. Subscriptions to the printed version of the newspaper are available at http://www.tampanews.com/subscribe.

Questions 150-152 refer to the following letter.

NOTICE TO ALL EMPLOYEES

The following changes will be effective as of Monday.

First of all, the dress code is changing slightly. You may still wear the ______ black pants and skirts as before along with the company shirt.

150. (a) pair(b) same(c) kind

Page 108: 101s1.downloadmienphi.net/file/downloadfile2/180/1403331.doc · Web viewDuring normal office hours, University Maintenance will respond directly to a power failure. After hours, the

(d) bothHowever, employees are no longer allowed to wear shoes with heels higher than two inches.

Secondly, we are implementing a new marketing promotion. From now on, when customers come through your register lineup, you should ______ ask them if they would like to provide an e-mail address.

151. (a) always(b) seldom(c) never(d) rarely

This will register them to receive notification of upcoming specials.

Finally, ______ we are switching to a new coupon-based system, we will be discontinuing the use of customer reward cards. Reassure customers that they can redeem points until the end of the year. Explain to customers that the coupon system will provide greater savings.

152. (a) although (b) unless (c) so (d) because

If a customer has any further questions, please refer him or her to the manager on duty.

Thank you for your hard work in recent weeks. Sincerely,Greg.

What's an Apprentice?

As everyone knows, opportunities in the skilled trades are set to explode in the next five to ten years.

Many skilled trades train workers through apprenticeships - new workers spend up to 80% of their days learning on the job, and only 20% in the classroom. This is accomplished through being paired with an expert in the trade who will guide the apprentice through their learning process.

Many countries have developed special ””pre-apprenticeship”” secondary school programs. If you are in high school, by participating in these programs you can learn job skills and earn credits towards your apprenticeship before you even graduate. Apprenticeships range from auto mechanics to cabinetmaking, construction, and pastry making

153. What is an apprentice according to this article?(a) Anyone who spends 20% of his or her time inside a classroom(b) An expert in a skilled trade who spends 80% of his or her time on the job(c) Any high school student who is learning job skills(d) A new worker matched with an expert to learn a skilled trade

154. How can high school students earn credits towards their apprenticeships?(a) By making matches with an expert in a skilled trade(b) By taking part in a pre-apprenticeship program(c) By interviewing auto mechanics or pastry makers(d) By spending only 20% of their time in class

From: Mary Unger [[email protected]]

Page 109: 101s1.downloadmienphi.net/file/downloadfile2/180/1403331.doc · Web viewDuring normal office hours, University Maintenance will respond directly to a power failure. After hours, the

To: Tina Barkley [[email protected]]Subject: PresentationSent: May 4, 10:20:07 A.M.

Dear Tina,

I wanted to get back to you about the presentation on Thursday. As of now, 23 people have responded. I expect a few more people to show up at the last minute.

We're still planning on holding the lecture in room 240B. I'll make sure we have at least 30 chairs set up. I was also planning to make photocopies of my presentation slides in case anyone would like a copy.

Sincerely,

Mary Unger

Outreach DirectorPayton Daniels Corporation

155. What does Tina Barkley find out regarding the presentation?(a) Her presentation will be videotaped.(b) A colleague will not be able to attend.(c) Her presentation is the first one scheduled.(d) Twenty-three people have said they will attend.

156. What does Mary Unger say she will do? (a) Set up the equipment by herself(b) Arrive 30 minutes early on Thursday(c) Make copies of her presentation materials(d) E-mail the presentation schedule later that day.

Item Number 21169Versatile Sports Coat

Made from lightweight wool, this versatile coat can be dressed up with a dress shirt and pair of slacks, or dressed down with a t-shirt and jeans. It is impeccably tailored with a perfect drape. Small shoulder pads, full polyester lining, and inside pocket, make this a durable and great looking coat. Other details include: two-button front, lined chest pocket, and reinforced hip pockets. Dry clean only.

Available in espresso, pine, and midnight

Price: $179, now on sale for $139!

Available in S, M, L, XL

157. Which feature does the Versatile Sports Coat have?(a) Wrist cuffs(b) A lined collar(c) Reinforced seams(d) An inside pockete.

158. Which color is the Versatile Sports Coat NOT available in?(a) Pine(b) Desert(c) Espresso(d) Midnight

Page 110: 101s1.downloadmienphi.net/file/downloadfile2/180/1403331.doc · Web viewDuring normal office hours, University Maintenance will respond directly to a power failure. After hours, the

159. How much does the Versatile Sports Coat now cost? (a) $114(b) $139(c) $179

(d) $211.

The Savvy Shoppers”Let us take the Worry out of Shopping!”

We all know how important dressing professionally can be.

You want to project a certain image with your clothes, and that can vary widely based on what kind of an event or meeting you need to attend. Approachable, tough, stylish… The options are endless, and it can be exhausting trying to find the right outfit for all of the jobs that you do!

Meet the Savvy Shoppers!Our team of professional personal shoppers can make your busy life a little easier by giving you back the time and energy you had previously been wasting on shopping for office clothes.

Trusted ProfessionalsWith extensive backgrounds in retail management, fashion design, and beauty consulting, our experienced team can put together a perfect wardrobe for any woman. Whether it's the right power suit for a big presentation, boardroom-appropriate skirts, or a high heel that won't leave you limping, the Savvy Shoppers can help!

The Savvy Shopping experienceIt begins with a private consultation in your home. Your shopper will look at your current wardrobe, talk to you about the work you do, and take your precise measurements. Once they begin shopping, Savvy Shoppers sends digital photos and price lists for items. You select what you want, and the items are delivered to your door with a free $20 tailoring coupon attached.

160. Who is this advertisement likely targeting?(a) Busy business men(b) Stay-at-home parents(c) Working professional women(d) Chief Executive Officers161. What experience does the Savvy Shoppers NOT have?(a) Managing in the retail sector(b) Clothing design(c) Hair styling(d) Consulting in the beauty sector.

162. What does Savvy Shoppers send to clients when shopping?(a) Prices and photographs of clothing items(b) Precise measurements and digital photos(c) Gift certificates for tailoring services(d) Information about the work they do163. How do clients get their clothes from the Savvy Shoppers? (a) They set up a consultation appointment.(b) They pick up the clothes from the office.(c) They receive them at their own home.(d) They meet their shopper at the store

Page 111: 101s1.downloadmienphi.net/file/downloadfile2/180/1403331.doc · Web viewDuring normal office hours, University Maintenance will respond directly to a power failure. After hours, the

The Bridgeport Revitalization Committee (BRC)13 Robin WayBridgeport, MA 02126

Kevin Taylor203 8th AvenueBridgeport, MA 02133

Dear Mr. Taylor,

I am excited to announce to our members that we have received a $750,000 grant from the Roger Coopers Foundation for our City Renovation Project.

The City Renovation Project focuses on the downtown area of Bridgeport, between First Avenue and Lincoln Way. We will begin by redesigning Baten Park and the pedestrian walkways to create a more inviting space for shoppers and workers on their lunch breaks.

This grant is the largest we have ever received, and we are excited about the work it will allow us to accomplish.

We look forward to improving the quality of life in our city through great design!

Denise LeBaron

Page 112: 101s1.downloadmienphi.net/file/downloadfile2/180/1403331.doc · Web viewDuring normal office hours, University Maintenance will respond directly to a power failure. After hours, the

164. What can be inferred about Mr. Taylor?(a) He is a member of the BRC.(b) He works for Denise LeBaron.(c) He lives between First Avenue and Lincoln Way.(d) He is on the board of the Roger Coopers Foundation.

165. What will the BRC use the grant money to do? (a) Build a museum(b) Redesign a park

(c) Expand the highway network(d) Create worker training programs

166. What is true about the grant?(a) It is for more than $1 million.(b) It is an extension of an existing grant.(c) It is awarded by the Bridgeport Revitalization Committee.(d) It is the largest the BRC has ever received.

Page 113: 101s1.downloadmienphi.net/file/downloadfile2/180/1403331.doc · Web viewDuring normal office hours, University Maintenance will respond directly to a power failure. After hours, the

Report: Kilmer Health Care Center in Top 10%

According to a report that was recently published in Consumer Quarterly, the Kilmer Health Care Center at University Village is ranked in the top 10 percent of all nursing homes in Ohio.

The Kilmer Health Care Center opened four years ago with 48 private rooms. Thirty-six of the rooms are for assisted living, and twelve are for constant care. The Kilmer Health Care Center offers residents an array of services from housekeeping and meal delivery, to transportation and medical services.

Consumer Quarterly looked at three main factors to come up with the nursing home rankings. The first was how the facility ranked in their state inspections. The second was the ratio of the number of care givers, (including nurses and nurse's aides), to the number of residents. Finally, they looked at the services the facility offers and compared those to the current and future needs of the residents who live there.

Consumer Quarterly hopes the report encourages those facilities ranked in the lower 10% to review and improve their operations.

167. How many rooms in Kilmer Health Care Center are intended for assisted living?(a) 12(b) 36(c) 48(d) 66.

168. What is learned about Kilmer Health Care Center? (a) It is near a hospital.(b) It opened four years ago.(c) It is run with state funds.(d) It has a highly-trained staff

169. What was NOT included in the ranking factors?

(a) The services offered(b) The state inspection numbers(c) The distance to nearby hospitals(d) The ratio of caregivers to residents.

170. What does Consumer Quarterly hope the report will do? (a) Increase state funding for all facilities(b) Encourage low-ranking facilities to improve(c) Increase awareness of the facilities that exist(d) Encourage more nurses to apply for jobs in the facilities

Page 114: 101s1.downloadmienphi.net/file/downloadfile2/180/1403331.doc · Web viewDuring normal office hours, University Maintenance will respond directly to a power failure. After hours, the

November 26th

ID: 733185Dylan BrownWorld Travel Inc.8840 Mayfair Ave.London, UK

Mr. Brown,

We were unfortunately unable to process your last monthly donation to SaveTheYouth because of an authorization issue with your company's credit card. Please confirm your credit card number in the space below and return this letter in the enclosed response envelope, or call us toll-free at 1-800-555-9090.

Confirmation of credit card information:

Card Number: _______________________________Expiration Date: ______________/________________

We would like to take this opportunity to thank you for supporting SaveTheYouth. By donating, your company is helping us pursue our vital mission of protecting children and young people the world over. The youth are our future, but millions suffer from malnutrition and are lacking in medical care, education, drinkable water and adequate public health while in the Western world we have an excess of material items and an embarrassment of riches. With your monthly donation, you are helping and protecting the lives of young people in over 150 countries.

We appreciate your company’s help. Together, we are working to create a better world.

With Regards,

Doreen EnsanProgram DirectorSaveTheYouth

171. What is Doreen's main purpose in writing this letter?(a) To note that Mr. Brown's last payment couldn't be processed(b) To re-confirm his credit card information for safety(c) To thank him for supporting SaveTheYouth(d) To explain SaveTheYouth's mission

172. How should Mr. Brown give SaveTheYouth his credit information?

(a) By fax(b) By e-mail(c) By returning the letter(d) By calling his bank.

173. What are the children SaveTheYouth helps NOT in need of?(a) Improved cleanliness(b) Food and water(c) Medical insurance(d) Doctors.

Page 115: 101s1.downloadmienphi.net/file/downloadfile2/180/1403331.doc · Web viewDuring normal office hours, University Maintenance will respond directly to a power failure. After hours, the

November 1st, 2006

STMP Capital158 Rue du Chateau des RentiersParis, FRA75014

To: Melanie Marie Bourgeois and Jessica Lee Lariviere:

We wish to remind you that you are presently bound to a lease from December 1st 2005 to November 30th 2006

We are informing you that for the period of prolongation of your lease, from December 1st, 2006 to November 30th 2007, our rent will be increased to $825 monthly. All other conditions of your lease will remain the same.

You are hereby notified that you have one month following receipt of the present notice to respond.

Sincerely,Sandro MilanoSTMP Capital

174. When is the current lease over? (a) November 1st, 2006(b) November 1st, 2007(c) December 1st, 2006(d) November 30th, 2006175. What will happen if the lease is prolonged?

(a) Rent will increase monthly.(b) Tenants will pay more per month.(c) New rental restrictions will apply.(d) Rent receipts will be required.

Lisa's Window Cleaning3362 River RoadCambridge, Ohio

Phone: (555) 782-9008Fax: (555) 789-0939E-mail: [email protected]

BILL TO: Claude EtheridgeSpark Network Systems89 Industrial Park WayCambridge, Ohio

Statement #: 1019976Date: January 23rd 2007Customer ID: V-941-A8

Date of Sale: November 5th 2006Invoice #: 2903Description: Bi-weekly window cleaning of high-rise building

Page 116: 101s1.downloadmienphi.net/file/downloadfile2/180/1403331.doc · Web viewDuring normal office hours, University Maintenance will respond directly to a power failure. After hours, the

Amount: $1,500.00 (x 6 cleanings)Payment: $0.00Balance: $4,500.00

Reminder: Please include the Statement Number on your check.Terms: Balance due in fourteen (14) days.

176. What is the purpose of this invoice?(a) To remind Lisa's Window Cleaning of money owed(b) To confirm payment of $4,500 to Lisa's Window Cleaning(c) To bill Spark Network Systems for services(d) To request payment for building construction.

177. What number should Claude Etheridge include on his check?(a) 1019976(b) 2903(c) V-941-A8(d) 01/23/07.

ATTENTION: ALL EMPLOYEES

As you may have heard, we are getting ready to implement a new computer system at the office. This may worry some of you, but the system is actually designed to make our lives easier. It will eliminate a lot of the repetitive paperwork that many of you dislike doing, and will give us a central system where all of our information can be stored.

In order to make sure everyone is comfortable with the new system, we will be holding several orientation sessions throughout next week. Please choose one that fits your schedule and sign up. You will learn the basics, get a guidebook for the system, and have an opportunity to get comfortable with the system by completing practice tasks with the help of the instructor.

The following sessions are available:

Tuesday, November 7th at 10:00 A.M.

Wednesday, November 8th at 11:00 A.M. and 3:00 P.M.

Friday November 10th at 9:30 A.M.

Please visit the bulletin board in the lunchroom and sign up for one. There will be snacks and refreshments for participants served after each session.

178. What is the purpose of the training sessions?

Page 117: 101s1.downloadmienphi.net/file/downloadfile2/180/1403331.doc · Web viewDuring normal office hours, University Maintenance will respond directly to a power failure. After hours, the

(a) To teach staff how to do repetitive work more quickly(b) To ensure the staff feels comfortable with the system(c) To introduce the basics of computer usage(d) To get together for snacks and refreshments179. What will participants in the orientation sessions NOT do?(a) Practice using the system(b) Learn the basics of the system

(c) Take a written test(d) Receive an instruction book.

180. What should an employee who has read the notice do next?(a) Send an e-mail indicating session preference(b) Show up for the Tuesday training session(c) Call the office manager to discuss scheduling(d) Go to the lunchroom and sign up for a session.

NOTICE TO ALL EMPLOYEES: 14th ANNUAL CONFERENCE OF IT PROFESSIONALS

CVD Inc. will be sending one employee from the IT Department to this conference, which will take place over a weekend in the spring at the Royal Embassy Hotel in Cambridge, Massachusetts. This years' conference topic is ”Information Technology and Human Resources: Bridging the Gap”. Interested employees should send an e-mail to Ashwin Jain at [email protected], stating why they believe they should be sent on the conference and how they would use what they learned there to better the company upon their return. If a suitable candidate is not found in the IT Department by January 15th, employees from Human Resources (HR) with strong computer skills will be invited to apply.

-------------------------------------------------------------------------------------------------------------------------

From: To: Subject: Conference

Ashwin,

I am writing to notify you of my interest in attending the 14th Annual Conference of IT Professionals. As soon as I saw the posting yesterday afternoon, I knew that I wanted to apply.

While I realize that I have not been with CVD Inc. for as long as many others in my department, I think my unique mix of skills and experience makes me the ideal person to attend. For the first half of my professional career, I worked as a Human Resources Manager for a software development firm. I am familiar with strategic management principles, HR policy administration, and organizational change and development trends. I frequently facilitated workshops and training sessions and am very comfortable presenting to a group.

When I decided five years ago to realize a dream and pursue an education in computer programming (which had previously been just a hobby), I knew that I wanted to work for a great company like CVD Inc. I love my job here but I do sometimes miss the human element of HR work. That's why I think that attending this conference would be an ideal situation for both myself and the company. Who better to absorb and circulate the information than someone who has had her feet in both worlds?

Thank you for considering me for this opportunity,

Page 118: 101s1.downloadmienphi.net/file/downloadfile2/180/1403331.doc · Web viewDuring normal office hours, University Maintenance will respond directly to a power failure. After hours, the

Louise CampineauIT DepartmentCVD Inc.

181. What is the main purpose of the conference?(a) Bringing human resources and technology closer together(b) How to apply for human resource jobs(c) Examining how IT and HR affect bridge-building(d) Examining how HR should look at bridge technology.182. Who would be most likely to attend the conference?(a) An employee from the Royal Embassy Hotel(b) A computer programmer from CVD Inc.(c) A new member of an IT department(d) An insurance policy administrator

183. What is Louise's experience with workshops? (a) She has attended many.(b) She has run many.(c) She has always wanted to attend one.

(d) She has difficulty participating in them.

184. What was Louise's dream?(a) To work for an excellent company such as CVD Inc.(b) To discover a new hobby unrelated to computers(c) To go to school to study computer programming(d) To attend the 14th Annual Conference of IT Professionals

185. What did Louise leave out of her response?(a) Why she believed she should be sent on the conference(b) The name of her previous employers and the school she attended(c) How she would use the information she learned at the conference(d) Critical information related to bridge-building technology

Page 119: 101s1.downloadmienphi.net/file/downloadfile2/180/1403331.doc · Web viewDuring normal office hours, University Maintenance will respond directly to a power failure. After hours, the

Looking for Author

I'm in the final proposal stages for a book aimed at female European athletes. I'm looking for an experienced author who is also an athlete.

The book is being pitched to a major publisher with a string of successful sports-themed books. This is a very interesting project, and I need to find someone who lives to write, is flexible, creative, energetic, and resides in the greater London area to be able to collaborate and have meetings.

I anticipate that the project will last for approximately three months. During this period, work will be very intense and will likely require a very large time commitment. The primary duties would be writing, researching, and fact-checking, but could be extended to other areas of the production process as the project evolves.

Please reply to Josh Abrams, 20 Stroud Green Road, London N43EH with credentials and athletic experience.-------------------------------------------------------------------------------------------------------------------------Josh Abrams20 Stroud Green RoadLondon, EnglandN4 3EH

Mr. Abrams,

I think that I'm the person you're looking for on this project. Why? I am an amateur athlete, and I can write in whatever tone, style, or voice is required. I belong to several networking groups of female amateur athletes - a great source for contacts - and I am very familiar with the unique challenges that women in the sports world face. I have also written extensively on the topics of the Olympics and particularly, lacrosse and football. I am a rugby player and track athlete.

After graduating from high school, I decided to pursue sports more actively. I spent two years playing semi-professional women's soccer and rugby, and placing very well in track events and marathons. I traveled extensively in Europe and became familiar with the rules and organizations there.

As a writer, I have published over 500 articles in national and community newspapers. My first book, ”Running for the Finish Line,” was a Canadian best-seller. I have edited three anthologies and contributed to several other books and magazines. I’m also a meticulous copy editor.

”Energetic” and ”creative” don't begin to describe me - if you decide that I'm right for your project, you'll see what I mean.

Regards,

Hayley Weeks8905 Kitsilano CourtVancouver, B.C.CanadaV68 3A9.

186. Why is Mr. Abrams looking for an author? (a) He has written a book and needs it to be edited.

Page 120: 101s1.downloadmienphi.net/file/downloadfile2/180/1403331.doc · Web viewDuring normal office hours, University Maintenance will respond directly to a power failure. After hours, the

(b) He has proposed a book and needs it to be written.(c) He wants media coverage for his book.(d) He wants a second opinion on his book.

187. What type of work will this project consist of?(a) Three months of hard work(b) A year with minimal time commitments(c) A year with an intense work schedule(d) Under three months of occasional work

188. Why does Hayley think she is ideal for the project? (a) She is an amateur female athlete.(b) She has traveled extensively in Europe.(c) She has written many articles and books.

(d) She is an able writer and a female athlete.

189. Why does Hayley mention her membership in athletic networking groups?(a) To show that she knows people to talk to about the book(b) To prove that she loves sports(c) To show how energetic and creative she is(d) To identify who she will sell the book to.

190. Why won't Hayley be selected for this job?(a) She copies other writers.(b) She doesn't have references.(c) She is a female.(d) She resides in Canada.

The Top Five longest-running outdoor theatrical productions

Production Name Opening Date Location---------------------------------------------------1. Abigail 1928 Landsdale, CA2. The New Dream 1934 Los Angeles, CA3. Born to Shine 1947 Saratoga, NY4. Now until Forever 1951 Peterson, KY 5. Vern in the Hills 1956 Bradley, MO ------------------------------------------------------------------------------------------------------------------------------”Vern in the Hills” 50th Anniversary Celebration, Bradley, MO

”Vern in the Hills” is celebrating its 50th year in production. A light-hearted production staged by the City Theater in Bradley, MO, ”Vern in the Hills” has been performed at the Thompson Outdoor Theater each summer since it first opened on May 28, 1956. Few theaters in such small cities are able to sustain a production over a course of so many years.

”Somehow we've become a classic, and part of the fabric of the community,” says artistic director David Frey, ”Going to ”Vern in the Hills” on a summer evening is just part of life here.” Mr. Frey is only the third director the production has seen in its fifty-year history. A fact that, Mr. Frey says contributes to the play's success.

”Vern in the Hills” does seem to be part of the fabric of the community. We loved the production. We observed several families with three generations having picnics and watching the play. It was clear that most people had seen it before. If you ever have the chance to see ”Vern in the Hills,” you will quickly understand how it became a classic.

191. Where is “The New Dream” playing?(a) Peterson, KY(b) Saratoga, NY(c) Landsdale, CA(d) LosAngeles, CA

192. Which production opened for the first time in 1947?(a) Abigail(b) The New Dream(c) Born to Shine(d) Now until Forever

Page 121: 101s1.downloadmienphi.net/file/downloadfile2/180/1403331.doc · Web viewDuring normal office hours, University Maintenance will respond directly to a power failure. After hours, the

193. What is learned about “Vern in the Hills”? (a) It is based on a novel.(b) It is shown year-round.(c) It has been in production for 50 years.(d) It has sold more tickets than any other show.

194. What is learned about David Frey?(a) He wrote “Vern in the Hills.”(b) He is the director of “Vern in the Hills.”

(c) He plays the lead role in “Vern in the Hills.”(d) He has seen every performance of “Vern in the Hills.”

195. In the article, the word “course” in paragraph 1, line 4, is closest in meaning to:(a) class(b) period(c) program(d) direction.

Job Opening

Position: Library ClerkLocation: District Library

We are looking for an experienced library clerk to perform various duties as assigned by the head librarian, including but not limited to processing of library materials and assisting with circulation duties. The ideal candidate has at least 1 year of experience (co-op work placements counted), can demonstrate superior customer-service skills, and is familiar with various library shelving systems. Relevant educational background is an asset.

Key Competencies:-- Knowledge of library practices, policies and procedures-- Computer skills: Word processing, Internet usage, computerized filing-- Effective communication with public and co-workers required

Please apply online at www.yourdistrictlibrary.com. No letters or phone calls please. Only candidates selected for an interview will be contacted.------------------------------------------------------------------------------------------------------------------------------ONLINE JOB APPLICATION FORM

Name: David Klein

Address: 280 Wellington Street

Phone: (555) 738-8876

EXPERIENCE:

Winter 2003-Present:

Shelving Technician, County Library

-- Experienced with all systems related to shelving: Dewey Decimal, shelving by genre, format, etc.

-- Dedicated to ensuring that library patrons find what they need and leave the library happy

Page 122: 101s1.downloadmienphi.net/file/downloadfile2/180/1403331.doc · Web viewDuring normal office hours, University Maintenance will respond directly to a power failure. After hours, the

Summer 2002, 2003:

Programming Assistant, City Reading Club

-- Responsible for planning and implementing various summer reading programs for children

-- Role included delivering story-times and liaising with parents

Fall 2002-Fall 2003:

Product Demonstrator, In-Store Demonstrations Inc.

-- Highly independent position, responsible for setting up display and giving out in-store samples

-- Made detailed reports of customer reactions and monitored stock levels.

EDUCATION:2006: McGraw University

Student in joint History/Library Sciences program

Awarded an entrance scholarship

2004: Maple County High School

Honor roll student

Editor of school newspaper.

196. Who would be in charge of the person hired for this job?(a) The shelving technician(b) The customer service representative(c) The head librarian(d) The co-op administrator.

197. What is required for this position?(a) Math skills(b) Social skills(c) Computer programming(d) A library science course

198. How does David's experience with “Dewey Decimal” relate to the job posting? (a) It demonstrates his superior customer service.(b) It shows his understanding of the Internet.

(c) It demonstrates his ability to hand out in-store samples.(d) It proves his familiarity with library shelving systems.

199. How is David's education an asset to his application?(a) It relates to working in a library.(b) It shows that he is a good reader.(c) It demonstrates his independence.(d) It proves his editing skills

200. What did David do in the summer of 2003?(a) He applied for job position # 1009-36.(b) He worked at more than one job.(c) He finished high school.(d) He took a summer reading program

ANSWER

Page 123: 101s1.downloadmienphi.net/file/downloadfile2/180/1403331.doc · Web viewDuring normal office hours, University Maintenance will respond directly to a power failure. After hours, the

101. A 102. B 103. A 104. B 105. D 106. B 107. B 108. A 109. B 110. D

111. B 112. C 113. A 114. B 115. D 116. B 117. C 118. B 119. B 120. C

121. D 122. A 123. B 124. C 125. A 126. C 127. B 128. C 129. A 130. B

131. A 132. D 133. B 134. A 135. D 136. C 137. D 138. D 139. C 140. B

141. C 142. A 143. A 144. D 145. C 146. C 147. C 148. A 149. D 150. B

151. A 152. D 153. D 154. B 155. D 156. C 157. D 158. B 159. B 160. C

161. C 162. A 163. C 164. A 165. B 166. D 167. B 168. B 169. C 170. B

171. A 172. C 173. C 174. D 175. B 176. C 177. A 178. B 179. C 180. D

181. A 182. B 183. B 184. C 185. C 186. B 187. A 188. D 189. A 190. D

191. D 192. C 193. C 194. B 195. B 196. C 197. B 198. D 199. A 200. B

Page 124: 101s1.downloadmienphi.net/file/downloadfile2/180/1403331.doc · Web viewDuring normal office hours, University Maintenance will respond directly to a power failure. After hours, the

101. Dr. Hooper at Michigan United Hospital is the world's ______ authority on knee replacement surgeries. (a) led(b) leader(c) leading(d) leaders

102. With ski season fast ______, investors are wondering whether the ski lodge will be completed by the first snow fall. (a) selling(b) snowing(c) completing(d) approaching

103. Althea Corporation ______ by many analysts to be the most innovative software company in the industry. (a) considerably(b) consideration(c) is considered(d) was considering

104. It is still unclear ______ the Mayor will accept the recommendations of the City Council. (a) yet(b) about(c) before (d) whether105. Salpert Corporation's stock ______ by 8% during the first quarter of the year. (a) rose(b) rise(c) risen(d) rising

106. A generation ago, no one could have predicted that computers and the Internet would be as ______ as they are today. (a) recent(b) continual(c) prevalent(d) communicated

107. Albertson and Kelman have decided to open a separate ______ for research and development. (a) office(b) officer(c) official(d) officially

108. Home sizes at Timberland Ranch ______ from 1,200 to 8,000 square feet. (a) cost(b) count(c) range(d) weigh

109. An overpass on Interstate 38 in Lakeview County ______ yesterday, due to high flood waters. (a) collapse(b) collapsed(c) collapses(d) collapsing

110. We anticipate ______ the first phase of the project will take approximately two weeks to complete. (a) that(b) which(c) there(d) those

111. The total cost to renovate the building was $13.75 million, ______ double the original estimate. (a) almost(b) mostly(c) the most(d) most all

112. We would like to thank the Elog Foundation for their generous ______ to our charity fundraiser. (a) donor(b) donate(c) donated(d) donation

113. The retail industry spends ______ $28 billion on IT and related systems annually. (a) nearly(b) yearly(c) monthly(d) continually

114. My ______ with Mr. Frantz was mostly focused on issues related to integrating the staff after the proposed merger. (a) discuss(b) discussed(c) discusses(d) discussion

Page 125: 101s1.downloadmienphi.net/file/downloadfile2/180/1403331.doc · Web viewDuring normal office hours, University Maintenance will respond directly to a power failure. After hours, the

115. The schedule, which you will find attached, ______ the specific delivery dates we require. (a) ships(b) places(c) details(d) succeeds

116. Senior analyst John Pierpoint expects SunDigit to post ______ of $4 billion for the year. (a) a profit(b) to profit(c) profitable(d) profitably

117. ______ the highway was built, the only practical way to brings goods into the town of Tilman was by rail. (a) Then(b) With(c) Since(d) Until118. Brantel, Inc. has reportedly ______ a 500-acre property in Berlin, Florida, where it plans to build a manufacturing plan. (a) purchase(b) purchased(c) purchases(d) purchasing

119. Director Gary Robertson's latest film, “Frontiers,” centers on the ______ of a young man who traveled to the American West in the 1850s, seeking fortune and fame. (a) book(b) novel(c) movie(d) story

120. The bird feeder ______ to keep squirrels from getting to the birdseed. (a) has design(b) is designed(c) was a designer(d) has been designing

121. ______ the Radio Communications Directory, there are 16,376 licensed radio stations currently broadcasting. (a) Due to(b) Taken by(c) Required by

(d) According to

122. On April 3rd, we will hold a meeting with the ______ of our business units. (a) manage(b) managed(c) managers(d) managing

123. The average ______ for a 2-bedroom apartment in Boulder, Colorado is $900 a month. (a) pay(b) size(c) rent(d) floor

124. Dr. Corley holds a Ph.D. in anthropology, and ______ a book about the Nile delta last October. (a) publish(b) published(c) publisher(d) publishing

125. The Metro Tech program offers a 6-month course leading to an ACAM ______ in Basic Automobile Maintenance upon successful completion. (a) teacher(b) training(c) classroom(d) certificate

126. The European Commerce Authority is seeking ______ imports of foreign-made textiles and garments. (a) limits(b) limiting(c) to limit(d) limitation

127. Be sure to use the Standard Operations checklist ______ time you prepare boxes for shipment. (a) few(b) each(c) many(d) none

128. The Museum of Natural History and Industry in Tanner, OH receives more than 40,000 ______ each month. (a) visit

Page 126: 101s1.downloadmienphi.net/file/downloadfile2/180/1403331.doc · Web viewDuring normal office hours, University Maintenance will respond directly to a power failure. After hours, the

(b) visited(c) visitors(d) visiting

129. Portable, affordable, and stylish: personal listening devices have never been a more popular ______. (a) area(b) gift(c) show(d) music

130. Johnson's paper ______ three processes for generating hydrogen in fuel cells. (a) compares(b) comparisons(c) is compared(d) was a comparison131. LinTex CEO Jason Reeves ______ abruptly this afternoon, and the company did not offer an explanation for his sudden departure. (a) promoted(b) resigned(c) descended(d) accounted

132. The agenda for Thursday's meeting ______ include new product development.(a) not(b) was not(c) has not(d) does not

133. The e-mail server will be down ______ between 9:30 and 10:00 p.m. on Wednesday so that we can upgrade the software. (a) permanently(b) immediately(c) temporarily(d) successfully

134. Unfortunately, the New York office has failed ______ its sales goals in each of the last three quarters. (a) achieves(b) to achieve(c) achievement(d) the achievements

135. While he does not say so directly, Dr. Taylor ______ that consumers assign value to products based on their perceptions of quality. (a) implies(b) examines(c) characterizes(d) distinguishes

136. About 40% of patients ______ come to the Dearfield Emergency Room have only minor injuries. (a) who(b) whom(c) whose(d) whose are

137. Please contact Jen Davis in the HR Department if you have any questions regarding the new overtime and vacation ______. (a) methods(b) policies(c) reactions(d) behaviors

138. Many ______ are nervous about the longterm financial health of the company, especially after last week's poor earnings report. (a) investors(b) investments(c) of investors(d) of the investments

139. Most of the country's ______ engages in agricultural activities of one sort or another. (a) land(b) nation(c) garden(d) population

140. ______ CDF Inc. stood to make a larger profit by selling its aging Texas plant to rival Albacore Inc., it decided instead to sell for less to the relative new-comer Telebrite, to help cement a strategic partnership aimed at checking Albacore's dominance. (a) Because(b) Although(c) As long as(d) Rather than

Page 127: 101s1.downloadmienphi.net/file/downloadfile2/180/1403331.doc · Web viewDuring normal office hours, University Maintenance will respond directly to a power failure. After hours, the

Questions 141-143 refer to the following letter.

New CEO for TNR's European Division

Automaker TNR announced yesterday that it has ______ Pierre Aldridge, the current CEO of its IntelliCar division, to the new position of CEO of the entire European division, effective immediately.

141. (a) appoint(b) appoints(c) appointed(d) appointment

Mr. Aldridge, 48 years old, will be responsible for overseeing the company's operations throughout Europe, and also for looking into potential relationships and expansion opportunities in Asia.

Although Mr. Aldridge will begin working in his new position immediately, he will continue acting as CEO of the IntelliCar division until the release of the IntelliCar 3.2, and until a ______ is hired.

142. (a) replace(b) replaced(c) replacing(d) replacement

Mr. Aldridge is ______ for Tim Dressler, who was reassigned to the North American office.143. (a) going away

(b) working at(c) taking over(d) looking into

Questions 144-146 refer to the following letter.

MEMORANDUM

To: Roger KhanFrom: Lisa McDermottDate: May 1st, 20—Subject: Lanyard Bay down zoning-------------------------------------------

Dear Roger,

______ is a copy of the staff report that will be sent to the City Planning Association.144. (a) Imported

(b) Invoiced (c) Attached (d) Suspended

Please note the ______ we have made regarding the down zoning of the property at Lanyard Bay. Our case will be heard at the regular meeting of the City Planning Association, on June 9th. The meeting will start at 3:00 p.m. in room 329-C at the City Hall.

Page 128: 101s1.downloadmienphi.net/file/downloadfile2/180/1403331.doc · Web viewDuring normal office hours, University Maintenance will respond directly to a power failure. After hours, the

145. (a) members (b) buildings (c) suggestions (d) neighborhoods

I think it is very important that we both ______ in person. I think our presence will help our request to be taken seriously. This will be one of many items on their agenda.

146. (a) attend (b) attended (c) attending (d) attendance

Please let me know if you will be able to go with me.

Thanks,

Lisa

Questions 147-149 refer to the following letter.From: Nathaniel NiTo: Gitah DavisSubject: Overtime workDate: August 10

Gitah,

I've attached the overtime work that I spoke with you about earlier. As we discussed before, I need you to ______ all of the information on a spreadsheet and write a brief summary of the figures. The work must be completed by Friday, August 14.

147. (a) organize (b) organizing (c) organization (d) organizationally

Once you're finished with the documents, please ______ them back to me with an indication of how long it took you to complete the work.

148. (a) sent (b) send (c) sends (d) sending

I'll fill out the necessary forms to secure your overtime ______.

149. (a) option (b) deposit (c) caution (d) payment

If you have any questions, do not hesitate to contact me at extension 3087.

Page 129: 101s1.downloadmienphi.net/file/downloadfile2/180/1403331.doc · Web viewDuring normal office hours, University Maintenance will respond directly to a power failure. After hours, the

Thank you,

Nathaniel Ni.

Questions 150-152 refer to the following letter.

Vision Cable249 W. Tenth StreetTaylor, ID 90834

Service Address:

Mary Ann Payton310 Main StreetTaylor, ID 90842

By ______ below, I certify that I am at least 18 years old and am the owner or tenant of the residence at the above address.

150. (a) aging (b) living (c) fixing (d) signing

I also certify that the repair or other work order has been completed ______. If this work order relates to initial installation of services,

151. (a) satisfied(b) satisfying (c) satisfactory (d) satisfactorily

I accept responsibility for all property loaned ______ by Vision Cable. I certify that I have received a welcome packet that contains my subscriber agreement as well as information about my services.

152. (a) to me (b) in them (c) for you

(d) about us

Customer Name: Mary Ann Payton Date: March 3, 20—

Customer Signature: Mary Ann Payton Date: March 3, 20--

On Wednesday, Dar Pfeiffer, one of the largest brokerage firms in the world, reported a second-quarter profit that was 53% larger than expected. The second-quarter profit was attributed to a one-time deal with money manager Tapcourt.

Shareholders saw this profit in the form of a $2.46 increase in share prices. The same period last year showed a

Page 130: 101s1.downloadmienphi.net/file/downloadfile2/180/1403331.doc · Web viewDuring normal office hours, University Maintenance will respond directly to a power failure. After hours, the

$1.07 increase a share. Excluding the deal with Tapcourt, Dar Pfeiffer would have reported a profit 23% larger than expected. Either way, Dar Pfeiffer topped market predictions for performance by at least $1.58 a share.

153. What is Dar Pfeiffer?(a) A brokerage firm(b) An accounting firm(c) A market analyst firm(d) An investment banking firm

154. What did Dar Pfeiffer report?(a) That it plans to offer new services(b) That it had purchased a smaller firm(c) That it plans to increase its prices(d) That it had made more profit than expected

21st Annual Spears Foundation Conference on Health

Conference Purpose

The 21st Annual Spears Foundation Conference on Health provides professional education and networking for doctors, dentists, nurses, social workers, health care instructors, students, and anyone else involved or interested in the health care profession. The conference is funded by grants to the Spears Foundation from the Department of Health, the Children's Health Network, the Family Health Association, the Organization for World Health and Nutrition, and with support from the Sebastian L. Turner Foundation and the Phillippe Endowment.

Conference Goals

The focus of this year's conference will be promoting proper nutrition among children. Childhood nutrition is a significant health care issue in light of the increased prevalence of childhood illnesses related to an improper diet. In addition to posing immediate health consequences, a poor diet can contribute to poor health throughout an individual's life and eventually lead to conditions like obesity and heart disease.

The goal of the 21st Annual Spears Foundation Conference on Health is to foster the awareness and early prevention of improper childhood nutrition within the field of healthcare.

The conference aims to:

■ broaden the discussion on nutrition to include health impacts at local, state, national and international levels among individuals under the age of 12■ describe the latest physical and mental health findings related to improper childhood nutrition■ highlight innovative research, practices, and programs that seek to improve the nutrition of children■ emphasize prevention and intervention strategies appropriate for children of different cultures, physical abilities, geographic settings, and communities

Conference Dates

Friday March 23 to Sunday March 25

Contact Information

Spears Foundation Conference on HealthAttn: Conference ManagerLawrence P. Martin, Jr.

Page 131: 101s1.downloadmienphi.net/file/downloadfile2/180/1403331.doc · Web viewDuring normal office hours, University Maintenance will respond directly to a power failure. After hours, the

8 Newbury StreetBoston, Massachusetts 02115Phone: 617-262-0044Fax: 617-262-0008E-mail: [email protected].

155. The word “promoting” in paragraph 2, line 1, is closest in meaning to:(a) benefiting(b) encouraging(c) advertising(d) contributing

156. Who is the conference for? (a) People in the retail industry(b) People in the banking industry(c) People in the education industry(d) People in the healthcare industry

157. What is the focus of the conference?(a) Safety on the job(b) Employee education(c) Childhood nutrition

(d) Employee health benefits

158. Who is Mr. Martin?(a) The manager of the conference(b) A member of the Children's Network(c) The president of the Spears Foundation(d) A major donor to the Spears Foundation

159. On what day will conference activities NOT take place? (a) March 22(b) March 23(c) March 24(d) March 25

Best Cities For Tech Workers

According to DataQuest's latest figures, the best jobs in technology are located in eight metropolitan areas: New York, San Francisco, Atlanta, Seattle, Portland, San Diego, Houston, and Philadelphia.

DataQuest first began collecting this data five years ago. The list of top-tech cities has changed since then, but demand for tech workers has not - it has risen by twenty-five percent.

If you are pursuing a career in technology, but do not live in or near one of the top-tech cities, there is still plenty of opportunity.

Matthew Martinez, CEO of NetStaff, a high-tech staffing firm with offices around the country, said that the companies his company recruits for are always looking for more employees with tech expertise. According to Martinez, ””For most areas it is currently an 'employees market,' meaning that many employees in technology are able to pick and choose their jobs, ask for better benefits, and arrange flexible work schedules.

160. According to the passage, which of the following is NOT one of the top-tech cities?(a) Atlanta(b) Houston(c) Seattle(d) Chicago

161. What is said about demand for tech workers?(a) It has continued to decline.

(b) It has been concentrated in 8 major areas.(c) It has risen by 25 percent over the past five years.(d) It has influenced the development of the toptech cities.

162. What does Matthew Martinez imply about tech workers in the current job market?(a) They need to be flexible in their requirements.(b) They should not expect conditions to change soon.

Page 132: 101s1.downloadmienphi.net/file/downloadfile2/180/1403331.doc · Web viewDuring normal office hours, University Maintenance will respond directly to a power failure. After hours, the

(c) They may be able to negotiate better work situations.

(d) They can get the best jobs through recruiting and staffing firms

Receipt

November 9, 20--Tony Parker

http://www.kramerstores.comKramer Retail StoresAmerton Mall 41 West Amerton ParkwayRoss Park, KSwww.kramerstores.com/amertonmall

Book Retail ReceiptItem: Future Oriented Investment StrategiesRF#: 2011CL0956Total: $21.98Payment Type: Credit CardAuthorization Code: XXXXXXXXXXX028Confirmation Number: 21PLTR4

All returns must be made within 14 days. Receipt is required for refund. Without receipt, only store credit can be given.

Page 133: 101s1.downloadmienphi.net/file/downloadfile2/180/1403331.doc · Web viewDuring normal office hours, University Maintenance will respond directly to a power failure. After hours, the

163. How much did Tony Parker pay for his purchase? (a) $20.11(b) $21.45(c) $21.98(d) $28.04

164. What did Tony Parker purchase?(a) A CD(b) Food(c) A book

(d) Clothing.

165. How long does Tony Parker have to return his purchase? (a) 7 days(b) 14 days(c) 28 days(d) 30 days

Page 134: 101s1.downloadmienphi.net/file/downloadfile2/180/1403331.doc · Web viewDuring normal office hours, University Maintenance will respond directly to a power failure. After hours, the

The Bridgeport Revitalization Committee(BRC)13 Robin WayBridgeport, MA 02126

Kevin Taylor203 8th AvenueBridgeport, MA 02133

Dear Mr. Taylor,

I am excited to announce to our members that we have received a $750,000 grant from the Roger Coopers Foundation for our City Renovation Project.

The City Renovation Project focuses on the downtown area of Bridgeport, between First Avenue and Lincoln Way. We will begin by redesigning Baten Park and the pedestrian walkways to create a more inviting space for shoppers and workers on their lunch breaks.

This grant is the largest we have ever received, and we are excited about the work it will allow us to accomplish.

We look forward to improving the quality of life in our city through great design!

Denise LeBaron.

166. What can be inferred about Mr. Taylor?(a) He is a member of the BRC.(b) He works for Denise LeBaron.(c) He lives between First Avenue and Lincoln Way.(d) He is on the board of the Roger Coopers Foundation.

167. What will the BRC use the grant money to do?(a) Build a museum

(b) Redesign a park(c) Expand the highway network(d) Create worker training programs.

168. What is true about the grant? (a) It is for more than $1 million.(b) It is an extension of an existing grant.(c) It is awarded by the Bridgeport Revitalization Committee.(d) It is the largest the BRC has ever received.

Page 135: 101s1.downloadmienphi.net/file/downloadfile2/180/1403331.doc · Web viewDuring normal office hours, University Maintenance will respond directly to a power failure. After hours, the

The Stemeresch Water Treatment Plant

It is the goal of the Stemeresch Water Treatment Plant to keep the Stemeresch River, the streams that flow into it, and our entire water supply as clean and healthy as possible. We want the people who live in San Realto Township and the visitors to this great city to enjoy all we have to offer, from parks and shopping, to golf courses and swimming pools. None of this would be possible without our water supply.

Each month, the Stemeresch Water Treatment Plant reclaims about a quarter of the water that passes through the plant(about 1.0 million gallons) and uses it to irrigate landscaping for the city's parks and recreation areas. Additionally, all so-called biosolids that are removed from the wastewater process are dried and tested to make sure they meet Class A requirements, which means the solids can be recycled as fertilizer.

The Mission of the Water Pollution Administration is to efficiently, effectively, and professionally:

* collect and treat wastewater* recycle biosolids responsibly* protect the health and safety of the local community* protect the water quality of the Stemeresch River and streams* supplement the water supply with recycled water for park irrigation* promote discussion about water treatment and conservation with the public.

169. Which of the following is a stated goal of the Stemeresch Water Treatment Plant?(a) To recycle biosolids(b) To educate the public(c) To create job opportunities(d) To test the township water supply.

170. Where is the Stemeresch Water Treatment Plant most likely located? (a) Near a pond(b) Near a lake(c) Near the sea(d) Near a river

171. How many millions of gallons of water does the plant claim to recycle each month?(a) 1.0(b) 2.0(c) 4.0(d) 5.2

172. How is the reclaimed water from the Stemeresch Water Treatment Plant used?(a) To fill city-owned swimming pools(b) To irrigate local farms and orchards(c) To water parks and other landscaping(d) To supplement the city's drinking water supply.

Page 136: 101s1.downloadmienphi.net/file/downloadfile2/180/1403331.doc · Web viewDuring normal office hours, University Maintenance will respond directly to a power failure. After hours, the

Jared O'Connor1205 Coconut RoadFt. Lauderdale, FL 33028

April 14 20—

Mr. Bradley ChangDirector of Human ResourcesUtmost.com9225 Perry Avenue, Suite 302Ft. Lauderdale, FL 33028

Dear Mr. Chang,

I am writing to thank you for the time you took to discuss the technical support manager position at Utmost.com with me. I really enjoyed learning more about the position and the company's operations.

I also enjoyed the opportunity to meet and observe your employees; I was impressed by the excitement and enthusiasm they had for their jobs, and for the company as a whole. It is no wonder that Utmost.com has been so successful.

I would like to confirm my strong interest in the position. I feel I would be a good fit. I have a strong work ethic, I enjoy working in teams, and I am a creative problem-solver.

I look forward to hearing from you about the position. If you have any additional questions please do not hesitate to call me.

Sincerely,

Jared O'Connor.

173. For what job is Jared O'Connor being considered?(a) Technical support manager(b) Director of human resources(c) Customer service team member(d) Digital information specialist174. According to the letter, what probably happened during Jared O'Connor's interview? (a) He attended a meeting with a client.(b) He was asked to give a presentation.(c) He observed employees doing their jobs.(d) He took a test to demonstrate his technical skills175. What does Jared O'Connor imply about the Utmost.com employees?

(a) They are highly skilled.(b) They do not require much supervision.(c) They do not all work in the same office.(d) They contribute to the company's success.

176. Which of the following is NOT a characteristic that Jared O'Connor claims to have?(a) A strong work ethic(b) Problem-solving skills(c) Ability to work on teams(d) Excellent communication skills

The US Hardware Manufacturing Sector

Page 137: 101s1.downloadmienphi.net/file/downloadfile2/180/1403331.doc · Web viewDuring normal office hours, University Maintenance will respond directly to a power failure. After hours, the

The hardware manufacturing sector in the United States generated $3.08 billion in sales last year. The industry currently employs approximately 18,000 people, nearly 15,000 of whom are engaged in production.

During the 1980s and 1990s, large corporations began to acquire smaller firms. For example, Twin Falls Hardware, long the number-two company in the industry in terms of sales, acquired eight companies between 1986 and 1993, further reinforcing its position.

In 1995, two large hardware manufacturers, Gator Inc. and Kairn Corporation, merged to form Kairn Holdings, an arrangement profitable for both companies. Kairn Holdings was the leading hardware manufacturer in the United States in 1998, and continues to be a market leader.

The majority of companies in the hardware manufacturing sector are privately owned. The industry is concentrated in the Southeastern United States, with over 40 percent of shipments originating there.

177. What is learned about the US hardware manufacturing sector?(a) It employs about 18,000 people.(b) It is entering a period of change.(c) It reached its peak during the 1990s.(d) It is a combination of three different industries.

178. According to the passage, what happened in the US hardware manufacturing sector during the 1980s and 1990s?(a) Sales topped $3 billion annually.(b) Large corporations acquired smaller ones.(c) Privately-owned firms became more powerful.(d) Quality control procedures led to increased efficiency.

179. What is learned about Twin Falls Hardware?

(a) It plans to double its employees.(b) It has expanded the types of items it produces.(c) It has the industry's second-largest sales volume.(d) It recently purchased a new manufacturing facility

180. What is learned about Kairn Holdings?(a) It was formed from a merger.(b) It was a market leader in 1995.(c) It was the most profitable manufacturer in 1999.(d) It was previously known as the Gator Corporation

181. The word “concentrated” in paragraph 4, line 2, is closest in meaning to:(a) merged(b) thought(c) growing(d) grouped.

Street Maintenance, McFarlen, MN

To report problems with street maintenance please call the public works department at 341-472-3680 during our normal office hours: 7:00 a.m. - 4:00 p.m. After hours, please leave a message on the Public Works Hotline at 341-472-3600, or e-mail the public works department at [email protected]. In case of an emergency, please contact the McFarlen Police Department at 341-472-3000.

If you notice any of the following, please report the problem:

* Potholes, street asphalt, guardrails, or reflective markers on City-maintained streets need to be repaired or replaced. This excludes areas where road maintenance is in progress.

* Street traffic signs need to be repaired or replaced

Page 138: 101s1.downloadmienphi.net/file/downloadfile2/180/1403331.doc · Web viewDuring normal office hours, University Maintenance will respond directly to a power failure. After hours, the

* Pavement marking needs repair (e.g., yellow lines between lanes)

* Parking meters are knocked down or not working

* Streets are flooded due to storms

* Spill response is needed

* Graffiti is found

-------------------------------------------------------------------------------------------------------------------------

To: From: Becky Price

Date: January 23, 20—

Subject: Stop sign down

------------------------------------------------------

To Whom It May Concern:

I am writing to let you know that there is a stop sign down, at the corner of Cherry Street and Tenth Avenue. It must have fallen down during the storm last night.

I'm concerned because this is a residential area that borders the high school property. The corner gets a lot of traffic, especially when school is in session, and I believe it could be dangerous without a stop sign. Normally, Cherry and Tenth is a four-way-stop intersection. As you can no doubt imagine, without the stop sign, the possibility for accidents is high.

If you would like to contact me, please feel free to do so:

742-488-3126.Thanks,Becky Price.

182. Which number should be called to report a nonemergency situation outside of normal work hours?(a) 341-472-3000(b) 341-472-3600(c) 341-472-3680(d) 742-488-3126.

183. According to Becky Price, what happened the previous night?(a) There was a storm.(b) A meeting was held.(c) There was a car accident.

Page 139: 101s1.downloadmienphi.net/file/downloadfile2/180/1403331.doc · Web viewDuring normal office hours, University Maintenance will respond directly to a power failure. After hours, the

(d) A road maintenance crew arrived.

184. What does Becky Price report? (a) A pothole(b) A flooded street(c) A fallen stop sign(d) A damaged guardrail.

185. What is learned about the corner of Cherry Street and Tenth Avenue?(a) It is near a high school.(b) It floods often due to storms.(c) It does not have reflective markers.(d) It has been the scene of several accidents.

-------------------------------------------------------------------------------------------------------------------------Product Trial's ReviewApril 20-- Spotlight: Paper Shredders

Most reviewers agree that cross-cut shredders are more secure than strip-cut shredders. This is because cross-cut shredders produce paper confetti, which is more difficult to put back together than the paper strips from strip-cut shredders. What our reviewers found was that for light-use paper shredders, the strip-cut models functioned better than the cross-cut models. The light-use cross-cut models that we tested were prone to paper jams.

Product Trial tested more models made by Shuffle because they make the most paper shredders of any company. We also noticed that Shuffle receives more complaints than any company, especially for their light and medium use models. Heavy use models by Shuffle are known for their quality and durability.

Our favorite paper shredder is the Privacy Solutions PL-7800, which retails for $120. This cross-cut paper shredder works well, did not jam in any of our tests, and is well worth the money. The Privacy Solutions PL-7800 is rated for ten sheets of paper and, although it operated faster with fewer sheets, it had no trouble with all ten. It can handle about 300 sheets per day. It can also shred CDs. If you don’t need a CD shredder and have minimal shredding needs, the Privacy Solutions PL-6100, which retails for $79, is a good deal.

186. Which model sells for $58?(a) Shuffle Shred GP710

(b) Shuffle Shred DC3088(c) Sander Cross-Cut RDL-611X

Page 140: 101s1.downloadmienphi.net/file/downloadfile2/180/1403331.doc · Web viewDuring normal office hours, University Maintenance will respond directly to a power failure. After hours, the

(d) Privacy Solutions PL-4200.

187. What is said about the Shuffle Shred GS-61?(a) It is easy to empty.(b) It has an extra-long cord.(c) It has three different settings.(d) It can cut up to eight sheets at a time.

188. What is claimed about cross-cut shredders? (a) They are considered more secure.(b) They can cut more pages at one time.(c) They do not jam as much as strip-cut shredders.(d) They are available only for medium or heavy use.

189. Which of the following is NOT true about the Privacy Solutions PL-7800?(a) It can shred CDs.(b) It retails for $79.(c) It can handle up to 300 pages per day.(d) It is Product Trial's favorite paper shredder.

190. What is learned about Shuffle?(a) They have excellent customer service.(b) They make very good medium use shredders.(c) Their products sell for more than others.(d) They make more shredders than most companies.

Page 141: 101s1.downloadmienphi.net/file/downloadfile2/180/1403331.doc · Web viewDuring normal office hours, University Maintenance will respond directly to a power failure. After hours, the

To: Steve Yang From: Arthur Goodman

Date: March 3, 20-- 11:02:24 a.m.Subject: Management Training Sessions

Attachments: goodman_calendar.doc----------------------------------------------

Hi Steve,

It was good to hear from you earlier today. It's been very busy around here too. We just finished our job candidate interviews yesterday. This means that today we will meet to figure out which candidates we would like to talk to further. It will probably be a long meeting.

I am glad you contacted me about when we can organize this year's executive management training sessions. It's hard to believe this will be the third year we've done it. I'm sending you my calendar for the month of April, so we can pick our dates. As you'll see, I'm pretty open at this point. Once we have the dates figured out, we should talk about whether we'd like to change any parts of the curriculum from last time. We should put together an initial list of people to invite.

I would also like to talk about turning our training materials into a book, but this is not urgent.

I hope you are well. Thanks for taking the initiative on organizing the third training session.

Art Goodman ------------------------------------------------------------------------------------------------------------------------------

191. What is the purpose of the e-mail?(a) To find a date for a training session(b) To recruit a training session coordinator

(c) To reserve a location for the training session(d) To create a guest list for the training session

Page 142: 101s1.downloadmienphi.net/file/downloadfile2/180/1403331.doc · Web viewDuring normal office hours, University Maintenance will respond directly to a power failure. After hours, the

192. What is learned about Arthur Goodman's company?(a) It is promoting a book.(b) It is hiring a new employee.(c) It is creating a new department.(d) It is updating employee information

193. What can be inferred about the executive management training session? (a) It lasts for 4 days.(b) It will be held at a hotel.(c) It is required for all managers.(d) It has been conducted twice before.

194. What is scheduled for April 20th?(a) A book Deadline(b) A managers' meeting(c) A meeting in Atlanta(d) An appointment with Dr. Lin

195. On which date is the Regional Meeting?(a) April 8(b) April 14(c) April 23(d) April 29

From: Kenan Beauchamp [[email protected]]To: Customer Service [[email protected]]Subject: December bill requestDate: January 11

To Whom It May Concern:

My account with Henderson Utilities Company is set up such that my monthly utility bills are directly deducted from my checking account. I noticed this month that $187.33 had been deducted from my account for utilities. Since becoming a customer of HUC, my monthly utility bill has never exceeded $150. Since I never received a paper bill from you, I'd like to request a copy of the bill to ensure that I was not overcharged for any services. Thank you.

Sincerely,

Kenan Beauchamp.------------------------------------------------------------------------------------------------------------------------------Henderson Utilities Company 43 Warm Springs Road

Henderson, Nevada 89014

(702) 493-4343

Account number: 920-53-10332

Name: Kenan Beauchamp

Address: 19 Galleria Drive

Henderson, Nevada 89002

Billing period: December 7 - January 7

Due date: January 23

Total amount due: $187.33

Page 143: 101s1.downloadmienphi.net/file/downloadfile2/180/1403331.doc · Web viewDuring normal office hours, University Maintenance will respond directly to a power failure. After hours, the

Water: $31.87

Electricity: $42.11

Gas:$113.35* Open an online billing account today to have your bills delivered to your e-mail account. With online billing, you can also pay your bills online. To sign up for an account, visit www.x.com/onlinebill.

196. According to the letter, what is true about Mr. Beauchamp's account?(a) It is an online account.(b) He opened in more than ten years ago.(c) He deactivated it at the beginning of December.(d) It allows bills to be deducted from his checking account.197. What does Mr. Beauchamp say he did NOT receive?(a) A bill(b) A service(c) A receipt(d) A discount

198. What is the due date of the bill? (a) December 7(b) January 7

(c) January 11(d) January 23.199. How much was Mr. Beauchamp charged for electricity?(a) $31.87(b) $42.11(c) $113.35(d) $187.33200. According to the bill, how can customers pay their bills online?(a) By filling out a form(b) By visiting a Web page(c) By e-mailing customer service(d) By calling a customer service hot line

ANSWER101. C 102. D 103. C 104. D 105. A 106. C 107. A 108. C 109. B 110. A

111. A 112. D 113. A 114. D 115. C 116. A 117. D 118. B 119. D 120. B

121. D 122. C 123. C 124. B 125. D 126. C 127. B 128. C 129. B 130. A

131. B 132. D 133. C 134. B 135. A 136. A 137. B 138. A 139. D 140. B

141. C 142. D 143. C 144. C 145. C 146. A 147. A 148. B 149. D 150. D

151. D 152. A 153. A 154. D 155. B 156. D 157. C 158. A 159. A 160. D

161. C 162. C 163. C 164. C 165. B 166. A 167. B 168. D 169. A 170. D

171. A 172. C 173. A 174. C 175. D 176. D 177. A 178. B 179. C 180. A

181. D 182. B 183. A 184. C 185. A 186. B 187. A 188. A 189. B 190. D

191. A 192. B 193. D 194. A 195. D 196. D 197. A 198. D 199. B 200. B

Page 144: 101s1.downloadmienphi.net/file/downloadfile2/180/1403331.doc · Web viewDuring normal office hours, University Maintenance will respond directly to a power failure. After hours, the

101. Lucy, a three-year-old Yorkshire terrier, ------- against 23 other dogs to win the 18th annual Courtland Dog Show. (a) formed(b) repeated(c) followed(d) competed

102. A law was ------- passed that gives tax benefits to landowners who sell their land to the government. (a) lately(b) hardly(c) recently(d) repeatedly

103. Foreman, Inc. usually ------- retiring employees luxury watches as farewell gifts. (a) gives(b) given(c) giving(d) is giving

104. The ambassador to Vietnam oversaw the opening ------- at the Music Festival. (a) studio(b) effect(c) ceremony(d) termination

105. Jan installed security software on her new computer to ------- it against viruses. (a) protect(b) protector(c) protection(d) protective

106. Bills for electricity ------- heating should be paid by the 15th of every month. (a) so(b) nor(c) but(d) and

107. Every year, the Healthy Heart Research Committee holds meetings in five different cities to spread information about the treatment and ------- of heart disease. (a) cured(b) access(c) prevention(d) carefulness

108. Customers who spend more than $100 will earn a 25% discount on ------- next purchase. (a) they(b) their(c) them(d) those

109. The sales meeting was ------- to after the four-day vacation. (a) entered(b) declared(c) revealed(d) postponed

110. "------- say that property taxes have increased faster than most homeowner's incomes. (a) Analyze(b) Analysts(c) Analysis(d) Analyzable111. Please ------- all of the information on the application form is accurate to the best of your knowledge. (a) true(b) copy(c) handle(d) verify

112. David complained to the Better Business Bureau because his landlord continued to ------- the broken heater in his apartment. (a) fail(b) order(c) neglect(d) disrespect

113. County legislators called for an update of the safety ------- for the Darcy Power Plant. (a) regulate(b) regulatory(c) regulative(d) regulations

114. The Davidson Car-Buying Guide lists prices for new and used cars in addition to ------- car safety ratings. (a) providing(b) canceling(c) supporting(d) demonstrating

Page 145: 101s1.downloadmienphi.net/file/downloadfile2/180/1403331.doc · Web viewDuring normal office hours, University Maintenance will respond directly to a power failure. After hours, the

115. This year's ------- for the Wilks Award for public service include a research scientist, a university professor, and a local politician. (a) nominees(b) nominate(c) nominator(d) nomination

116. If we win the Best Product Design Award, our team ------- a bonus. (a) receive(b) received(c) receiving(d) will receive

117. Our beautiful floral bouquets each come with a card for you to write a personal ------. (a) gift(b) limit(c) season(d) message

118. Peter Parson's directorial debut, Legend Has It, ------- the biggest box office numbers of the year, earning more than $107 million on opening weekend alone. (a) achieved(b) achiever(c) achievable(d) achievement

119. Internet voting in local and parliamentary ------- is used throughout Estonia. (a) speeches(b) elections(c) politicians(d) installations

120. Karen was able to include the last-minute article in the evening ------- of the newspaper. (a) edition(b) remark(c) volumes(d) limitation

121. Last year, nearly $14 billion was ------- in funds across Asia and Latin America. (a) invest(b) invested (c) investor (d) investment

122. To request more ------- about any of our products and services, please visit www.appliancepro.com/info. (a) brochure(b) problems(c) information(d) representation

123. Basin Air announced that it would begin running flights at Stewart Airport, which is currently ------- the State Port Authority control. (a) as(b) than(c) under(d) between

124. Two public forums will be held to discuss options for ------- the Mission Bridge. (a) repair(b) repaired(c) repairing(d) repairable

125. Customers purchasing a Camcom 65T digital camera are entitled to a factory ------- of $75. (a) film(b) rebate(c) visual(d) challenge

126. Chef Lendo Taverni's popular spicy broiled shark steak and mushroom salad is a favorite ------- customers. (a) past(b) among(c) toward(d) despite

127. Thom Gilbert will be ------- on the All-Star Comedy Tour, which starts on September 4. (a) staged(b) featured(c) created(d) detailed

128. All ------- are advised to use the printer in room 11B until the one in the faculty room has been repaired. (a) employ(b) employees(c) employers(d) employable

Page 146: 101s1.downloadmienphi.net/file/downloadfile2/180/1403331.doc · Web viewDuring normal office hours, University Maintenance will respond directly to a power failure. After hours, the

129. Searchlaw.com ------- an online directory of local lawyers and law services. (a) offers(b) guides(c) educates(d) distinguishes

130. Please take all personal belongings with you ------- leaving the train. (a) when(b) what(c) whom(d) which

131. A $20 fine will be ------- to replace any lost or stolen card keys. (a) charge(b) charged(c) charges(d) charging

132. Because of the ------- of hospital employees at Bon Secours Hospital, the head of administration began advertising job openings in the newspaper.(a) short(b) shorter(c) shorten(d) shortage

133. Any player who wishes to participate in the Gracey Tennis Tournament should register ------- August 14. (a) until(b) about(c) since(d) before

134. Teaching ------- must attend orientation sessions before their first day of class. (a) booklets(b) promoters(c) assistants

(d) instructions

135. Passports must be valid for at least six months to be ------- for a travel visa. (a) enable (b) quality (c) eligible (d) appropriate136. Your wage ------- at a rate of $21 for every hour of work. (a) calculated(b) calculation(c) to calculation(d) will be calculated

137. Morocco's ------- industry provides the third largest source of revenue for the North African country. (a) tour(b) touring(c) tourism(d) tour's

138. Many real-estate agencies are ------- professional photographers to take pictures of houses and properties for sale. (a) hiring(b) residing(c) excluding(d) converting

139. ------- the weather forecast, Clayton City should get more than fifteen inches of snow this winter. (a) At will of(b) According to(c) Depending on(d) Agreeing with

140. Beginning next week, managers will be ------- performance evaluations for all employees within their department. (a) conduct(b) conducts(c) conducted(d) conducting

Page 147: 101s1.downloadmienphi.net/file/downloadfile2/180/1403331.doc · Web viewDuring normal office hours, University Maintenance will respond directly to a power failure. After hours, the

Questions 141-143 refer to the following letter.

June 12

Victor Lee25 East Michigan AvenueLansing, Michigan 48912

Dear Mr. Lee,

Thank you for choosing to stay at the Maximus Hotel on your recent trip to Cleveland. Our records show that you stayed in our Grand Suite ______ the dates of May 4 and May 7.

141. (a) in(b) between (c) amongst(d) without

We hope that you enjoyed your stay with us and would appreciate any feedback you could give us on how you found our ______ and facilities.

142. (a) funds(b) storage(c) services(d) fractions

If you are interested in ______ a survey regarding your stay, please visit www.maximushotel.com/survey. Thank you for your time.

143. (a) complete(b) completing(c) completion(d) completely

Sincerely,

June WeaverHead of Customer RelationsMaximus Hotel, Cleveland

Questions 144-146 refer to the following letter.

Beginning in October, Voyage Bus Co. will add a new bus route ______ commuters from Maxwell County with the Harrison Metro train station line to Harrison City.

144. (a) connect (b) connector(c) connection(d) connecting

The company currently runs 23 buses throughout the tri-county area and is the preferred mode of transportation for commuters to ______ Yorktown and Harrison City, providing low fares and quick service.

145. (a) none(b) both(c) either(d) neither

Voyage Bus Co. is able to keep its fares down with the ______ it receives from advertisers. Advertisements appear,

Page 148: 101s1.downloadmienphi.net/file/downloadfile2/180/1403331.doc · Web viewDuring normal office hours, University Maintenance will respond directly to a power failure. After hours, the

not only on the side of the bus, but also in the magazines provided on the bus, and in TV commercials aired on the bus's television station during the commute.

146. (a) compensation(b) distribution(c) requirements(d) probabilities

Questions 147-149 refer to the following letter.Kaplan Park's 24th Annual Ice Festival

Visit Kaplan Park for the 24th annual Ice Festival during the week of Monday December 26 to Sunday January 1 ______ 10 A.M. until 8 P.M. each day.

147. (a) from(b) with(c) after(d) without

There will be ice skating on Dorchester Pond, horse-drawn sleigh rides around the park, an ice sculpting contest, and an ice skating ______ by local pairs figure skating champions Lawrence Novak and Francesca Temple.

148. (a) perform(b) performer(c) performance(d) performable

* Tickets to the event will be sold at the gate.* The $10 admission fee will ______ you to enjoy any event you choose, including free food and beverages from

the park's refreshment stand. 149. (a) share

(b) advise (c) contest(d) entitle

Questions 150-152 refer to the following letter.

Attention all employees:

There has been a change in policy regarding office supply orders. From now on, all orders must be ______ through Cameron in administration.If you would like to order something, please fill out an office supply request form, submit it to Cameron, and she will contact the supply company. Forms will be available in the faculty room.

150. (a) printed (b) purchased(c) processed(d) particular

______, Cameron will distribute all of the supplies once we receive them.151. (a) Added

(b) Adding to(c) Addition to(d) In addition

Any extra supplies will be kept in the cabinet at the back of the faculty room. Please check there for supplies before ______ an order.

152. (a) place

Page 149: 101s1.downloadmienphi.net/file/downloadfile2/180/1403331.doc · Web viewDuring normal office hours, University Maintenance will respond directly to a power failure. After hours, the

(b) places(c) placed (d) placing

Thank you,Anne.

Career.com -- Job Profile

Job: Architect (entry level)

Duties associated with this job include: Data compilation, design computation, and elementary architectural assignments.Possible duties: Estimate and plan preparation or structure inspection.Requirements: Bachelor's degree in architecture. No professional experience required.Base salary range: $30,000 to $50,000 a year.

* data specific to the New York City area

153. What is being offered?(a) A marketing position(b) An architect position(c) A graphic design position(d) A elementary school teaching position

154. According to the information, what is true about the job?(a) It requires a master's degree.

(b) It pays between $50,000 and $80,000 a year.(c) It is a high level position within a company.(d) It does not require professional experience

Shopping Center Proposal Worries Residents

Residents of the town of Crowley are worried that the shopping center Hertz Corp. has proposed to build on Burlingham Road will destroy the small town feel of their quiet community. More than 20 residents spoke against the proposal at a town board meeting held on Monday. “I moved here from the city to escape this kind of development,” said Tyrone Devlin, who lives near the proposed construction site. ”The traffic and crowds this shopping center will bring is exactly what we don't want here.”

The shopping center would be more than 600,000 square feet of commercial space, including a movie theater, department store, family restaurant, and several smaller retail shops.The town council will vote on the proposal on Monday, March 16 after a thorough environmental evaluation of the construction site is conducted over the weekend.

155. What is inferred about Crowley?(a) It is a small community.(b) It is a popular tourist area.(c) Its new campaign welcomes investors.(d) It currently has one major shopping mall156. According to the news article, what will be included in the shopping center?

(a) A pet store(b) A movie theater(c) A sports complex(d) A roller skating rink.

157. What will happen on Monday, March 16?(a) A protest will be held.

Page 150: 101s1.downloadmienphi.net/file/downloadfile2/180/1403331.doc · Web viewDuring normal office hours, University Maintenance will respond directly to a power failure. After hours, the

(b) The city council will vote on a proposal.(c) An environmental evaluation will be conducted.

(d) The city council will tour the construction

site.Owner's Choice Home Insurance

At Owner's Choice, we offer a variety of coverage options for your home through our comprehensive insurance policies. Our goal is to guarantee the protection of your home should you need to file an insurance claim due to damage caused by fire, flood, or burglary.

* You Can Lower Your Insurance RatesYou may already be doing things that could lower the cost of your homeowner's insurance. Owning a number of protective devices such as security systems can lower your insurance premium by up to seven percent.

For more information on policies and a list of ways to lower your insurance payments, visit www.ownerschoice.com/lowerpayments.

158. What type of insurance is being offered?(a) Home(b) Health(c) Travel(d) Automobile

159. What is mentioned as a way for customers to lower insurance payments? (a) By renovating a home(b) By owning a security system(c) By meeting special health requirements(d) By maintaining an accident-free record.

Study International Business in Beijing with the Excel Business Institute

Program DescriptionThe Excel Business Institute is an accredited international business school affiliated with Chicago's Milford University. The international business program in Beijing is hosted by Zheng Luo University located in the heart of the bustling eastern capital. The year-round study abroad program focuses on three areas: Chinese language and culture, Chinese business and economics, and Chinese business law. Student instruction will be supplemented with business related field trips and guest lectures by CEOs of major Chinese corporations. At the program's completion, internship opportunities in China will be offered to students who have successfully completed the coursework.

CreditCourse credit is available through Milford University.

Tuition$11,000 (price excludes housing)

Living ArrangementsOnce enrolled, students will be given a choice of three housing arrangements: independent living, home-stay, or dormitory.

Application RequirementsAll applicants must be majoring in business, economics, or a related field. Applicants must submit two letters of reference, a transcript, an application form, and a recent record of health.

160. Where does the program take place?(a) In Taipei

(b) In Beijing(c) In Shanghai(d) In Hong Kong

Page 151: 101s1.downloadmienphi.net/file/downloadfile2/180/1403331.doc · Web viewDuring normal office hours, University Maintenance will respond directly to a power failure. After hours, the

161. What is NOT a focus of the program?(a) Politics(b) Business law(c) Language and culture(d) Business and economics.

162. According to the information, what is available once the program is completed?

(a) A degree(b) A certificate(c) A letter of reference(d) An internship opportunity.

163. What document does the application require? (a) A resume(b) An essay(c) A degree(d) A transcript

To: Connie Caruthers [[email protected]]From: [[email protected]]Subject: Gas CardDate: June 19

Dear Ms. Caruthers,

As a preferred credit card holder of Quanex, we would like to offer you our new gas card. The Quanex Gas Card will give you a five percent cash back bonus reward every time you use it to purchase gas, have your car repaired, or buy something at our online automotive store. There is no limit on how many cash back bonus rewards you receive, and your rewards will never expire. You may request to receive your cash back bonus rewards in the form of a check or an account credit at any time. Additionally, the Quanex Gas Card comes with advanced security features to ensure that you are never responsible if your card is lost or stolen.

If you sign up for a Quanex Gas Card within the next two weeks, you will receive zero percent APR until March.

To receive an application for your card today, visit our Web site at www.quanex.com/gascard, call our 24-hour customer service line at 1-800-267-4442, or respond to this e-mail.

Page 152: 101s1.downloadmienphi.net/file/downloadfile2/180/1403331.doc · Web viewDuring normal office hours, University Maintenance will respond directly to a power failure. After hours, the

164. What is implied about Ms. Caruthers?(a) She owns more than one car.(b) She owns another Quanex credit card.(c) She has recently had her car repaired.(d) She has visited the Quanex online store before.165. The word “preferred” in paragraph 1, line 1, is closest in meaning to: (a) agreed(b) favored(c) accepted(d) improved166. According to the e-mail, what is true about the offer?

(a) Customers can use the card for car repairs.(b) Zero percent APR is available until January.(c) Security features must be purchased separately.(d) Customers can get 10 percent in cash back bonus rewards.

167. What is NOT listed a way for customers to receive an application?(a) Visiting a Web site(b) Calling a phone number(c) Responding to an e-mail(d) Visiting the Quanex store.

Page 153: 101s1.downloadmienphi.net/file/downloadfile2/180/1403331.doc · Web viewDuring normal office hours, University Maintenance will respond directly to a power failure. After hours, the

Ocean Bay Lunch Specials

* Lunch specials are available every weekday from 12 P.M. until 3 P.M.

Entrees

Seafood Platter… $13.99A wonderful trio of three of our best dishes: lightly breaded shrimp, buttered scallops, and a seasoned fish fillet.

Fish of the Day… $10.99Each day we serve a different fish, which you may choose to have fried, grilled, or broiled to perfection.

Seafood Caesar Salad… $8.99Your choice of broiled lobster or shrimp served over crisp romaine lettuce and fresh tomatoes with Caesar salad dressing and garlic croutons.

Seafood Pasta… $9.99Your choice of lobster, crab, or shrimp served with vermicelli in a tomato or Alfredo sauce.

* Each lunch special comes with your choice of a drink, soup or salad, and two side dishes.

168. With what meal can customers choose crab? (a) The Seafood Pasta(b) The Seafood Platter(c) The Fish of the Day(d) The Seafood Caesar Salad

169. How much is the Fish of the Day?(a) $8.99

(b) $9.99(c) $10.99(d) $13.99.

170. What is NOT free with a lunch special? (a) A drink(b) A salad(c) A dessert (d) A side dish

Dear Ms. Robinson,

I am the vice president of Chestnut Development Group, the real estate development company that will be leading the development of 40 acres of property at Philadelphia's south end. We strongly believe that the development of this area will be vital to the city's economic growth and revitalization.

We would like to invite you and your constituency to attend a weeklong series of urban planning meetings to determine how best to restore the property. Among participants in the meetings will be top architects and urban planners, local politicians and community leaders.

Our series of planning sessions will be lead by famed urban planner Gustav Temoin, who founded the School of Temoin Urbanism.

Our first meeting will be held on Monday November 23 from 6 P.M. to 8 P.M. in the Wallace Town Hall building. Four other meetings will follow during that week. I hope to see you on Monday.

Sincerely,

Page 154: 101s1.downloadmienphi.net/file/downloadfile2/180/1403331.doc · Web viewDuring normal office hours, University Maintenance will respond directly to a power failure. After hours, the

Leonard Cook, Sr.Vice PresidentChestnut Development Group

171. What is the purpose of this letter?(a) To confirm his attendance at a meeting(b) To invite Ms. Robinson to attend a meeting(c) To thank Ms. Robinson for organizing a meeting(d) To accept Ms. Robinson's invitation to speak at a meeting

172. The word “restore” in paragraph 2, line 2, is closest in meaning to:(a) retain(b) reform(c) replace(d) reconstruct.

173. According to the letter, who will NOT attend the meeting?

(a) Architects(b) Politicians(c) Professors(d) Urban planners.

174. Who is Mr. Temoin? (a) A famous urban planner(b) The superintendent of a school(c) A Philadelphia community leader(d) The president of Chestnut Development Group

175. When will the first meeting be held?(a) On November 6(b) On November 15(c) On November 18

(d) On November 23.

From: Camille StoneTo: All employeesSubject: Next week's faculty meetingDate: Tuesday, January 6

Hello,

Because next week's holiday falls on a Thursday, we will have to reschedule our weekly faculty meeting to Friday. Please meet in the faculty room by 5 P.M. on Friday January 16. If you're taking Friday as a vacation day, please let Shirley in the general affairs know by Wednesday.Thank you and enjoy your day off!

Camille

176. On what day is the holiday?(a) Tuesday(b) Wednesday(c) Thursday(d) Friday.

177. What will be rescheduled?

(a) A presentation(b) A holiday party(c) A faculty meeting(d) A training workshop.

Love Animal Shelter

Foster Owner ProgramYou can help save a dog's life by becoming a foster owner until we find a loving owner to adopt your pet or until we

Page 155: 101s1.downloadmienphi.net/file/downloadfile2/180/1403331.doc · Web viewDuring normal office hours, University Maintenance will respond directly to a power failure. After hours, the

reunite them with their original owner. We have dogs of all ages for you to care for, and many require special attention.

Although being a foster owner is one of the most rewarding experiences in the world, its emotional, physical, and financial demands should be given careful consideration. For this reason, we require all of our applicants go through an orientation program at our shelter so that they can get used to the responsibilities of caring for a dog and decide for themselves if they would like to continue doing it. We will also have one of our staff conduct a home and lifestyle evaluation to determine if each applicant's living situation is suitable for one of our dogs. Additionally, once a dog has been placed in your care, our adoption center will be on call 24 hours a day, seven days a week to help with any situations that arises.

For more information on our foster owner program, including testimonials from current and previous foster owners, visit www.loveanimalshelter.com.

178. What is the purpose of the advertisement? (a) To encourage people to adopt a pet(b) To encourage people to become foster owners(c) To encourage people to donate to the animal shelter(d) To encourage people to volunteer at the animal shelter

179. What does Love Animal Shelter require interested people to do?(a) Send a personal check

(b) Fill out some paper work(c) Complete an orientation program(d) Visit the adoption center often.

180. According to the advertisement, what is available on the Web site?(a) Forms(b) Pictures(c) Guidelines(d) Testimonials.

Natasha GoldenMcIntyre and BaranowskiSuite 11431 Echo Lake AvenueMahtomedi, Minnesota 55115

Dear Ms. Golden,

I've enclosed an employee reimbursement form along with the receipts from my business trip to Orlando. As I was able to use the company card to book my flight and hotel reservation, those expenses are not accounted for on the list.

Please advise on the time it takes to be reimbursed. It usually takes one month, however, it took three months to be reimbursed for my last business trip to New York.

Thank you for your help in this matter.

Sincerely,

Bob Yuengling

Page 156: 101s1.downloadmienphi.net/file/downloadfile2/180/1403331.doc · Web viewDuring normal office hours, University Maintenance will respond directly to a power failure. After hours, the

-------------------------------------------------------------------------------------------------------------------------

McIntyre & Baranowski

Employee Reimbursement Form

Employee Name: Bob Yuengling

Employee ID Number: 002938

Reason for Expenses Incurred: Business trip to meet with client

Account of Expenses:

Dinner- Luna's Restaurant $16.34 Tuesday May 20

Transportation to client's office- Taxi $13 Wednesday May 21

Dinner with client- The Kitchen Cafe $107.03 Wednesday May 21

Transportation back to hotel- Taxi $13 Wednesday May 21

Dinner- Tres Amigos Grill $23.03 Wednesday May 21

Employee Signature: Bob Yuengling

181. Where is implied about Mr. Yuengling?(a) He travels monthly.(b) He lives in Florida.(c) He works as an accountant.(d) He works for McIntyre and Baranowski.

182. Where did Mr. Yuengling go?(a) To Orlando(b) To New York(c) To Minneapolis(d) To Mexico City

183. How long does it ordinarily take to get a reimbursement? (a) One month

(b) Two months(c) Three months(d) Four months

184. What day did Mr. Yeungling take a taxi?(a) Monday(b) Tuesday(c) Wednesday(d) Thursday

185. According to the list, where was the most money spent?(a) At Tres Amigos Grill(b) At The Kitchen Café(c) At Antonio's Restaurant

Page 157: 101s1.downloadmienphi.net/file/downloadfile2/180/1403331.doc · Web viewDuring normal office hours, University Maintenance will respond directly to a power failure. After hours, the

(d) At Luna's Business Hotel

To: Gloria Stefani [[email protected]]From: Reesa Bonjani [[email protected]]Subject: February fireworks scheduleDate: December 16

Dear Ms. Stefani,

Thank you for inquiring about the fireworks at Harbor Falls waterfall. Attached to this message you'll find a schedule of our fireworks displays for the month of February. I'm sure you'll be able to find a suitable date around which to plan your trip.

Also, if you're interested in staying at one of the hotels that offers a view of the waterfall and fireworks, please visit our Web site at www.harborfalls.net and click on the link for nearby hotels. Don't hesitate to contact me with any more questions.

Enjoy your stay at Harbor Falls.

Sincerely,

Reesa BonjaniHarbor Falls Department of Tourism.-------------------------------------------------------------------------------------------------------------------------Harbor Falls Fireworks Schedule

February Thursday February 1(for the beginning of February)

Saturday February 10

Wednesday February 14(for Valentine's Day)

Saturday February 17

Sunday February 18(for the Lunar New Year)

Monday February 19(for President's Day)

Saturday February 24

* Fireworks displays during the month of February will start at 9 p.m.* Dates and times are subject to change and will be updated on the Web site when that information becomes available.

Page 158: 101s1.downloadmienphi.net/file/downloadfile2/180/1403331.doc · Web viewDuring normal office hours, University Maintenance will respond directly to a power failure. After hours, the

* Fireworks displays may be cancelled due to adverse weather conditions.

186. According to the e-mail, what has Ms. Stefani done?(a) Visited Harbor Falls(b) Reserved a hotel room(c) Requested information(d) Contacted her travel agent.

187. What does Ms. Bonjani suggest that Ms. Stefani do?(a) Reschedule her trip(b) Read the Harbor Falls guidebook(c) Stay at a hotel with a view of the waterfall(d) Speak with the head of the tourism department

188. On what day will fireworks NOT be displayed? (a) February 1

(b) February 7(c) February 10(d) February 14

189. Why will fireworks be displayed on February 18?(a) For the weekend(b) For President's Day(c) For the Lunar New Year(d) For the hotel's anniversary.

190. According to the schedule, what information will be updated on the Web site?(a) Hotel room rates(b) Fireworks display times(c) Hotel room availability(d) Fireworks display ticket prices

Page 159: 101s1.downloadmienphi.net/file/downloadfile2/180/1403331.doc · Web viewDuring normal office hours, University Maintenance will respond directly to a power failure. After hours, the

Paradise Cineplex70 Red RoadMiami, Florida 33126305-463-2226

The MuseumThree middle school students left behind on a class trip spend the night in a history museum and uncover the secrets to an ancient mystery.

Running time: 2 hours, 15 minutesGenre: Mystery, FamilyStarring: Jessie-Grace Turner, Clint Sandoval, Danika Brooke-Pierce, Vivienne Aldrich, and PhillipBanks

Director(s): Harvey FeddermanProducer(s): Louisa Marquez, Daniel LevistonWriter(s): April PascucciStudio: Cinereve

Show times: Beginning August 1210:00 A.M., 11:30, 1:00 P.M., 2:30, 4:00, 5:30, 7:00, 8:30, 10:00, 11:30 ------------------------------------------------------------------------------------------------------------------------------Paradise Cineplex

* Please print the following ticket information. You may need to present it before gaining admittance to the theater.

Online TicketTicket issue number: 002948275H

Movie: The MuseumDate: August 14Time: 4:00 P.M.

Credit Card InformationName: Sarah AdesidaAddress: 3027 West Flagler StreetMiami, Florida 33126

Phone number: 305-463-9738Account number: XXXX XXXX XXXX 2230.

191. What is true about the movie?(a) It is two hours long.(b) It debuted on August 1.(c) It takes place in a museum.(d) It was written by Daniel Leviston

192. Who is Phillip Banks?

(a) An actor(b) A writer(c) A director(d) A producer193. What is the genre of the movie? (a) Drama(b) Comedy

Page 160: 101s1.downloadmienphi.net/file/downloadfile2/180/1403331.doc · Web viewDuring normal office hours, University Maintenance will respond directly to a power failure. After hours, the

(c) Action(d) Mystery

194. According to the ticket, what should customers do?(a) Print a copy of the ticket(b) Reserve seats at the ticket booth(c) Arrive ten minutes before the movie begins

(d) Present their credit card at the ticket booth

195. When does Ms. Adesida plan to see the movie?(a) At 1 P.M.(b) At 4 P.M.(c) At 7 P.M.(d) At 10 P.M.

Investment Tips

By Scott Russell

Investing in stocks that are less than $5 per share is a good way to boost your stock portfolio. Many big investors ignore these stocks because of the potential risks involved, however, these stocks often grow significantly over the course of time. Investing in inexpensive stocks is a wise move, provided that you hold on to them for at least one year to allow enough time for them to develop.

Four stocks that I suggest for less than $5 per share are High Standard Pharmaceuticals Company, Nova Oil, Inc., Direct Access Publishing Group, and Peak Media Holdings. If you are new to the market, you might want to try investing in them through an online brokerage firm that does not charge a high commission for their services, and remember: only invest up to five percent of your entire stock portfolio in any stock, including these.------------------------------------------------------------------------------------------------------------------------------Quick Stock Quotes -- Thursday April 22 -- 2:40 P.M.(Quotes delayed by 20 minutes)Nova Oil, Inc. (NOI)3.93 +0.08 +2.08%

Previous Close 3.85Open 3.87High 3.94Low 3.79Volume 864,300Bid 3.91Bid Size 600Ask 3.97Ask Size 3,30052 Week range 1.64-8.90

196. What type of stocks is the article about?(a) Oil company stocks(b) Inexpensive stocks(c) Pharmaceutical stocks(d) Stocks from new companies

197. Why does Mr. Russell say big investors ignore these stocks?

(a) They are risky.(b) They are too popular.(c) They often lose money rapidly.(d) It is difficult to make money from them.

198. What investment strategy does Mr. Russell NOT suggest in the article? (a) Investing in more than one stock at once

Page 161: 101s1.downloadmienphi.net/file/downloadfile2/180/1403331.doc · Web viewDuring normal office hours, University Maintenance will respond directly to a power failure. After hours, the

(b) Investing through an online brokerage firm(c) Investing in the stock for at least one year(d) Investing up to five percent of a stock portfolio.

199. What company's stock is profiled?(a) Nova Oil, Inc.(b) Peak Media Holdings(c) Direct Access Publishing Group(d) High Standard Pharmaceuticals Company

200. At what price did the company's stock close on the previous day?(a) $3.79 per share(b) $3.85 per share(c) $3.91 per share(d) $3.93 per share

ANSWER101. D 102. C 103. A 104. C 105. A 106. D 107. C 108. B 109. D 110. B

111. D 112. C 113. D 114. A 115. A 116. D 117. D 118. A 119. B 120. A

121. B 122. C 123. C 124. C 125. B 126. B 127. B 128. B 129. A 130. A

131. B 132. D 133. D 134. C 135. C 136. D 137. C 138. A 139. B 140. D

141. B 142. C 143. B 144. D 145. B 146. A 147. A 148. C 149. D 150. C

151. D 152. D 153. B 154. D 155. A 156. B 157. B 158. A 159. B 160. B

161. A 162. D 163. D 164. B 165. B 166. A 167. D 168. A 169. C 170. C

171. B 172. D 173. C 174. A 175. D 176. C 177. C 178. B 179. C 180. D

181. D 182. A 183. A 184. C 185. B 186. C 187. C 188. B 189. C 190. B

191. C 192. A 193. D 194. A 195. B 196. B 197. A 198. A 199. A 200. B

Page 162: 101s1.downloadmienphi.net/file/downloadfile2/180/1403331.doc · Web viewDuring normal office hours, University Maintenance will respond directly to a power failure. After hours, the

101. The gross ------- for Amanti soft drink beverages went down 40 percent in the first quarter. (a) profit(b) profits(c) profiting(d) profitable

102. All employees are required ------- their identification card visible at all times. (a) has(b) had(c) having(d) to have

103. Please ------- your security code for entrance into the building. (a) type(b) enter(c) insert(d) compute

104. Proposals for the Canwood River project must be ------- before April 1. (a) submit(b) submits(c) submitted(d) submission105. Mr. Willis and Mr. Atwood have not ------- their attendance at the meeting. (a) checked(b) settled

(c) approved(d) confirmed

106. Returns will not be accepted ------- a receipt and the item must be in its original package. (a) without(b) apart from(c) in terms of(d) not including

107. Concert dates are not guaranteed and are subject to changes, modifications, or -------. (a) cancel(b) cancels(c) cancelled(d) cancellation

108. Please ------- the copy machine to warm up for five minutes before using it. (a) allow(b) allowed(c) has allowed (d) having allowed109. Contest participants are required to sign a release form ------- receiving any prize money. (a) sooner(b) before(c) earlier(d) ahead of

110. Ms. Robins is ------- to give a presentation on market mechanics on Thursday at 3 p.m. (a) schedule(b) schedules(c) scheduled(d) scheduling

111. There are still twenty seats ------- for the Saturday night banquet. (a) ready(b) prepare(c) available(d) convenient

112. The rapid growth of Marshall City led to a ------- of housing for many of its new residents. (a) lack(b) failure(c) decline(d) incomplete

113. A video teleconference ------- place on Thursday between the executives from the London and New York headquarters. (a) take(b) takes(c) took(d) taking

114. The addition ------- a safety latch has reduced work-time accidents by more than 35 percent. (a) of(b) in(c) on(d) since

Page 163: 101s1.downloadmienphi.net/file/downloadfile2/180/1403331.doc · Web viewDuring normal office hours, University Maintenance will respond directly to a power failure. After hours, the

115. As a Hamilton Value Club Member, you will continue to receive more great offers in the -------. (a) mail(b) mails(c) mailed(d) mailing

116. The company will celebrate its twenty-fifth anniversary at the Indiana State Fairgrounds, where its product originally -------. (a) began(b) debuted(c) entered(d) admitted

117. The magazine printed a ------- in the July issue of Build It! (a) retract(b) retracting(c) retracted(d) retraction

118. To read the full license agreement, please ------- to your instruction manual. (a) look(b) refer(c) direct(d) mention

119. The company is ------- a new initiative to reduce paper waste at work. (a) start(b) started(c) starting(d) having started

120. Sales of the video game Mutant Ties tripled those of ------- competitors during its weekend debut. (a) its(b) their(c) those(d) these

121. Passengers should arrive two hours ------- to departure time for international flights. (a) front(b) prior(c) before(d) above

122. To boost morale, the company will now ------- coffee, tea, and pastries in the employee break room. (a) make(b) contain(c) present(d) furnish

123. ------- the recent renovations to their kitchen, the value of the Spencer household remained the same. (a) Due to(b) Even if(c) Despite(d) Although

124. Traditional cooks ------- ginger is the key ingredient for a good stir-fry sauce. (a) insist(b) expect(c) demand(d) request

125. You are ------- for a Marble Merchandise bonus with your next in-store purchase. (a) eligible(b) acceptable(c) appropriate(d) satisfactory

126. ------- items for the movie Game Time include a sports thermos, gym towel, and a football. (a) Promotion(b) Promoting(c) Promotions(d) Promotional

127. The median household income for ------- in the area went up two percent last year. (a) houses(b) district(c) community(d) residents

128. In January, Mendel Tech will ------- Pioneer, its new software program that will replace its outdated system. (a) introduce(b) introduces(c) introducing(d) introduction

Page 164: 101s1.downloadmienphi.net/file/downloadfile2/180/1403331.doc · Web viewDuring normal office hours, University Maintenance will respond directly to a power failure. After hours, the

129. Saul Phillips and Mike Fishman ------- the creators of the new social networking site, Connect, that is currently sweeping the nation. (a) is(b) am(c) are(d) being

130. Setway product specials are exclusive ------- Setway customers and may not be used in conjunction with other rewards programs. (a) to(b) at(c) by(d) for

131. Travel prices are often ------- by gas price changes and consumers' desires to travel. (a) changed(b) depended(c) affected(d) suggested

132. Effective -------, all official WBJ statements must be cleared by the legal department.(a) soon(b) nearly(c) shortly(d) immediately

133. Money for two new full-time employees ------- into the budget for the next fiscal year. (a) was figured(b) is figuring(c) been figured(d) having figured

134. Statue National Bank will be closed in ------- of Memorial Day. (a) ceremony(b) awareness(c) acknowledge

(d) observance135. Mayor Bromwell pledged to make the City of Stockton more environmentally -------. (a) friend(b) friendly (c) befriended (d) friendship

136. The winner of the contest will be selected by a panel of judges ------- from the audience. (a) choose(b) chosen(c) chooses(d) choosing

137. Littleton Industries announced a fifteen percent profit in its third fiscal quarter, ------- analysts' expectations. (a) beat(b) beating(c) been beat(d) having beat

138. Employees are now required to get manager ------- for reimbursement requests. (a) support(b) agreement(c) approval(d) recognition

139. Please contact Marian Silva in the finance department if you are ------- in signing up as a volunteer. (a) involved(b) concerned(c) committed(d) interested

140. Priority consideration will ------- to current Danover, Inc. employees. (a) gives(b) be given(c) have given(d) been giving

Page 165: 101s1.downloadmienphi.net/file/downloadfile2/180/1403331.doc · Web viewDuring normal office hours, University Maintenance will respond directly to a power failure. After hours, the

Questions 141-143 refer to the following letter.

Gloria Sujarto19 Roadwell DriveMemphis, TN 16914

Ms. Sujarto,

This letter is to _____ you that we have received your customer inquiry concerning your recent Monumental Moments order.

141. (a) talk (b) inform(c) describe(d) communicate

We are reviewing our records and will contact you about your refund _____. Please disregard any payment notices you may receive in regards to your Monumental Moments order.

142. (a) appeal(b) request(c) comment(d) question

_____ you have any questions, please call our customer service line at 891-1290.143. (a) If

(b) Since(c) Maybe(d) Perhaps

Sincerely,

Jim WarnamontCustomer Services ManagerMonumental Moments.

Questions 144-146 refer to the following letter.

To: All EmployeesFrom: Heather DonnellySubject: Peter Yuki Visit

This is a _____ that on Friday, June 12, President Peter Yuki will visit our office.144. (a) memo

(b) mention (c) comment (d) reminder

He will be touring the Sanford & Sanford grounds and will meet _____ department heads.

145. (a) each (b) both (c) with (d) together

At 2:30 p.m., Mr. Yuki will give a short speech in the auditorium and respond to any employee concerns. _____ is

Page 166: 101s1.downloadmienphi.net/file/downloadfile2/180/1403331.doc · Web viewDuring normal office hours, University Maintenance will respond directly to a power failure. After hours, the

mandatory.146. (a) Attend

(b) Attends (c) Attending (d) Attendance

Questions 147-149 refer to the following letter.Looking for a creative work atmosphere? Ready for an exciting and challenging job? Sportage Studios is the industry's _____ sports game developer and exclusive game licensor of the Great Britain Football Association.

147. (a) lead (b) leads (c) leader (d) leading

We are _____ and need experienced Game Animators to join our team and work in our London offices.148. (a) beginning

(b) enlarging (c) expanding (d) developing

Ideal candidates should _____ at least three years experience in the industry and possess an advanced degree in computer animation.

For more information, please visit our careers website at www.sportagestudios.com/jobs.

149. (a) has(b) had (c) have (d) having

Questions 150-152 refer to the following letter.

Rate Increase

Effective October 12, _____ rates for our Sunday edition will increase.150. (a) advertise

(b) advertises (c) advertising (d) advertisement

Full-page ads will increase _____ $125 to $145. Half-page ads will increase from $55 to $65. Quarter page ads will stay the same at $35.

151. (a) from(b) while

Page 167: 101s1.downloadmienphi.net/file/downloadfile2/180/1403331.doc · Web viewDuring normal office hours, University Maintenance will respond directly to a power failure. After hours, the

(c) between (d) through

We are sorry for the price changes but find these measures necessary to _____ our costs.152. (a) keep

(b) control (c) maintain

(d) continue

Thank you for your understanding.

To: Bradley Rice ([email protected])From: Sarah Janovitch ([email protected])Subject: Paperwork ApprovalDate: November 10, 9:35 a.m.

Mr. Rice,

As of 10 November, I have not received the expense report for your October 15 trip to San Diego. Please remember that all paperwork must be turned in within one month after trip completion. To process your paperwork and receive your reimbursement, I must receive a completed report by November 15. Thank you for your cooperation.

Sarah Janovitch

Administrative AssistantBertelman Group

153. What does Ms. Janovitch request?(a) An expense report(b) A trip itinerary(c) An original receipt(d) A reimbursement form

154. When is the deadline?(a) October 10(b) October 15(c) November 10(d) November 15

SALES OF DARK CHOCOLATE UP

Gross sales of dark chocolate are up more than 30 percent this year, according to industry executive Marshall Wilcox. National Chocolate and Confection Association Vice President Wilcox made his announcement at a trade show in Providence on Tuesday. He attributed dark chocolate's rise in popularity to its reported health benefits and Americans' increasing appetite for gourmet foods. Years ago, white chocolate was trendy with milk chocolate gradually replacing it in popularity. Industry experts agree however, that dark chocolate sales will continue to rise and the sales growth is not merely a trend.

Demand for the antioxidant-rich dark chocolate is largely credited to the surge in a more health conscious public. People are getting more concerned about their health, and with consistent studies that support the benefits of eating cacao, tastes are shifting in favor of dark chocolate, said Wilcox.

Increased availability of the product was also cited as a contributing factor for the rise. The National Chocolate and Confection Association meets yearly to discuss the trends in chocolate and confection sales. The conference ended

Page 168: 101s1.downloadmienphi.net/file/downloadfile2/180/1403331.doc · Web viewDuring normal office hours, University Maintenance will respond directly to a power failure. After hours, the

on Thursday.

155. Who is Marshall Wilcox?(a) A chocolate maker(b) An industry analyst(c) An association executive(d) A confection company owner

156. Where was the announcement made? (a) At a trade show(b) At a factory tour(c) At a press conference(d) At a shareholder's meeting

157. According to the article, why are sales of dark chocolate increasing?(a) Lower prices(b) Richer flavor(c) More selection

(d) Increased availability

158. On what trend does the article focus?(a) The demand for exotic foods(b) The attention to food studies(c) The consumer desire for healthier foods(d) The increase in dark chocolate production

159. How often does the association meet? (a) Weekly(b) Monthly(c) Quarterly(d) Yearly

Tiny's Tacos (TNTC) Business Profile

Summary

Tiny's Tacos franchises, operates, and licenses fast food restaurants in the food service industry. Tiny's Tacos has 2,590 restaurants in the United States and 217 restaurants in more than 50 countries. The company operates 2,459 of those with the rest existing as franchises. Tiny's Tacos comes in either freestanding units for street corners or gas stations and as kiosks for shopping malls. The Tiny's Tacos Mexican food menu includes its signature Build-a-Taco bar and favorites such as burritos, rice, quesadillas, soft drinks. Tiny's Tacos also offers promotional food items monthly.

Management

Chief Executive Officer Samuel AnswellChief Operating Officer Wendy PritchardChief Financial Officer Warren BillingsPresident Charles ClintonSenior Vice President Fabrizio Ducali.

160. How many restaurants does Tiny's Tacos operate?(a) 50(b) 217(c) 2459(d) 2590

161. What is NOT true about Tiny's Tacos?

(a) It has international locations.(b) There are weekly promotions.(c) Mr. Answell is the CEO.(d) Customers can choose what to put in their tacos.

162. Where can Tiny's Tacos restaurants be found?(a) At gas stations(b) At street vendors(c) At movie theaters

Page 169: 101s1.downloadmienphi.net/file/downloadfile2/180/1403331.doc · Web viewDuring normal office hours, University Maintenance will respond directly to a power failure. After hours, the

(d) At sports stadiums

163. Who serves as the chief operating officer?(a) Samuel Answell

(b) Warren Billings(c) Charles Clinton(d) Wendy Pritchard

Stefan Wilder: In the 40sFilm Retrospective

On behalf of the Stefan Wilder Foundation, it is our pleasure to invite you to the exclusive opening film screening of Stefan Wilder: In the 40s on July 12.

The retrospective of Stefan Wilder's films from 1940-1949 will be screened on July 12, 13, and 14. It includes seven different films. Discussions with leading film theorists and Wilder scholars will follow each screening.

The Stefan Wilder Foundation was created to honor the work of one of the greatest and most productive film directors of all time. We hope you join us for our first annual retrospective.

Please confirm your attendance for the invitation-only screening by July 8. Contact Judy Slattery at 452-3920.

Page 170: 101s1.downloadmienphi.net/file/downloadfile2/180/1403331.doc · Web viewDuring normal office hours, University Maintenance will respond directly to a power failure. After hours, the

164. What is the invitation for? (a) A theater show(b) A film screening(c) A gallery opening(d) A musical performance

165. When will the event begin?(a) July 8(b) July 12(c) July 13(d) July 14.

166. What is true about the event? (a) It is free.(b) It is popular.(c) It is exclusive.(d) It lasts for four days

Page 171: 101s1.downloadmienphi.net/file/downloadfile2/180/1403331.doc · Web viewDuring normal office hours, University Maintenance will respond directly to a power failure. After hours, the

A New Taste SensationBy Smith Jackson

We asked recent Wilson School of Business graduate Marcus Jenkins to describe a normal workday as a product innovator at food industry giant Selling Incorporated.

I was hired out of Wilson to be an associate Product Idea Innovator at Selling Inc. I am responsible for creating, tasting, researching and helping to bring to market new food products for Selling.

Management at Selling encourages out-of-the-box thinking and I am constantly challenging myself. On any given day, I could be tasting new products, writing a summary evaluation, presenting a new idea to executives, or working on my own projects.

A Day in the Life

7:15 a.m. I head towards the office. My commute is about forty minutes long, so I try to catch up on my e-mails using my digital organizer. Sometimes I read the newspaper on my way to work.

Click here for the full article.*A Day in the Life - Business Times Feature*Must subscribe to Business Times magazine for full access to special features.

167. Who is Smith Jackson?(a) A student(b) A journalist(c) A researcher(d) A food tester

168. What type of organization is Selling Incorporated?(a) A research company(b) A business consulting firm(c) A food products company(d) A business school recruiting agency

169. What is NOT part of a typical day for Mr. Jenkins? (a) Meeting with clients(b) Coming up with ideas(c) Summarizing products(d) Researching new items.

170. How can a person read the full article?(a) By subscribing to the magazine(b) By entering their access code(c) By visiting the magazine website(d) By obtaining a print version of the magazine.

Page 172: 101s1.downloadmienphi.net/file/downloadfile2/180/1403331.doc · Web viewDuring normal office hours, University Maintenance will respond directly to a power failure. After hours, the

Monteverde Cosmopolitan pen

Monteverde pens have a tradition of excellence ever since founder Guiseppe Monteverde's first handmade pen was built in 1912. With years of experience and a dedication to the craft, Monteverde is an industry leader in creating refined, high-quality pens.

AXB8 - Monteverde Silverado - Delight the sophisticated business client with this handcrafted, sterling silver capped pen. Decorously tipped with an 18k gold nib, the ink flows freely and is aided by the comfortable rubber grip. The pen is available in both fountain and roller ball. Each pen comes engraved with a unique serial number to ensure authenticity. It also comes with a two-year warranty and satisfaction guarantee and is available in five designer colors. Act now and get free shipping.

Fountain pen $495 Roller ball pen $400Colors: Canary Yellow, Turquoise, Brick Red, Forest Green, Silver.

171. What is true about the Monteverde Compan? (a) It is a new company.(b) It produces stationery.(c) It creates luxury pens.(d) It makes products by custom-design only

172. What is NOT included with the pen?(a) A warranty(b) A free shipping offer

(c) A satisfaction-guarantee(d) A certificate of authenticity

173. What options are available for the pen?(a) The color(b) The plating(c) The engraving(d) The band coating.

Learn how to play golf with the experts! At Chuck Landry's School of Golf, we pride ourselves on our professionalism, dedication to sport, and proven performance. With state-of-the-art equipment that analyzes your swings and coaching from leading experts, Chuck Landry's School of Golf will get you swinging in no time.

Chuck Landry's School of Golf is offering a seasonal package special. For our seasonal summer special, get three days for the price of two.

For just $2000 you will get:- Luxury accommodation in the Biltmont Hotel- Transportation to and from the hotel- Personal one-on-one coaching with professionals- Daily lunch- Computer analysis- Nine-hole playing lesson

Call Chuck Landry's School of Golf today. This offer will not last long. 291-1920.

174. What promotion is being offered? (a) A seasonal package(b) An equipment sale(c) A fifty percent discount(d) An additional guest coupon

175. In the second paragraph, line 2, "special" is closest in meaning to:(a) sale(b) unique(c) correct(d) different.

Page 173: 101s1.downloadmienphi.net/file/downloadfile2/180/1403331.doc · Web viewDuring normal office hours, University Maintenance will respond directly to a power failure. After hours, the

176. What is included in the school's golf lessons?(a) A personal caddy(b) A computer analysis(c) A golf etiquette course

(d) A complimentary equipment cleaning

To: William Forest ([email protected])From: Janet Duncan ([email protected])Subject: New Employee HiringDate: September 9, 3:23 p.m.Attachment: hill-resume.doc, garcia-resume.doc, hill-cl.doc, garcia-cl.doc

Mr. Forest,

Attached are the resumes and cover letters of two potential candidates. I know you wanted to speak with each job applicant before a decision is made, so if you could, please review the information and tell me when it would be possible to schedule an interview. I have already had an informal interview with both Ms. Hill and Mr. Garcia.

Janet DuncanAssociate Financial ManagerWeinz Investments.

177. What does Ms. Duncan ask Mr. Forest to do?(a) To turn in a resume(b) To choose a candidate(c) To schedule a meeting(d) To review job applicants

178. According to the e-mail, how many people did Ms. Duncan interview?(a) Two(b) Three(c) Four(d) Five.

4. Insulate your water heater.

A cheap and easy way to cut down on heating costs is to insulate your water heater. Not all water heaters need it, but if your water heater is warm to the touch, you could be losing heat. In this case, insulating your water heater will reduce heat loss and can be done without professional help. Self-installation is easy. You can pick up a water heater insulation kit at your local hardware store for about $20. The cost of the kit will be made up with the money you will save on your energy costs.

179. When is insulation recommended? (a) If heating bills exceed $20(b) If a professional recommends it(c) If the water heater is warm to the touch(d) If the water heater is more than five years old

180. What is true about installing insulation?(a) It is easy to do.(b) It is a daylong project.(c) The kit must be ordered.(d) It should be handled by a professional.

Stafford Rent-a-Car Standard Rental Rates (Rates shown are for one 24-hour period)

Car Model Rate

Page 174: 101s1.downloadmienphi.net/file/downloadfile2/180/1403331.doc · Web viewDuring normal office hours, University Maintenance will respond directly to a power failure. After hours, the

Subcompact Car Geo Metro or similar $55.95Compact Car Ford Escort or similar $65.95Sport Utility Vehicle Ford Escape or similar $80.95Convertible Ford Mustang or similar $95.95Luxury Cadillac Seville or similar $115.95Mini Van Chevron Highlander or similar $125.95

Show your Stafford Rent-a-Car Membership and receive a 10% discount off of the daily rental rate-------------------------------------------------------------------------------------------------------------------------Date: January 5, 8:34 p.m.To: Reservations Center ([email protected])From: Peter Jimenez ([email protected])Subject: Car Reservation

Stafford Rent-a-Car Reservation Form

Name: Peter JimenezAddress: 1218 Watson DrCity: BaltimoreState: MDZip Code: 21075

Driver's License Number: B22BE38State: MD

Membership Card Identification Number: 18239-192191-11

Car InformationCar: Ford EscortSeats 5Holds 1 large suitcase, two medium-size bags

Rental Term: January 8-January 9, 1 dayCar Return: At same locationRate: $65.95Tax: $9.95Fees: $6.95Subtotal: $82.85Membership Card Discount: $8.29Total: $74.56

Thank you for choosing Stafford Rent-a-Car. You will receive a confirmation e-mail after license verification.

The total above is an estimate and is subject to change if location, dates, car type, or membership identification information change. Credit card information is not needed for reservations.

Page 175: 101s1.downloadmienphi.net/file/downloadfile2/180/1403331.doc · Web viewDuring normal office hours, University Maintenance will respond directly to a power failure. After hours, the

For any questions about your reservation, please call (888) 458-1921.

181. Which is the most expensive car to rent?(a) Geo Metro(b) Ford Escape(c) Ford Mustang(d) Chevron Highlander.182. How much would a luxury car cost for a one-day rental?(a) $80.95(b) $95.95(c) $115.95 (d) $125.95.183. What entitles Mr. Jimenez to a discoun?(a) A coupon(b) A package deal(c) A membership card

(d) An employee rate.

184. For how long does Mr. Jimenez make the reservation? (a) 1 day(b) 2 days(c) 3 days(d) 4 days.185. What must be verified?(a) A date request(b) A driver's license(c) A credit card payment(d) A specific car availability.

Page 176: 101s1.downloadmienphi.net/file/downloadfile2/180/1403331.doc · Web viewDuring normal office hours, University Maintenance will respond directly to a power failure. After hours, the

Attention Malfoy Corp's Employees- Free Seminar Series

Friday, March 20 is the beginning of Malfoy Corp.'s seminar series. First in the series is International Relations: Business Etiquette for China.

All seminars will be led by Malfoy Corp.'s employees and/or leading experts in their field. For a complete listing of available classes, please consult the Malfoy website at www.malfoy.com/seminar.

The classes are free for all Malfoy Corp.'s employees, but registration is required. Please sign up by March 18 to secure a seat.-------------------------------------------------------------------------------------------------------------------------International Relations: Business Etiquette for ChinaFriday, March 20, Hoover Conference RoomSignup deadline: March 18

Please leave your name, e-mail, phone extension, and office number.1. Ted Wilmont [email protected] 7619 3012. Judy Summers [email protected] 1201 1203. Ralph Tresvant [email protected] 7890 5204. Tejas Patel [email protected] 7892 5205. Phillip Lawrence [email protected] 1282 1236. Kristen Salley [email protected] 1910 1357. Misty Fitzgerald [email protected] 1019 1058. Thomas Vincent [email protected] 1012 1089. Sally Cross [email protected] 1018 10710. Justin Davis [email protected] 7620 30311. Lyle McKnight [email protected] 1029 10512. Cedric Green [email protected] 1020 105

*Coming next week, Friday, March 27: Presentations and Public Speaking, Conference Room B Signup deadline: March 25*.

186. Who is the seminar open to?(a) Local citizens(b) Company employees(c) Business students(d) Corporate executives.187. What is required for seminar attendance?(a) Payment(b) Approval(c) Enrollment(d) Registration.

188. When is the signup deadline for this week's seminar?

(a) March 18(b) March 20(c) March 25(d) March 27

189. Where will this week's seminar be held?(a) The Conference Room B(b) The employee training room(c) The Hoover conference room(d) The human resources meeting room

190. Which of the following employees does NOT work in the same office?(a) Cedric Green(b) Sally Cross(c) Lyle McKnight

Page 177: 101s1.downloadmienphi.net/file/downloadfile2/180/1403331.doc · Web viewDuring normal office hours, University Maintenance will respond directly to a power failure. After hours, the

(d) Misty Fitzgerald.

To: All EmployeesFrom: HR DepartmentSubject: Employee UniformsDate: April 28, 8:45 a.m.

Employee uniform orders for the summer quarter will be accepted beginning on May 1. Please remember the following rules when placing your order:1. New employees are allowed five shirts and two pants/skirts.2. Existing employees may order two shirts and one pant or skirt.3. Orders must be placed before the May 15 cutoff deadline.4. Incomplete order forms or those without the employee identification number will not be processed.5. Employee uniforms will be sent to the employee's home address.

----------------------------------------------------------------------------------------------------------------------------

191. When will ordering begin?(a) April 28(b) May 1(c) May 15(d) May 28

192. Where will employee uniforms be sent?(a) To the company(b) To the employee's home(c) To the corporate store(d) To the employee's office

193. What is required for order processing? (a) A method of payment(b) A supervisor's approval(c) An identification number(d) An employment verification form

194. Which item did Mr. Bishop order the most of?(a) Black pants(b) Blue shirts(c) White shirts(d) Navy Blue pants

Page 178: 101s1.downloadmienphi.net/file/downloadfile2/180/1403331.doc · Web viewDuring normal office hours, University Maintenance will respond directly to a power failure. After hours, the

195. What can be inferred about Mr. Bishop from the passages?(a) He is a manager.(b) He is a new employee.

(c) He is from Cleveland.(d) He likes the color white.

Taylor Beckett17 Montgomery DriveSt. Louis, MO 63141

Mr. Beckett,

Attached is a suggested itinerary for your June 12 trip to Athens. A representative from Mediterranean Travels will pick you up at the airport. A bilingual travel guide will be available to you during your trip, should you need one. The local contact number for our Athens office is 210-7281110.

Your trip package includes local transportation, travel guide access, luxury accommodations, and a twentyfour hour concierge to assist you with whatever you may need.

Please keep in mind that the itinerary is more of a suggestion than set plans, as you have the freedom and flexibility to go out on your own.

Thank you for traveling with Mediterranean Travels. We hope you enjoy your trip!

Hampton StoddardTravel ConciergeMediterranean Travels.------------------------------------------------------------------------------------------------------------------------------TravelJune 12 – DepartureDepart 2:32 p.m. St. LouisArrive 3:52 p.m. Chicago O'HareDepart 4:46 p.m. Chicago O'HareArrive 8:20 a.m. (+1) Munich, GermanyDepart 8:55 a.m. Munich, GermanyArrive 12:25 p.m. Athens, Greece

June 16 ReturnDepart 12:00 p.m. Athens, GreeceArrive 4:05 p.m. (+1) New York CityDepart 4:55 p.m. New York CityArrive 8:10 p.m. St. Louis

AccommodationJune 13-June 15Acropolis Grand HotelDeluxe Suite

Page 179: 101s1.downloadmienphi.net/file/downloadfile2/180/1403331.doc · Web viewDuring normal office hours, University Maintenance will respond directly to a power failure. After hours, the

Day 1: AthensAirport Transfer to hotelSyntagma, Syntagma Square, Vouli, Tomb of the Unknown SoldierPlaka, Vizantino

Day 2: AthensThe AcropolisThe Parthenon, The Erechtheion, Temple of Athena Nike

Day 3: Around AthensIsland-hoppingAegina, Boros, Hydra.

196. What will happen when Mr. Beckett's arrives in Athens?(a) He will go on a bus tour.(b) He will be taken to his hotel.(c) He will pick up his rental car.(d) He will meet a local travel agent.

197. What is included in the Athens trip package?(a) Sightseeing vouchers(b) Roundtrip train fare(c) Luxury accommodations(d) Chauffeured transportation

198. What is true about the itinerary of events? (a) It cannot be changed.

(b) It is custom-tailored.(c) It is already confirmed.(d) It is offered as a suggestion.199. Where is Mr. Beckett's longest layover?(a) In Athens(b) In Munich(c) In New York(d) In Chicago

200. How long is Mr. Beckett's vacation?(a) Four days(b) Five days(c) Six days(d) Seven days

ANSWER101. B 102. D 103. B 104. C 105. D 106. A 107. D 108. A 109. B 110. C

111. C 112. A 113. C 114. A 115. D 116. B 117. D 118. B 119. C 120. A

121. B 122. D 123. C 124. A 125. A 126. D 127. D 128. A 129. C 130. A

131. C 132. D 133. A 134. D 135. B 136. B 137. B 138. C 139. D 140. B

141. B 142. B 143. A 144. D 145. C 146. D 147. D 148. C 149. C 150. C

151. A 152. B 153. A 154. D 155. C 156. A 157. D 158. C 159. D 160. C

161. B 162. A 163. D 164. B 165. B 166. C 167. B 168. C 169. A 170. A

171. C 172. D 173. A 174. A 175. A 176. B 177. D 178. A 179. C 180. A

181. D 182. C 183. C 184. A 185. B 186. B 187. D 188. A 189. C 190. B

191. B 192. B 193. C 194. B 195. B 196. D 197. C 198. D 199. D 200. B